【在職研究生輔導(dǎo)資料】工程碩士復(fù)習(xí)資料 3

    發(fā)布人: 訪問量: 發(fā)布日期:2010-08-24 16:12:25

2010年GCT考試邏輯模擬附答案解析

 

 

 

(50題,每題2分,滿分100分)

  1.任何人都沒有吃過雅各島上的任何水果,所以無法知道雅各島上任何水果的口味。

  為了合乎邏輯的推出上述結(jié)論,需要假設(shè)下面哪項(xiàng)為前提?

  A.如果一種水果有人品嘗過,就可以知道其口味。(充分條件,不是前提)

  B.只憑某些人的品嘗無法真正知道某種水果的口味。

  C.要知道某種水果的口味,需要有人去品嘗。

  D.人們是通過嗅覺來確定水果口味的。

  解析:這道題需要補(bǔ)全假設(shè),結(jié)論是:任何人都無法知道任何水果的味道,前提是:任何人都沒有吃過雅各島上的任何水果。補(bǔ)充前提是:要知道水果的口味,需要有人去品嘗,應(yīng)該選擇C。選項(xiàng)A,是充分條件,但不是構(gòu)成結(jié)論的原因,屬于強(qiáng)拉因果;B、D都為不合理的假設(shè)。

  2.最近的一項(xiàng)研究指出:“經(jīng)常吃沙棘果對兒童的智力發(fā)育有益。”研究人員對560名兒童進(jìn)行調(diào)查,發(fā)現(xiàn)那些經(jīng)常吃沙棘果的兒童,其智力水平較很少吃沙棘果的兒童要高。因此,研究人員發(fā)現(xiàn)了沙棘果與兒童智力發(fā)育之間的聯(lián)系。

  以下哪項(xiàng)如果為真,最不可能削弱上述論證?

  A.對成年人的研究發(fā)現(xiàn),每天吃沙棘果的人智力水平并不比很少吃沙棘果人的高。

  B.調(diào)查顯示:沙棘果價(jià)格非常高,只有富裕家庭的兒童才經(jīng)常吃,同時(shí)這些家庭有條件實(shí)現(xiàn)兒童的早期智力開發(fā)。

  C.這項(xiàng)兒童發(fā)育研究的課題負(fù)責(zé)人是沙棘果生產(chǎn)商,其目的就是要擴(kuò)展沙棘果的銷售渠道。

  D.沙棘果是兒童喜歡的食品,家長經(jīng)常把沙棘果作為禮物獎(jiǎng)給智力表現(xiàn)優(yōu)異的孩子。

  解析:吃沙棘果與兒童智力發(fā)展是否存在必然聯(lián)系。通過樣本調(diào)查判斷出吃沙棘果與兒童智力發(fā)展存在正相關(guān)的關(guān)系。B選項(xiàng)說明家庭條件是否富裕與兒童智力發(fā)展存在關(guān)聯(lián)聯(lián)系;C選項(xiàng)說明題目中的結(jié)論是由于生產(chǎn)商的廣告產(chǎn)生的以偏概全的結(jié)論;D選項(xiàng)屬于因果倒置,成績優(yōu)異是因,吃沙棘果是結(jié)果。

  3.凡金屬都是導(dǎo)電的。銅是導(dǎo)電的,所以銅是金屬。

  下面哪項(xiàng)與上述推理結(jié)構(gòu)最相似?

  A.所有的鳥都是卵生動物,蝙蝠不是卵生動物,所以,蝙蝠不是鳥。

  B.所有的鳥都是卵生動物,天鵝是鳥,所以天鵝是卵生動物。

  C.所有從事工商管理的都要學(xué)習(xí)企業(yè)管理,老陳是學(xué)習(xí)企業(yè)管理的,所以,老陳是從事工商管理工作的。

  D.華山險(xiǎn)于黃山,黃山險(xiǎn)于泰山,所以華山險(xiǎn)于泰山。

  解析:題目結(jié)構(gòu)是所有的A具有某中屬性,B屬于A,因此B具有A的屬性,選項(xiàng)C與上述結(jié)論類似。

  4.為了有助于人們選擇最滿意的城市居住,有關(guān)部門實(shí)施了一項(xiàng)評選“最舒適城市”的活動。方法是,選擇十個(gè)方面,包括社會治安、商業(yè)設(shè)施、清潔程度、綠化程度、教育設(shè)施、旅游文化景點(diǎn)等等,每個(gè)方面按實(shí)際質(zhì)量的高低,評以1分至10分之間的某一分值,然后求得十個(gè)分值的平均數(shù)即是這個(gè)城市的舒適性指數(shù)。

  以下哪項(xiàng)是實(shí)施上述活動需要預(yù)設(shè)的前提?

  Ⅰ、城市的各種舒適性質(zhì)量程度都可以用準(zhǔn)確的數(shù)字表達(dá)。

  Ⅱ、城市的各種舒適性對于居民來說都是同等重要的。

 、、居民有自由選擇居住城市的權(quán)利并且大都樂于這樣做。

  A.僅Ⅰ。      B.僅Ⅲ。      C.僅Ⅰ和Ⅱ。      D.Ⅰ、Ⅱ和Ⅲ。

  解析:這道題是補(bǔ)充假設(shè)的問題,由于每個(gè)人對“舒適程度”的感覺是不一樣的,因此題干結(jié)論若要正確,條件Ⅱ是必須的前提;若題目中結(jié)論正確,需要假設(shè)題目中的十個(gè)指標(biāo)存在量化標(biāo)準(zhǔn),因此條件Ⅰ為必選項(xiàng);條件Ⅲ存在爭議,不過可以從另外一個(gè)角度理解,便于理解題目的結(jié)論,如果居民沒有選擇居住地的權(quán)利,那么題目中的滿意度調(diào)查就沒有意義,所以也得不到上述結(jié)論。

  5.衛(wèi)生部的官員們對牟定縣狂犬病疫情有以下斷定:

  (1)該縣所有的狗都得了狂犬病。

  (2)該縣有些斑點(diǎn)狗得了狂犬病。(真)

  (3)該縣有些狗得了狂犬病。

  (4)該縣有些狗沒得狂犬病。(真)

  其實(shí)上述斷定中只有兩個(gè)與事實(shí)相符。根據(jù)如上的情況,以下哪項(xiàng)結(jié)論是可能成立的?

  I.該縣的狗都是斑點(diǎn)狗。

  II. 該縣沒有斑點(diǎn)狗可能得狂犬病。

  III. 該縣的斑點(diǎn)狗并非可能沒有得狂犬病。

  IV. 該縣的狗不可能都沒得狂犬病。

  A.僅I      B.僅III      C.僅II和IV      D.僅IV。

  解析:這道題屬于邏輯推理中真假話題的問題。解析這種問題按照1、先找矛盾;2、繞開矛盾;3、推出答案的步驟進(jìn)行。首先先找矛盾,首先1和2是一對矛盾,3和4是一對矛盾,只有一個(gè)正確,2、繞開矛盾,如果1正確,那么3和4都假,因此只能2為真,如果2為真,只能判斷4為真,3為假,3、推出答案,條件2有些斑點(diǎn)夠得了狂犬病,可推出該縣沒有斑點(diǎn)狗可能得狂犬病,大家注意必然=沒有……必然;同時(shí)根據(jù)條件4可以得出該縣的狗不可能都沒得狂犬病。

  6.如果一個(gè)兒童的體重與身高的比值超過本地區(qū)80%兒童的水平,就稱其為肥胖兒。根據(jù)歷年的調(diào)查結(jié)果,15年來,臨江市的肥胖兒的數(shù)量一直在穩(wěn)定增長。

  如果以上斷定為真,則以下哪項(xiàng)也必為真?

  A.臨江市每一個(gè)肥胖兒的體重都超過全市兒童的平均體重。

  B.15年來,臨江市的兒童的體育鍛煉越來越不足。

  C.臨江市的非肥胖兒的數(shù)量15年來不斷增長。

  D.15年來,臨江市體重不足標(biāo)準(zhǔn)體重的兒童數(shù)量不斷下降。

  解析:這道題屬于迷惑推理題,大家務(wù)必要小心。首先大家可以看出臨江市兒童數(shù)量是在增長的;兒童由肥胖和非肥胖組成;肥胖兒童的數(shù)量穩(wěn)定增長,說明非肥胖兒童的數(shù)量也在穩(wěn)定增長;沒有證據(jù)表明非肥胖兒童的數(shù)量在不斷下降。

  7.孩子出生后的第一年在托兒所里度過,會引發(fā)孩子的緊張不安。在我們的研究中,有464名12~13歲的兒童接受了特異情景測試法的測驗(yàn),該項(xiàng)測驗(yàn)意在測試兒童1歲時(shí)的狀況與對母親的依附心理之間的關(guān)系。其結(jié)果:有41.5%曾在托兒所看護(hù)的兒童和25.7%曾在家看護(hù)的兒童被認(rèn)為緊張不安,過于依附母親。

  以下哪項(xiàng)如果為真,最沒有可能對上述研究的推斷提出質(zhì)疑?

  A.出生后第一年在家看護(hù)的孩子多數(shù)是由祖父母或外祖父母看護(hù)的,并形成濃厚的親情。

  B.這項(xiàng)研究的主持者被證實(shí)曾經(jīng)在自己的幼兒時(shí)期受到過長時(shí)間來自托兒所阿姨的冷落。

  C.針對孩子母親的另一部分研究發(fā)現(xiàn):由于孩子在家里表現(xiàn)出過度的依附心理,父母因此希望將其送入托兒所予以矯正。

  D.研究中所測試的孩子并不是從托兒所看護(hù)和在家看護(hù)兩種情況下隨機(jī)抽取的。因此,這兩組樣本兒童的家庭很可能有系統(tǒng)性的差異存在。

  解析:這種題最好的解決辦法是否定推斷,把你認(rèn)為可能的答案否定,帶入題目中判斷。如果能夠提出質(zhì)疑,說明你的判斷是正確的。A選項(xiàng)。

  8.在玉米地中套種的小麥有可能得小麥黃葉病,這種病是由于光照不足造成的,一旦光照充足,比如減少玉米植株數(shù)量,或者玉米收割后,癥狀很快就會消失,且不會影響產(chǎn)量。而小麥銹病是由花斑銹菌引起,如果在病癥初起時(shí)不及時(shí)恰當(dāng)應(yīng)對,就會使小麥植株很快枯死。由于小麥銹病和小麥黃葉病初期癥狀很難區(qū)分,所以,在玉米地中套種小麥,必須高度注意小麥銹病的定期篩查和預(yù)防。

  以下哪項(xiàng)最可能是上述論證所假設(shè)的?

  麥黃葉病不會誘發(fā)小麥銹病。

  B.花斑銹菌不會因?yàn)槌渥愎庹斩π←湹那趾αΑ?/p>

  C.小麥銹病如果及時(shí)恰當(dāng)處理,就不會使小麥減產(chǎn)。

  D.科學(xué)家一直沒有找到有效遏制小麥銹病的藥物。

  解析:假設(shè)推斷。陽光充足,小麥黃葉病會消失,產(chǎn)量不影響;花斑銹菌,使小麥植株枯死。因此需要假設(shè)陽光充足不會抑制花斑銹菌的發(fā)病,才能得出小麥黃葉不發(fā)病時(shí),花斑銹菌仍然會發(fā)病,導(dǎo)致兩種病情難以區(qū)分。換一種角度,如果小麥黃葉病不會和花斑銹菌同時(shí)發(fā)病,那么就不存在難以區(qū)分病情的問題了。

  9.公司準(zhǔn)備從五個(gè)業(yè)務(wù)骨干中選幾個(gè)人去中央戲劇學(xué)院進(jìn)修。鑒于業(yè)務(wù)員們的工作關(guān)系,總經(jīng)理建議:

 。1)如果選鞏俐,那么必須選章子怡并且不能選周迅。

  (2)如果選章子怡或者選周迅,則不能選吳孟達(dá)。

 。3)不能既不選周星馳也不選吳孟達(dá)。

  考慮總經(jīng)理的建議,董事會認(rèn)為,鞏俐必須去中央戲劇學(xué)院進(jìn)修,這樣誰將跟她一起去進(jìn)修?

  A.章子怡和周星馳    B.章子怡和吳孟達(dá)   C.周星馳和周迅    D.吳孟達(dá)和周迅

  解析:邏輯推斷。如果鞏麗必須去,根據(jù)條件1章子儀必須去,而周迅一定不去,從而排除C、D選項(xiàng),根據(jù)條件2,如果選章子儀,那么吳孟達(dá)不能去。排除B 。

  繼續(xù)進(jìn)行推理,根據(jù)條件3必須在周星馳和吳孟達(dá)中選一個(gè),因?yàn)楦鶕?jù)條件2已經(jīng)排除吳孟達(dá),所以周星馳必須去。選擇A。

  10.20世紀(jì)60年代早期之前,挪威的斯塔溫格是一個(gè)安靜而和平的小鎮(zhèn)。進(jìn)入60年代以來,這里成為了挪威近海石油勘探中心。從此,暴力犯罪和毀壞公物的現(xiàn)象在斯塔溫格小鎮(zhèn)急劇增加。顯然這些社會問題是是石油繁榮的副產(chǎn)品。

  下列哪一個(gè)選項(xiàng)最強(qiáng)烈的支持上面的論述。

  A.暴力犯罪和毀壞公物在沒有石油繁榮的挪威城鎮(zhèn)保持低水平。

  B.斯塔溫格居民很少對這里是近海石油勘探中心感到遺憾。

  C.挪威社會學(xué)家對斯塔溫格的暴力犯罪和毀壞公物現(xiàn)象持續(xù)增加表示關(guān)切。

  D.非暴力犯罪、毒品、離婚,在這個(gè)小鎮(zhèn)也以同樣的速度增加著。

  解析:題目把社會問題的原因歸結(jié)為石油繁榮,最強(qiáng)烈支持的論據(jù)為在沒有石油繁榮的時(shí)期,社會問題處于低水平。因此選A。

11.認(rèn)為只傷害自己而沒有傷害到別人的行為并沒有什么錯(cuò)誤的想法,通常伴隨著對人與人之間相互依賴的人際關(guān)系的忽視。毀壞一個(gè)人自己的健康或者生命意味著不能你將不能為家庭或者社會提供幫助,相反意味著你將額外享有那些本來就有限的諸如食物、健康服務(wù)和教育等社會資源,而不是相反將它們回報(bào)給社會。

  以下哪個(gè)選項(xiàng)如果正確,最強(qiáng)烈的地支持題干所表達(dá)的觀點(diǎn)。

  A.本來可以避免的疾病和意外事故所造成的花銷增加了每個(gè)人的健康保險(xiǎn)金。

  B.傷害一個(gè)人能夠?qū)е麻g接的利益,諸如可以使別人獲得與健康領(lǐng)域相關(guān)聯(lián)的工作。

  C.一個(gè)人對社會所作的貢獻(xiàn)可以通過他的健康程度來衡量。

  D.由喝酒、吸煙和吸食毒品所導(dǎo)致的主要傷害是由使用那些東西的人來承受的。

  解析:題目中的結(jié)論是傷害自己的行為與傷害別人的行為同樣都是錯(cuò)誤的。原因是傷害自己將不能為家庭和社會提供幫助,同時(shí)會向社會索取的更多。這道題的關(guān)鍵是大家要讀懂題干,這樣從選項(xiàng)中我們不難選出A,本來可以避免的個(gè)人傷害回增加健康保險(xiǎn)金,從而增加社會的負(fù)擔(dān)。

  12.大多數(shù)道路的修理比預(yù)算的要花費(fèi)更多的時(shí)間和金錢,但是去年夜間修理京津塘高速公路和類似的道路并未比預(yù)算花費(fèi)更多的時(shí)間或金錢。因此,在夏季,夜間修理主要道路可能省時(shí)省錢。

  下列哪一個(gè),假如正確,最支持上面得到的結(jié)論?

  A.修理京津塘高速公路的預(yù)算足夠用,所以不可能超過預(yù)算。

  B.夏季,夜間路上通行車輛較少,而且溫度較舒適,允許修路工人工作得更快。

  C.愿意在晚上工作的修路工人較容易找到工作機(jī)會,因?yàn)榇蠖鄶?shù)人寧愿白天工作。

  D.用于道路修理的瀝青在較高溫度下膨脹,在溫度降低的時(shí)候收縮。

  解析:補(bǔ)充論據(jù)。從題干可以看出從特殊樣本推出結(jié)論明顯論據(jù)不足。夏天夜間施工本來和省時(shí)省錢本來沒有關(guān)系,屬于不相關(guān)關(guān)系,因此需要補(bǔ)充適當(dāng)?shù)恼摀?jù),在這兩個(gè)問題中間搭起橋梁,建立關(guān)系,所以選擇B。

  13.偏頭痛一直被認(rèn)為是由食物過敏引起的。但是,如果讓患者停止食用那些已經(jīng)證明會不斷引起過敏性偏頭痛的食物,他們的偏頭痛并沒有停止,因此,顯然存在別的某種原因引起偏頭痛。下列哪項(xiàng)如果為真,最能削弱上面的結(jié)論?

  A.許多患者說誘發(fā)偏頭痛病的那些食物往往是他們最喜歡吃的食物。

  B.許多普通食物只在食用幾天后才誘發(fā)偏頭痛,因此,不容易觀察患者的過敏反應(yīng)和他們食用的食物之間的關(guān)系。

  C.很少有食物過敏會引起像偏頭痛那樣嚴(yán)重的癥狀。

  D.許多不患偏頭痛的人同樣有食物過敏反應(yīng)。

  解析:題目結(jié)論是過敏引發(fā)偏頭痛,但是引發(fā)過敏的問題不僅僅是食物引起的。結(jié)論指明食物過敏與偏頭痛之間不存在這因果關(guān)系,原因是停用了這種食物之后,偏頭痛的病情仍然存在,若要削弱這個(gè)結(jié)論,需要針對上述論據(jù)提出反對。本著這個(gè)思路我們看選項(xiàng),結(jié)論B。表明了停用食物之后仍然存在偏頭痛的原因是因?yàn)樵S多普通食物在食用幾天后才發(fā)作,因此雖然停用了食物,仍會發(fā)作一段時(shí)間。

  14.世界衛(wèi)生組織在全球范圍內(nèi)進(jìn)行了一項(xiàng)有關(guān)獻(xiàn)血對健康的影響的跟蹤調(diào)查。調(diào)查對象分為三組。第一組對象中均有二次以上的獻(xiàn)血記錄,其中最多的達(dá)數(shù)十次;第二組中的對象均僅有一次獻(xiàn)血記錄;第三組對象均從未獻(xiàn)過血。調(diào)查結(jié)果顯示,被調(diào)查對象中癌癥和心臟病的發(fā)病率,第一組分別為0.3%和0.5%,第二組分別為0.7%和0.9%,第三組分別為1.2%和2.7%。一些專家依此得出結(jié)論,獻(xiàn)血有利于減少患癌癥和心臟病的風(fēng)險(xiǎn)。這兩種病已經(jīng)不僅在發(fā)達(dá)國家而且在發(fā)展中國家成為威脅中老年人生命的主要?dú)⑹。因此,獻(xiàn)血利己利人,一舉兩得。以下哪項(xiàng)如果為真,將削弱以上結(jié)論?

  I 60歲以上的調(diào)查對象,在第一組中占60%,在第二中占70%,在第三組中占80%。

  II 獻(xiàn)血者在獻(xiàn)血前要經(jīng)過嚴(yán)格的體檢,一般具有較好的體質(zhì)。

  III 調(diào)查對象的人數(shù),第一組為1700人,第二組為3000人,第三組為7000人。

  A.只有I和II。    B.只有II和III。    C.只有I和III    D.只有I

  解析:題目中對獻(xiàn)血與癌癥、心臟病的發(fā)病幾率建立了正相關(guān)關(guān)系。但是樣本仍然存在問題,如果鮮血的人本身身體素質(zhì)就比沒有獻(xiàn)血的身體素質(zhì)好,就不能退出上述結(jié)論。這是最簡單容易想出的悖論。因此淘汰C、D。下面在條件1和3中選一個(gè),條件3說明樣本容量不同,但是題目中使用的是百分?jǐn)?shù)這個(gè)相對指標(biāo),因此不能形成反對結(jié)論的悖論。選A。

  15.許多孕婦都出現(xiàn)了維生素缺乏的癥狀,但這通常不是由于孕婦的飲食中缺乏維生素,而是由于腹內(nèi)嬰兒的生長使她們比其他人對維生素有更高的需求。為了評價(jià)上述結(jié)論的確切程度,以下哪項(xiàng)操作最為重要?

  A.對某個(gè)不缺乏維生素的孕婦的日常飲食進(jìn)行檢測,確定其中維生素的含量。

  B.對孕婦的科學(xué)食譜進(jìn)行研究,以確定有利于孕婦攝入足量維生素的最佳選擇。

  C.對日常飲食中維生素足量的一個(gè)孕婦和一個(gè)非孕婦進(jìn)行檢測,并分別確定她們是否缺乏維生素。

  D.對日常飲食中維生素不足量的一個(gè)孕婦和另一個(gè)非孕婦進(jìn)行檢測,并分別確定她們是否缺乏維生素。

  解析:支持論據(jù)。題目需要根據(jù)結(jié)論找出支持論據(jù),顯然對同量的維生素給孕婦和非孕婦服用,觀察他們是否缺乏維生素,對得出結(jié)論最重要。選C

  16.經(jīng)A省的防疫部門檢測,在該省境內(nèi)接受檢疫的長尾猴中,有1%感染上了狂犬病。但是只有與人及其寵物有接觸的長尾猴才接受檢疫。防疫部門的專家因此推測,該省長尾猴中感染有狂犬病的比例,將大大小于1%。

  以下哪項(xiàng)如果為真,將最有力地支持專家的推測?

  A.在A省境內(nèi),與人及其寵物有接觸的長尾猴,只占長尾猴總數(shù)的不到10%。

  B.在A省,感染有狂犬病的寵物,約占寵物總數(shù)的0.1%。

  C.在與A省毗鄰的B省境內(nèi),至今沒有關(guān)于長尾猴感染狂犬病的疫情報(bào)告。

  D.與健康的長尾猴相比,感染有狂犬病的長尾猴更愿意與人及其寵物接觸。

  解析:選擇D,其他選項(xiàng)均不能支持題干給出的結(jié)論。在與人和動物接觸的長尾猴進(jìn)行檢驗(yàn),發(fā)現(xiàn)1%感染了狂犬病,推斷出該省長尾猴感染狂犬病的比例小于1%,那么只有感染狂犬病的長尾猴更喜歡與人和動物接觸能夠有效地支持這個(gè)結(jié)論。

  17.作為市電視臺的攝像師,最近國內(nèi)電池市場的突然變化讓我非常頭疼。進(jìn)口電池缺貨,我只能用國產(chǎn)電池來代替作為攝像的主要電源。盡管每單位的國產(chǎn)電池要比進(jìn)口電池便宜,但我估計(jì)如果持續(xù)用國產(chǎn)電池替代進(jìn)口電池來提供同樣的電源供應(yīng)的話,我在能源上的支付將會提高。

  說這番話的人在上面這段話中隱含了以下哪項(xiàng)假設(shè)?

  A. 持續(xù)使用國產(chǎn)電池,攝像的質(zhì)量將無法得到保障。

  B. 每單位的進(jìn)口電池要比國產(chǎn)電池價(jià)格貴。

  C. 生產(chǎn)國產(chǎn)電池要比生產(chǎn)進(jìn)口電池成本低。

  D. 以每單位電池提供的電能來計(jì)算,國產(chǎn)電池要比進(jìn)口電池提供得少。

  解析:這是日常生活中經(jīng)常會遇到的問題,答案一目了然選擇D。得出結(jié)論的原因是盡管國產(chǎn)電池比進(jìn)口電池單價(jià)便宜,但是就單位電能來講,國產(chǎn)電池的價(jià)格卻比進(jìn)口電池高。

  18.最近由于氣候的異常變化,使得山東省的大蒜產(chǎn)量受到較大影響而減產(chǎn)。山東大蒜的價(jià)格比平時(shí)同期上漲了兩倍,這就大大提高了大蒜素生產(chǎn)的成本,估計(jì)未來大蒜素的市場價(jià)格將有大幅度的提高。以下哪項(xiàng)如果是真的,最能削弱上述結(jié)論?

  A.去年大蒜的價(jià)格是歷年最低的。

  B.其他替代原料可以用來生產(chǎn)人工大蒜素。

  C.除了山東省外,其他省份也可以提供大蒜。

  D.最近的天氣異常不如專家們估計(jì)的那么嚴(yán)重。

  解析:題目中的推斷是沒有問題的,成本的提高會促使價(jià)格的提高,但是其他省也提供大蒜,而成本并沒有提高,價(jià)格不會上升,在這樣的市場上,山東省的大蒜盡管成本提高了,但是價(jià)格將會保持市場水平,因此C有效地消弱了結(jié)論。

  19.某學(xué)校開設(shè)選修課,規(guī)定:同學(xué)只有選了文化史課的同學(xué)才能選古典音樂課。如果選古典音樂課,那他就不能選文學(xué)欣賞課。如果選文學(xué)欣賞課,就不能選古典音樂課。

  如果以上陳述為真,以下哪項(xiàng)陳述不可能假?

  A.同學(xué)不選文學(xué)欣賞課 或者不選古典音樂課。

  B.同學(xué)選文學(xué)欣賞課 或者不選古典音樂課。

  C.同學(xué)不選古典音樂課,或者不選文化史課。

  D.同學(xué)選古典音樂課,或者不選文學(xué)欣賞課

  解析:邏輯推斷題。不可能假=一定真,因此題目要求在選項(xiàng)中找出一定真的項(xiàng)。選了文化史,才能選古典音樂,這是個(gè)必要條件;但是選了古典音樂就不能選文學(xué)欣賞,二選其一;選項(xiàng)A的論述與題干一樣。

  20.20年前,幾乎所有的公司首腦在選擇重新設(shè)置公司總部的時(shí)候,主要關(guān)心的是土地價(jià)格。今天一個(gè)高級執(zhí)行官計(jì)劃重設(shè)總部時(shí)主要關(guān)心的東西更廣泛了,經(jīng)常包括當(dāng)?shù)氐膶W(xué)校和住房質(zhì)量。

  假如上述信息可靠,下面哪一項(xiàng)最好地解釋了公司首腦們主要關(guān)心問題的變化?

  A.20年前高質(zhì)量的住房和學(xué)校像今天一樣難以發(fā)現(xiàn)。

  B.近年來優(yōu)秀專業(yè)辦公人員缺乏的問題迫使公司找到盡可能多的方法來留住老員工,并且吸引優(yōu)秀新員工的加盟。

  C.公司執(zhí)行官總是考慮自己的決定將怎樣影響公司的利潤。

  D.在過去20年,一些地區(qū)比其他地區(qū)土地價(jià)格變化少。

  解析:選項(xiàng)A屬于強(qiáng)拉因果,不存在相關(guān)關(guān)系;選項(xiàng)B說明了高級執(zhí)行官在公司選址時(shí)關(guān)注學(xué)校和住房質(zhì)量的原因,分別是吸引優(yōu)秀新員工加盟和留住老員工;選項(xiàng)C與題干沒有關(guān)系,并未涉及到利潤這個(gè)因素;選項(xiàng)D,最多可以說明高級執(zhí)行官不在關(guān)心土地價(jià)格的原因,但是并不能說明關(guān)注學(xué)校和住房質(zhì)量的原因。

21.用蒸餾麥芽渣提取的酒精作為汽油的替代品進(jìn)入市場,使得糧食市場和能源市場發(fā)生了前所未有的直接聯(lián)系。到1995年,谷物作為酒精的價(jià)值己經(jīng)超過了作為糧食的價(jià)值。西方國家已經(jīng)或正在考慮用從谷物提取的酒精來替代一部分進(jìn)口石油。

  如果上述斷定為真,對于那些己經(jīng)用從谷物提取的酒精來替代一部分進(jìn)口石油的西方國家。以下哪項(xiàng),最可能是1995年后進(jìn)口石油價(jià)格下跌的后果?

  A.一些谷物從能源市場轉(zhuǎn)入糧食市場。

  B.一些谷物從糧食市場轉(zhuǎn)入能源市場。

  C.谷物的價(jià)格面臨下跌的壓力。

  D.谷物的價(jià)格出現(xiàn)上浮。

  解析:谷物可以提取出酒精來代替石油出口,因此可以判斷谷物和石油成為了相互競爭的替代品,因此如果石油價(jià)格下跌,那么谷物的價(jià)格也必然會下調(diào),否則進(jìn)口國將會進(jìn)口價(jià)格低廉的石油取代谷物。

  22.有八個(gè)教授參加學(xué)術(shù)會議后,互留通信地址,以保持聯(lián)絡(luò)。一年后,統(tǒng)計(jì)他們之間的通信情況如下:有一人給其他三個(gè)人寫過信,給其他人寫過兩封信的有三人,給其他人寫過一封信的有四個(gè)人。若以上統(tǒng)計(jì)屬實(shí),則最能得出以下哪項(xiàng)結(jié)論?

  A.通過書信來往,他們遞進(jìn)了友誼,也加強(qiáng)了學(xué)術(shù)聯(lián)系。

  B.八個(gè)教授都收到過其他教授的來信。

  C.至少有一個(gè)人收到信件后沒有都回復(fù)。

  D.雖然參加學(xué)術(shù)會議的時(shí)候大家不熟悉,但現(xiàn)在他們已經(jīng)成為至交。

  解析:選項(xiàng)首先排除A、D,沒有任何證據(jù)表明通過書信加強(qiáng)了學(xué)術(shù)聯(lián)系,也沒有證據(jù)表明他們成為了至交。下面看選項(xiàng)B、C,我們可以通過畫圖的方法來解決這個(gè)問題相對比較簡單,從而得出正確答案為C。

  23.金鳳扒雞是一個(gè)享有盛譽(yù)的百年扒雞品牌,在石家莊銷路很好,深受當(dāng)?shù)匕傩障矏,每天一上市就會被搶購一空。為了打開北京市場,金鳳扒雞改進(jìn)了工藝,用真空包裝,延長了保質(zhì)期,試圖在北京各大超市打開銷路,但是金鳳扒雞在北京卻很難銷售出去。

  下面除了哪一項(xiàng),都有助于解決上述現(xiàn)象?

  A.北京人和石家莊人的飲食習(xí)慣不同。他們更喜歡吃烤鴨而不是扒雞。

  B.恒慧通燒雞也采用真空包裝,保質(zhì)期一樣長,在北京的銷路卻很好。

  C.人們喜歡吃剛出鍋的新鮮扒雞,保質(zhì)期短的反而更受消費(fèi)者青睞。

  D.雖然金鳳扒雞是百年品牌,但是它的銷售地點(diǎn)僅在石家莊市區(qū),北京人對這個(gè)品牌不熟悉。

  解析:這是一道頗具爭議的題目,選項(xiàng)A、B、C、D分別從消費(fèi)者的喜好,競爭者,消費(fèi)偏好和品牌對金鳳扒雞在北京銷路受阻提供了良好的解釋。但是選項(xiàng)B可以從另外一個(gè)角度思考,恒惠通燒雞同樣采用真空包裝,保質(zhì)期一樣長,什么都和金鳳扒雞一樣,為什么銷路很好,而金鳳扒雞為什么銷路不好,這就不能解釋為什么金鳳扒雞在北京銷路不好的原因。

  24.中國自1978年改革開放以來28年間,從人口數(shù)量上說:農(nóng)民的數(shù)量并沒有減少,而是持續(xù)增加的,只不過這種增長速度相對于總?cè)丝诘脑鲩L速度來說,是比較低的。

  下列那一句話直接與上述信息矛盾?

  A.在總?cè)丝谥修r(nóng)民的數(shù)量在1978年到2006年間略微增長了。

  B.農(nóng)民在總?cè)丝谥械谋嚷蕪?978年的81.4%增加到了2006年的85.3%。

  C.中國勞動力的增長率和總?cè)丝诘脑鲩L率在1978年至今的28年里同時(shí)增加了。

  D.中國的勞動力中,農(nóng)民的比率從1978年的78%下降到了75.6%。

  解析:選項(xiàng)B可以看出,農(nóng)民相對總?cè)丝诘脑鲩L速度并不低,增長速度也是增加的。

  25.有三戶人家,每家有一孩子,他們的名字是:小萍(女)、小紅(女)、小虎。孩子的爸爸是老王、老張和老陳;媽媽是劉蓉、李玲和方麗。對于這三家人,已知:

 。1)老王家和李玲家的孩子都參加了少女舞蹈隊(duì)。

 。2)老張的女兒不是小紅。

 。3)老陳和方麗不是一家人。

  根據(jù)以上條件,確定以下哪項(xiàng)是正確的?

  A.老王、劉蓉和小萍是一家。

  B.老張、李玲和小紅是一家。

  C.老王、方麗和小紅是一家。

  D.老陳、方麗和小虎是一家。

  解析:邏輯推斷題。從條件1可以看出小虎不是老王和李玲的孩子,因?yàn)樾』⑹悄泻。那么可以推出小萍和小紅的爸爸可能是老王。根據(jù)條件2,老張的女兒不是小紅,所以可以確定老王的女兒是小萍。鎖定選項(xiàng)A,如果A正確,那么根據(jù)條件3老陳和李玲是一家,根據(jù)條件1,女兒是小紅;得出老陳、方麗和小虎是一家。那么選項(xiàng)D也正確。因此同時(shí)排除A、D。選項(xiàng)B與條件2不符,因此選擇C。

  26.5名犯罪嫌疑人被警察詢問,事后得知其中一名是罪犯。下面是五個(gè)嫌疑人的供述,其中只有3句真話。問,誰是罪犯?

  甲:丁是罪犯。

  乙:我是無辜的。

  丙:戊不是罪犯。

  。杭自谌鲋e。

  戊:乙說的是實(shí)話。

  A.戊    B.甲和丙    C.丁    D. 乙和戊

  解析:從選項(xiàng)入手,選項(xiàng)B,甲和丙是罪犯,可以推出乙、戊、丁、丙都為真,而題目表明只有3項(xiàng)為真,所以放棄B;選項(xiàng)C,丁是罪犯,可以推出甲、乙、丙、戊為真,同樣與題目要求不符;選項(xiàng)D,乙和戊是罪犯,只能推出丁為真。因此這道題選擇A。

  27.學(xué)生上完體育課后回到教室,有15人喝了飲水機(jī)里的純凈水,其中5人很快產(chǎn)生了腹瀉。飲水機(jī)里的純凈水馬上被送去檢驗(yàn),檢驗(yàn)的結(jié)果不能肯定其中有造成腹瀉的有害物質(zhì)。因此,喝了飲水機(jī)里的純凈水不是造成腹瀉的原因。

  如果上述檢驗(yàn)結(jié)果是正確的,則以下哪項(xiàng)對上述論證的評價(jià)最為恰當(dāng)?

  A.題干的論證有漏洞,因?yàn)樗鼪]有考慮到另一個(gè)事實(shí):哪些沒有喝了飲水機(jī)里的純凈水的人沒有造成腹瀉。

  B.題干的論證有漏洞,因?yàn)樗讶鄙僮C據(jù)證明某種情況存在,當(dāng)作有充分證據(jù)證明某種情況不存在。

  C.題干的論證有漏洞,因?yàn)樗鼪]有利用一個(gè)有力的論據(jù):為什么有更多人喝了飲水機(jī)里的純凈水沒有造成腹瀉。

  D.題干的論證有漏洞,因?yàn)樗鼪]有指出造成腹瀉的真正原因。

  解析:理解這道題非常簡單,舉個(gè)例子說明,如果一個(gè)人確實(shí)殺了人,但是法官沒有證據(jù)表明他是兇手,那么可能無法定罪,但終不能得出他不是兇手的結(jié)論,因?yàn)橥瑯記]有證據(jù)表明他不是兇手。題目中給出了同樣的問題,檢驗(yàn)結(jié)果不能肯定有造成腹瀉的有害物質(zhì),但是并不能說明純凈水不是造成腹瀉的原因。

  28.一項(xiàng)全球范圍的調(diào)查顯示,近 10年來:吸煙者的總數(shù)基本保持不變;每年只有10%的吸煙者改變自己的品牌,即放棄原有的品牌而改吸其他品牌;煙草制造商用在廣告上的支出占其毛收入的10%。在Z煙草公司的年終董事會上,懂事A認(rèn)為,上述統(tǒng)計(jì)表明,煙草業(yè)在廣告上的收益正好等于其支出,因此,此類廣告完全可以不做。以下哪項(xiàng),構(gòu)成對懂事A的結(jié)論的最有力質(zhì)疑?

  A.懂事A的結(jié)論忽視了:今年來各種品牌的香煙的價(jià)格都有了很大的變動。

  B.懂事A的結(jié)論基于一個(gè)錯(cuò)誤的假設(shè):每個(gè)吸煙者在某個(gè)時(shí)候只喜歡一種品牌。

  C.懂事A的結(jié)論基于一個(gè)錯(cuò)誤的假設(shè):每個(gè)煙草制造商只生產(chǎn)一種品牌。

  D.懂事A的結(jié)論忽視了:世界煙草業(yè)是一個(gè)由處于競爭狀態(tài)的眾多經(jīng)濟(jì)實(shí)體組成的。

  解析:董事A的判斷為廣告的支出正好與收益相等,因此得出廣告可以不做的結(jié)論。但是忽略了如果不做廣告是否能夠獲得與做廣告時(shí)除去廣告費(fèi)用后相同的收益,因?yàn)檫@部分收入也是由廣告產(chǎn)生的效益。因?yàn)橥袠I(yè)的其他競爭者會因?yàn)閺V告,增加10%的顧客,而不做廣告的煙草公司將有減少10%顧客的風(fēng)險(xiǎn)。因此選擇D

  29.在收款局的5個(gè)賬單收款員中,楊先生收款的不成功率最高。然而楊先生是這個(gè)局的職員中最好的賬單收款員。

  下面哪一項(xiàng),如果正確,最有助于解決上述短文中的明顯分歧?

  A.在加入收款局之前,楊先生是一個(gè)大百貨公司的信貸部的一名職員。

  B.收款局中的其他四個(gè)收款員都認(rèn)為楊先生是一個(gè)非常能干的帳單收款員。

  C.楊先生在過去的幾年內(nèi),每年收款成功的比率都保持相當(dāng)?shù)胤(wěn)定。

  D.這個(gè)收款局的大多數(shù)最困難的事情都是派楊先生去做的。

  解析:從題干入手分析,楊先生是最好的賬單收款員,但是收款不成功率最高。題目要求找到解釋這一矛盾的原因。顯然D最合適。

  30.在一次工程碩士入學(xué)考試中,據(jù)統(tǒng)計(jì),在英語、數(shù)學(xué)、語文和邏輯四個(gè)科目中:70%的考生英語及格了,75%的考生數(shù)學(xué)及格了,80%的考生語文及格了,85%的考生邏輯及格了。如果以上情況屬實(shí),那么,以下哪個(gè)選項(xiàng)一定正確:

  A.在此次考試中,大部分考生四門課都及格了。

  B.在此次考試中,大部分考生四門課沒有都及格。

  C.在此次考試中,有不少于10%的考生,四門課程都及格了。

  D.在此次考試中,至少有10%的考生,四門課程都沒及格。

  解析:不少于=大于等于,至少=大于等于。選線排除A、B,選項(xiàng)過于籠統(tǒng),并且與C、D存在重復(fù)關(guān)系。選項(xiàng)D與題干明顯不符合,選擇C。

 31.某游戲的游戲規(guī)則有這樣一條規(guī)定:任何在游戲中拒絕成為選手的人將在拒絕的時(shí)候被減掉10分。

  下列哪一個(gè)是此條規(guī)則所暗含的?

  A.同意參加游戲的所有人的得分將高于那些因?yàn)榇艘?guī)則而被判罰分的人。

  B.一個(gè)人在拒絕玩這個(gè)游戲的同時(shí),依然是這個(gè)游戲的參與者。

  C.最初同意成為游戲參加者然后在游戲進(jìn)行中退出的人可以避免被罰10分。

  D.一個(gè)拒絕玩這個(gè)游戲的人不能在游戲中被宣布為失敗者。

  解析:由于拒絕玩這個(gè)游戲,會被剪掉20分,說明依舊是游戲的參與者,選B。

  32.受到廣泛贊揚(yáng)的斯坦尼斯拉夫斯基演員訓(xùn)練法來自他做年輕演員時(shí)對戲劇不熟練和易受影響的老套做法。對該方法的理解必須基于斯坦尼斯拉夫斯基個(gè)人的演藝研究,從而從典型化的姿勢、反復(fù)嘗試的嗓音語調(diào)和標(biāo)準(zhǔn)的感情模式的誘惑中解脫出來。盡管他的美國門徒有過要求,但是這位俄國導(dǎo)演從未打算寫一本對表演問題提出嚴(yán)格解決方法的教科書。

  關(guān)于表演這個(gè)問題,上文作者對斯坦尼斯拉夫斯基的演員訓(xùn)練法持有怎樣的觀點(diǎn)?

  A.斯坦尼斯拉夫斯基的美國門徒,在使用他的方法訓(xùn)練時(shí),使得這種方法喪失了原有的靈活性和探索性。

  B.表演本質(zhì)上是自發(fā)的感情表達(dá),系統(tǒng)的訓(xùn)練經(jīng)常干涉它。

  C.實(shí)際上唯一需要給予年輕演員的建議是他們必須系統(tǒng)地抵制他們在戲劇表演中所表現(xiàn)出來的那些老套的模式。

  D.斯坦尼斯拉夫斯基的方法主要是針對那些必須克服有做作和不成熟表演行為的年輕演員。

  解析:斯坦尼斯拉夫斯基之所以未打算對表演問題提出嚴(yán)格解決方法的教科書,本意是不想使該方法失去靈活性和探索性。因此選擇A。

  33.已發(fā)現(xiàn)有國產(chǎn)嬰幼兒奶粉三聚氰胺含量超標(biāo)。

  如果上述斷定是真的,那么在下述三個(gè)斷定中不能確定真假的是:

 、、國產(chǎn)嬰幼兒奶粉沒有三聚氰胺含量不超標(biāo)的。

 、颉⒂袊a(chǎn)嬰幼兒奶粉三聚氰胺含量沒超標(biāo)。

 、蟆⑺袊a(chǎn)嬰幼兒奶粉三聚氰胺含量都未超標(biāo)。

  A.只有Ⅰ和Ⅱ。    B.Ⅰ、Ⅱ和Ⅲ。    C.只有Ⅰ和Ⅲ。    D.只有Ⅱ。

  解析:選項(xiàng)1不能確定真假,從部分不能推出整體;選項(xiàng)2不能確定真假,從一部分不能退出另一部分具有相反的結(jié)論。條件3明顯可以判定為假,選擇A。

  34.股民購買股票的目的就是盈利。股民購買股票后,股票價(jià)格上漲,就會盈利,否則,就會虧損。一般情況下,盈利或者虧損達(dá)到10%就應(yīng)該拋出,以避免那些不可預(yù)測的突然變化所引起的損失。在股市上,有經(jīng)驗(yàn)的股民往往會選擇跟莊戰(zhàn)術(shù)來買賣股票。即大股東買進(jìn)的時(shí)候,他們也買進(jìn),大股東拋出的時(shí)候,他們就拋出。因?yàn)榇蠊蓶|有可能掌握更多上市公司和市場行情方面的信息。這些天,天虹公司的股票一直在下跌,但是大股東們卻一直在大量購進(jìn)這只股票。

  以上事實(shí)最能支持以下哪種預(yù)測?

  A.天虹公司的股票價(jià)格會繼續(xù)下降,但速度會放慢。

  B.天虹公司的股票價(jià)格會在短期內(nèi)大幅度上漲。

  C.股民盲目跟莊不是一個(gè)好的盈利方法。

  D.最近大股東對天虹公司股價(jià)變動判斷失誤,有可能造成巨大經(jīng)濟(jì)損失。

  解析:題目結(jié)論如下:大股東買進(jìn)的股票會漲,因?yàn)樗麄冋莆樟烁嗌鲜泄竞褪袌鲂星榉矫娴男畔ⅰK赃x擇B。

  35.作為身體的精密平衡系統(tǒng)的一部分,人類心臟分泌某種激素,這些激素能夠控制血液中的含鹽量和體內(nèi)參與循環(huán)的血量。人體僅需要極少的這種激素。它在控制血壓時(shí)極其重要,并且在患心臟病的病人血中經(jīng)常檢出。

  基于上面的研究成果,下面的哪個(gè)論點(diǎn)一定正確?

  A.假如心臟分泌的這種激素缺乏,將導(dǎo)致低血壓。

  B.一個(gè)僅僅由機(jī)械閥制作的設(shè)備被用作人工心臟,它將不能執(zhí)行人類心臟的所有功能。

  C.少量這種激素的分泌對人體的影響是長期性的。

  D.任何控制血壓的藥將通過影響心臟此種激素分泌量達(dá)到效果。

  解析:選擇B,因?yàn)橛蓹C(jī)械閥制作的人工心臟,顯然不具有分泌激素的功能,因此就不能執(zhí)行人類心臟的所有功能,題干反復(fù)說明就是為了得出這樣的結(jié)論。

  36.某出版社近年來出版物的錯(cuò)字率較前幾年有明顯的增加,引起了讀者的不滿和有關(guān)部門的批評,這主要是由于該出版社大量引進(jìn)非專業(yè)編輯所致。當(dāng)然,近年來該社出版物的大量增加也是一個(gè)重要原因。

  上述議論的漏洞,也類似地出現(xiàn)在以下哪項(xiàng)中?

  I 美國航空公司近兩年來的投訴率比前幾年有明顯的下降。這主要是由于該航空公司在裁員整頓的基礎(chǔ)上,有效地提高了服務(wù)質(zhì)量。當(dāng)然,“9.11”事件后航班乘客數(shù)量的銳減也是一個(gè)重要原因。

  II 統(tǒng)計(jì)數(shù)字表明:近年來我國心血管病的死亡率,即由心血管病導(dǎo)致的死亡率在整個(gè)死亡人數(shù)中的比例,較前有明顯增加,這主要是由于隨著經(jīng)濟(jì)的發(fā)展,我國民眾的飲食結(jié)構(gòu)和生活方式發(fā)生了容易誘發(fā)心血管病的不良變化。當(dāng)然,由于心血管病主要是老年病,因此,我國人口的老齡化,即人口中老年人比例的增加也是一個(gè)重要原因。

  III 某市去年的高考錄取率比前年增加15%,這主要是由于各中學(xué)狠抓了教育質(zhì)量。當(dāng)然,另一個(gè)重要原因是,該市今年參加高考的人數(shù)比去年增加了20%。

  A.僅僅I和II。   B.僅僅II和III。   C.僅僅I和III。   D.僅僅I。

  解析:題目中的邏輯錯(cuò)誤是強(qiáng)加因果,出版物數(shù)量的增加與錯(cuò)字率的增加沒有因果關(guān)系。根據(jù)這一思路對選項(xiàng)進(jìn)行判斷,選項(xiàng)I航班數(shù)銳減與投訴率的下降沒有因果關(guān)系,與題目中犯了同樣的錯(cuò)誤;選項(xiàng)II,心血管病是老年病,我國人口老齡化,因此我國心血管死亡率增加,這是一個(gè)有效地推理過程;選項(xiàng)III,參加高考的人數(shù)增加,并不能推出錄取比例增加的這一結(jié)論。也是屬于沒有因果關(guān)系。另外從絕對和相對概念這個(gè)角度分析,同樣可以得到相似的結(jié)論。選C

  37.從前,一個(gè)孤島上有一個(gè)奇怪的風(fēng)俗:凡是漂流到這個(gè)島上的外鄉(xiāng)人都要作為祭品被殺掉,但允許被殺的人在臨死前說一句話,然后由這個(gè)島上的長老判定這句話是真的還是假的。如果說的是真話,則將這個(gè)外鄉(xiāng)人在真理之神面前殺掉;如果說的是假話,則將他在錯(cuò)誤之神面前殺掉。有一天,一位哲學(xué)家漂流到了這個(gè)島上,他說了一句話,使得島上的人沒有辦法殺掉他。

  該哲學(xué)家必定說了下面哪一句話?

  A.你們這樣做不合乎理性。    B.我將死在真理之神面前。

  C.無論如何我都會死。      D.我將死在錯(cuò)誤之神面前。

  解析:A選項(xiàng),可以判斷為真,會在真理之神面前殺掉;B選項(xiàng),可以判定為真,因?yàn)樗麜涝谡胬碇衩媲;選項(xiàng)C,同樣可以判定為真,選項(xiàng)D我將死在錯(cuò)誤之神面前,如果為真,那么說明他說的是假話,表明他將死在真理之神面前為真,這本身與這句話矛盾了,所以這句話自背反命題,選D。

  38.有的地質(zhì)學(xué)家認(rèn)為,如果地球的未勘探地區(qū)中單位面積的平均石油儲藏量能和已勘探地區(qū)一樣的話,那么,目前關(guān)于地下未開采的能源含量的正確估計(jì)因此要乘上一萬倍,由此可得出結(jié)論,全球的石油需求,至少可以在未來五個(gè)世紀(jì)中得到滿足,即便此種需求每年呈加速上升的趨勢。為使上述論證成立,以下哪項(xiàng)是必須假設(shè)的?

  A. 地球上未勘探地區(qū)的總面積是已勘探地區(qū)的一萬倍。

  B. 在未來至少五個(gè)世紀(jì)中,世界人口的增長率不會超過對石油需求的增長率。

  C. 新技術(shù)將使未來對石油的勘探和開采比現(xiàn)在更為可行。

  D. 地球上未勘探地區(qū)中儲藏的石油可以被勘測和開采出來。

  解析:答案為D,題目中通過假設(shè)得出了未來五個(gè)世紀(jì)的石油需求相會得到滿足的結(jié)論,假設(shè)條件為地球上未勘探地區(qū)的單位面積石油儲量與已勘探地區(qū)一樣多。但僅有這個(gè)前提還是不夠的,還需要這些為勘探的時(shí)候都是可以被開采出來的這一假設(shè)。

  39.某礦山發(fā)生了一起嚴(yán)重的安全事故。關(guān)于事故原因,甲乙丙丁四位負(fù)責(zé)人有如下斷定:

  甲:如果造成事故的直接原因是設(shè)備故障,那么肯定有人違反操作規(guī)程。

  乙:確實(shí)有人違反操作規(guī)程,但造成事故的直接原因不是設(shè)備故障

  丙:造成事故的直接原因確實(shí)是設(shè)備故障,但并沒有人違反操作規(guī)程。

  。涸斐墒鹿实闹苯釉蚴窃O(shè)備故障。

  如果上述斷定只有一個(gè)人的斷定為真,那么以下斷定都不可能為真,除了:

  A.甲的斷定為真,有人違反了操作規(guī)程。

  B.甲的斷定為真,但沒有人違反操作規(guī)程。

  C.乙的斷定為真。

  D.丙的斷定為真。

  解析:這道題屬于邏輯推理題,由于只有一個(gè)人判斷為真,因此丙、丁的判斷首先排除,因?yàn)檫@兩個(gè)判定并不沖突。甲、乙的推斷有矛盾,因此兩項(xiàng)必有一個(gè)是正確的。甲的推斷結(jié)構(gòu)是如果B推出A,那么 C必然推出B;乙的推斷結(jié)構(gòu)是C推出B,但B不能退出A,是甲判定的一個(gè)逆向命題。選項(xiàng)B是正確答案,因?yàn)檫x項(xiàng)B肯定了甲的推斷,同時(shí)否定了乙的發(fā)生,因此是真命題。其他選項(xiàng)均不正確。

  40.心臟的搏動引起血液循環(huán)。對同一個(gè)人,心率越快,單位時(shí)間進(jìn)入循環(huán)的血液量越多。血液中的紅血球運(yùn)輸氧氣。一般地說,一個(gè)人單位時(shí)間通過血液循環(huán)獲得的氧氣越多,他的體能及其發(fā)揮就越佳。因此,為了提高運(yùn)動員在體育比賽中的競技水平,應(yīng)該加強(qiáng)他們在高海拔地區(qū)的訓(xùn)練,因?yàn)樵诟吆0蔚貐^(qū),人體內(nèi)每單位體積血液中含有的紅血球數(shù)量,要高于在低海拔地區(qū)。

  以下哪項(xiàng)是題干的論證必須假設(shè)的?

  A.運(yùn)動員在高海拔地區(qū)的心率不低于在低海拔地區(qū)。

  B.不同運(yùn)動員的心率基本相同。

  C.運(yùn)動員的心率比普通人慢。

  D.在高海拔地區(qū)訓(xùn)練能使運(yùn)動員的心率加快。

  解析:此題論證說明運(yùn)動員在高海拔地區(qū)訓(xùn)練體內(nèi)的紅血球數(shù)量高于低海拔地區(qū),從而達(dá)到最佳體能的目的。B和C與題意不符,選A。

 41~45題基于以下題干:

  七個(gè)學(xué)生R、S、T、V、W、X、Y,被分成兩個(gè)學(xué)習(xí)小組。第一組有三名成員,第二組有四名成員。學(xué)生們的分組必須符合以下要求:

  R和T不能在同一個(gè)小組;

  如果S在第一組,那么V必須在第一組;

  如果W在第一組,那么T必須在第二組;

  X必須在第二組。

  41.如果W在第一組,那么以下哪項(xiàng)也一定在第一組?

  A.R    B.S    C.T    D.V

  解析:根據(jù)條件第一組有三名成員。因?yàn)镽和T不能在一組,又因?yàn)闂l件:如果W在第一組,那么T必須在第二組,則R可以在第一組

  42.如果T和Y都在第一組,那么以下哪項(xiàng)一定是真的?

  A.S和V在同一組。   B.S和W在同一組。   C.V和R在同一組。   D.W和T在同一組。

  解析:如果T和Y在第一組,R和T又不能在一組,可知R只能在第二組,而X必須在第二組。如果S在第一組,根據(jù)條件,V也需在第一組,而第一組只有三個(gè)成員,那么S和V就不能在同一組。而如果W在第一組,那么T必須在第二組。但是T已經(jīng)在第一組,則W必須在第二組,加之第二組有四人,那么S也需在第二組,那么S和W為一組。

  43.如果W和T在同一組,那么以下哪項(xiàng)可能在同一組,除了:

  A.R和S    B.S和Y    C.T和Y    D.V和Y

  答案:B

  解析:

  條件分析:

  1、R1且T2;或者 R2且T1

  2、S1→V1;即S2的時(shí)候,V可以自由分配。

  3、W1→T2;即W2的時(shí)候,T可以自由分配。

  4、X2

  43題的臨時(shí)題干要求WT在同一組,根據(jù)條件3,他們不可能在第一組,不然就違背了題設(shè)。所以,這個(gè)時(shí)候,WT都在都在第二組。第二組已經(jīng)有了WTX三個(gè)成員,只有一個(gè)空位了。根據(jù)條件1,可以知道,R在第一組。因此,除了跟WTX搭伴的元素外,任何二人組合都必須在第一組了。

  選項(xiàng)A,第一組RSV;第二組WTXY,有可能。

  選項(xiàng)B,第一組RSY;則第一組還必須有V,顯然超編了。因?yàn)榈谝唤M只能有三個(gè)人。因此,此項(xiàng)為答案。

  選項(xiàng)C,依然是第一組RSV;第二組WTXY這樣的組合。

  選項(xiàng)D,第一組RVY;第二組WTXS,不矛盾。

  44.如果V和Y在同一組,那么以下哪項(xiàng)一定是真的?

  A.R在第一組。    B.S在第一組。    C.W在第二組。    D.Y在第二組。

  解析:如果V和Y在第一組,那么根據(jù)條件如果S在第一組,那么V必須在第一組;說明S可以在第一組,又因?yàn)槿绻鸚在第一組,那么T必須在第二組,但是R和T又不能在一組,則R必須在第一組,W和T可以在第二組。則答案C正確。

  45.如果S在第一組,那么以下哪項(xiàng)一定是真的?

  A.T在第一組。    B.T在第二組。    C.Y在第一組。    D.Y在第二組。

  解析:如果S在第一組,根據(jù)條件V必須在第一組,如果W在第一組,那么T必須在第二組;R和T不能在同一個(gè)小組;那么W或R可以在第一組,X必須在第二組。因此在第一組的可以是S或V或R或W,因此答案D正確。

  46~50題基于以下題干:

  六位教授F、G、H、J、K、L,將評審馬、任、孫、吳博士的論文4篇。評審需遵守以下原則:

  (1)每位教授只評審一篇博士論文;

  (2)每篇博士論文至少有一位教授評審;

 。3)H與F評審?fù)黄┦空撐模?/p>

  (4)L只與其他教授中的一位同評一篇博士論文;

  (5)G評審馬博士的論文;

 。6)J評審馬博士或吳博士的論文;

 。7)H不評審吳博士的論文。

  46.如果K不評審孫博士的論文,那么以下哪項(xiàng)一定是真的?

  A.L評審馬博士的論文           B.L評審孫博士的論文

  C.F和H評審任博士的論文        D.F和H評審孫博士的論文

  解析:H不評審吳博士論文,H與F又評審?fù)黄撐;那么H和F有可能評審馬或任或?qū)O博士的論文,但是G評審馬博士的論文,J評審馬博士或者吳博士的論文,那么H與F就有可能評審任或?qū)O博士的論文,K不評審孫博士的論文,K就有可能評審任博士的論文,那么H和F就會評審孫博士的論文。

  47.以下哪項(xiàng)完整地列出了L可能評審的博士論文?

  A.馬、任      B.馬、孫、吳     C.馬、任、吳       D.馬、任、孫、吳

  解析:根據(jù)條件四,L只與其他教授中的一位同評一篇博士論文,而每篇博士論文至少有一位教授評審,也就是說一篇論文有可能是兩個(gè)教授或者三個(gè)教授一起評論。因此,L可以評論馬、任、孫、吳的博士論文。

  48.以下哪項(xiàng)可能是真的?

  A.F和G評審馬博士的論文。

  B. F和L評審吳博士的論文。

  C. K評審吳博士的論文并且L評審馬博士的論文。

  D. L評審任博士的論文并且F評審孫博士的論文。

  解析:G評審馬博士的論文,J評審馬博士或吳博士的論文,H不評審吳博士的論文,H與F又評審?fù)黄撐模荒敲碒和F有可能評審馬或任或?qū)O博士的論文,那么H與F就有可能評審任或?qū)O博士的論文。加之題一,K不評審孫博士的論文,K就有可能評審任博士的論文,條件三H與F評審?fù)黄┦空撐,那么H和F就會評審孫博士的論文。條件四:L只與其他教授中的一位同評一篇博士論文,L可以與K評審任博士論文,故D正確

  49.以下哪項(xiàng)不可能是真的?

  A. L和G評審馬博士的論文        B. L和K評審馬博士的論文

  C. L和K評審任博士的論文        D. L和K評審孫博士的論文

  解析:根據(jù)條件(4)L只與其他教授中的一位同評一篇博士論文,而條件(5)G評審馬博士的論文,那么可知L和G可評審馬博士論文,故A正確。根據(jù)條件(6)J評審馬博士或吳博士的論文,可以得知L可以和J可以評審吳博士或者馬博士的論文,H不評審吳博士的論文,那么H可以評論任博士和孫博士的論文。K可以評論吳博士、任博士、孫博士的論文。加之條件(4),得知答案C和D正確,故B錯(cuò)誤

  50.如下哪位教授不可能審閱任博士的論文?

  A.F   B.K    C.J   D. L

  解析:條件(6)J評審馬博士或吳博士的論文;而條件(1)每位教授只評審一篇博士論文,根據(jù)兩個(gè)條件可以推斷J不可能審閱任或?qū)O博士的論文,故選C

  模擬題1參考答案:

01. C 02. A 03. C 04. D 05. C
06. C 07. A 08. B 09. A 10. A
11. A 12. B 13. B 14. A 15. C
16. D 17. D 18. C 19. A 20. B
21. C 22. C 23. B 24. B 25. C
26. A 27. B 28. D 29. D 30. C
31. B 32. A 33. A 34. B 35. B
36. C 37. D 38. D 39. B 40. A
41. A 42. B 43. B 44. C 45. D
46. D 47. D 48. D 49. B 50. C
 

 

 

 

(50題,每題2分,滿分100分)

  1.任何人都沒有吃過雅各島上的任何水果,所以無法知道雅各島上任何水果的口味。

  為了合乎邏輯的推出上述結(jié)論,需要假設(shè)下面哪項(xiàng)為前提?

  A.如果一種水果有人品嘗過,就可以知道其口味。(充分條件,不是前提)

  B.只憑某些人的品嘗無法真正知道某種水果的口味。

  C.要知道某種水果的口味,需要有人去品嘗。

  D.人們是通過嗅覺來確定水果口味的。

  解析:這道題需要補(bǔ)全假設(shè),結(jié)論是:任何人都無法知道任何水果的味道,前提是:任何人都沒有吃過雅各島上的任何水果。補(bǔ)充前提是:要知道水果的口味,需要有人去品嘗,應(yīng)該選擇C。選項(xiàng)A,是充分條件,但不是構(gòu)成結(jié)論的原因,屬于強(qiáng)拉因果;B、D都為不合理的假設(shè)。

  2.最近的一項(xiàng)研究指出:“經(jīng)常吃沙棘果對兒童的智力發(fā)育有益。”研究人員對560名兒童進(jìn)行調(diào)查,發(fā)現(xiàn)那些經(jīng)常吃沙棘果的兒童,其智力水平較很少吃沙棘果的兒童要高。因此,研究人員發(fā)現(xiàn)了沙棘果與兒童智力發(fā)育之間的聯(lián)系。

  以下哪項(xiàng)如果為真,最不可能削弱上述論證?

  A.對成年人的研究發(fā)現(xiàn),每天吃沙棘果的人智力水平并不比很少吃沙棘果人的高。

  B.調(diào)查顯示:沙棘果價(jià)格非常高,只有富裕家庭的兒童才經(jīng)常吃,同時(shí)這些家庭有條件實(shí)現(xiàn)兒童的早期智力開發(fā)。

  C.這項(xiàng)兒童發(fā)育研究的課題負(fù)責(zé)人是沙棘果生產(chǎn)商,其目的就是要擴(kuò)展沙棘果的銷售渠道。

  D.沙棘果是兒童喜歡的食品,家長經(jīng)常把沙棘果作為禮物獎(jiǎng)給智力表現(xiàn)優(yōu)異的孩子。

  解析:吃沙棘果與兒童智力發(fā)展是否存在必然聯(lián)系。通過樣本調(diào)查判斷出吃沙棘果與兒童智力發(fā)展存在正相關(guān)的關(guān)系。B選項(xiàng)說明家庭條件是否富裕與兒童智力發(fā)展存在關(guān)聯(lián)聯(lián)系;C選項(xiàng)說明題目中的結(jié)論是由于生產(chǎn)商的廣告產(chǎn)生的以偏概全的結(jié)論;D選項(xiàng)屬于因果倒置,成績優(yōu)異是因,吃沙棘果是結(jié)果。

  3.凡金屬都是導(dǎo)電的。銅是導(dǎo)電的,所以銅是金屬。

  下面哪項(xiàng)與上述推理結(jié)構(gòu)最相似?

  A.所有的鳥都是卵生動物,蝙蝠不是卵生動物,所以,蝙蝠不是鳥。

  B.所有的鳥都是卵生動物,天鵝是鳥,所以天鵝是卵生動物。

  C.所有從事工商管理的都要學(xué)習(xí)企業(yè)管理,老陳是學(xué)習(xí)企業(yè)管理的,所以,老陳是從事工商管理工作的。

  D.華山險(xiǎn)于黃山,黃山險(xiǎn)于泰山,所以華山險(xiǎn)于泰山。

  解析:題目結(jié)構(gòu)是所有的A具有某中屬性,B屬于A,因此B具有A的屬性,選項(xiàng)C與上述結(jié)論類似。

  4.為了有助于人們選擇最滿意的城市居住,有關(guān)部門實(shí)施了一項(xiàng)評選“最舒適城市”的活動。方法是,選擇十個(gè)方面,包括社會治安、商業(yè)設(shè)施、清潔程度、綠化程度、教育設(shè)施、旅游文化景點(diǎn)等等,每個(gè)方面按實(shí)際質(zhì)量的高低,評以1分至10分之間的某一分值,然后求得十個(gè)分值的平均數(shù)即是這個(gè)城市的舒適性指數(shù)。

  以下哪項(xiàng)是實(shí)施上述活動需要預(yù)設(shè)的前提?

 、、城市的各種舒適性質(zhì)量程度都可以用準(zhǔn)確的數(shù)字表達(dá)。

 、、城市的各種舒適性對于居民來說都是同等重要的。

 、蟆⒕用裼凶杂蛇x擇居住城市的權(quán)利并且大都樂于這樣做。

  A.僅Ⅰ。      B.僅Ⅲ。      C.僅Ⅰ和Ⅱ。      D.Ⅰ、Ⅱ和Ⅲ。

  解析:這道題是補(bǔ)充假設(shè)的問題,由于每個(gè)人對“舒適程度”的感覺是不一樣的,因此題干結(jié)論若要正確,條件Ⅱ是必須的前提;若題目中結(jié)論正確,需要假設(shè)題目中的十個(gè)指標(biāo)存在量化標(biāo)準(zhǔn),因此條件Ⅰ為必選項(xiàng);條件Ⅲ存在爭議,不過可以從另外一個(gè)角度理解,便于理解題目的結(jié)論,如果居民沒有選擇居住地的權(quán)利,那么題目中的滿意度調(diào)查就沒有意義,所以也得不到上述結(jié)論。

  5.衛(wèi)生部的官員們對牟定縣狂犬病疫情有以下斷定:

  (1)該縣所有的狗都得了狂犬病。

  (2)該縣有些斑點(diǎn)狗得了狂犬病。(真)

  (3)該縣有些狗得了狂犬病。

  (4)該縣有些狗沒得狂犬病。(真)

  其實(shí)上述斷定中只有兩個(gè)與事實(shí)相符。根據(jù)如上的情況,以下哪項(xiàng)結(jié)論是可能成立的?

  I.該縣的狗都是斑點(diǎn)狗。

  II. 該縣沒有斑點(diǎn)狗可能得狂犬病。

  III. 該縣的斑點(diǎn)狗并非可能沒有得狂犬病。

  IV. 該縣的狗不可能都沒得狂犬病。

  A.僅I      B.僅III      C.僅II和IV      D.僅IV。

  解析:這道題屬于邏輯推理中真假話題的問題。解析這種問題按照1、先找矛盾;2、繞開矛盾;3、推出答案的步驟進(jìn)行。首先先找矛盾,首先1和2是一對矛盾,3和4是一對矛盾,只有一個(gè)正確,2、繞開矛盾,如果1正確,那么3和4都假,因此只能2為真,如果2為真,只能判斷4為真,3為假,3、推出答案,條件2有些斑點(diǎn)夠得了狂犬病,可推出該縣沒有斑點(diǎn)狗可能得狂犬病,大家注意必然=沒有……必然;同時(shí)根據(jù)條件4可以得出該縣的狗不可能都沒得狂犬病。

  6.如果一個(gè)兒童的體重與身高的比值超過本地區(qū)80%兒童的水平,就稱其為肥胖兒。根據(jù)歷年的調(diào)查結(jié)果,15年來,臨江市的肥胖兒的數(shù)量一直在穩(wěn)定增長。

  如果以上斷定為真,則以下哪項(xiàng)也必為真?

  A.臨江市每一個(gè)肥胖兒的體重都超過全市兒童的平均體重。

  B.15年來,臨江市的兒童的體育鍛煉越來越不足。

  C.臨江市的非肥胖兒的數(shù)量15年來不斷增長。

  D.15年來,臨江市體重不足標(biāo)準(zhǔn)體重的兒童數(shù)量不斷下降。

  解析:這道題屬于迷惑推理題,大家務(wù)必要小心。首先大家可以看出臨江市兒童數(shù)量是在增長的;兒童由肥胖和非肥胖組成;肥胖兒童的數(shù)量穩(wěn)定增長,說明非肥胖兒童的數(shù)量也在穩(wěn)定增長;沒有證據(jù)表明非肥胖兒童的數(shù)量在不斷下降。

  7.孩子出生后的第一年在托兒所里度過,會引發(fā)孩子的緊張不安。在我們的研究中,有464名12~13歲的兒童接受了特異情景測試法的測驗(yàn),該項(xiàng)測驗(yàn)意在測試兒童1歲時(shí)的狀況與對母親的依附心理之間的關(guān)系。其結(jié)果:有41.5%曾在托兒所看護(hù)的兒童和25.7%曾在家看護(hù)的兒童被認(rèn)為緊張不安,過于依附母親。

  以下哪項(xiàng)如果為真,最沒有可能對上述研究的推斷提出質(zhì)疑?

  A.出生后第一年在家看護(hù)的孩子多數(shù)是由祖父母或外祖父母看護(hù)的,并形成濃厚的親情。

  B.這項(xiàng)研究的主持者被證實(shí)曾經(jīng)在自己的幼兒時(shí)期受到過長時(shí)間來自托兒所阿姨的冷落。

  C.針對孩子母親的另一部分研究發(fā)現(xiàn):由于孩子在家里表現(xiàn)出過度的依附心理,父母因此希望將其送入托兒所予以矯正。

  D.研究中所測試的孩子并不是從托兒所看護(hù)和在家看護(hù)兩種情況下隨機(jī)抽取的。因此,這兩組樣本兒童的家庭很可能有系統(tǒng)性的差異存在。

  解析:這種題最好的解決辦法是否定推斷,把你認(rèn)為可能的答案否定,帶入題目中判斷。如果能夠提出質(zhì)疑,說明你的判斷是正確的。A選項(xiàng)。

  8.在玉米地中套種的小麥有可能得小麥黃葉病,這種病是由于光照不足造成的,一旦光照充足,比如減少玉米植株數(shù)量,或者玉米收割后,癥狀很快就會消失,且不會影響產(chǎn)量。而小麥銹病是由花斑銹菌引起,如果在病癥初起時(shí)不及時(shí)恰當(dāng)應(yīng)對,就會使小麥植株很快枯死。由于小麥銹病和小麥黃葉病初期癥狀很難區(qū)分,所以,在玉米地中套種小麥,必須高度注意小麥銹病的定期篩查和預(yù)防。

  以下哪項(xiàng)最可能是上述論證所假設(shè)的?

  麥黃葉病不會誘發(fā)小麥銹病。

  B.花斑銹菌不會因?yàn)槌渥愎庹斩π←湹那趾αΑ?/p>

  C.小麥銹病如果及時(shí)恰當(dāng)處理,就不會使小麥減產(chǎn)。

  D.科學(xué)家一直沒有找到有效遏制小麥銹病的藥物。

  解析:假設(shè)推斷。陽光充足,小麥黃葉病會消失,產(chǎn)量不影響;花斑銹菌,使小麥植株枯死。因此需要假設(shè)陽光充足不會抑制花斑銹菌的發(fā)病,才能得出小麥黃葉不發(fā)病時(shí),花斑銹菌仍然會發(fā)病,導(dǎo)致兩種病情難以區(qū)分。換一種角度,如果小麥黃葉病不會和花斑銹菌同時(shí)發(fā)病,那么就不存在難以區(qū)分病情的問題了。

  9.公司準(zhǔn)備從五個(gè)業(yè)務(wù)骨干中選幾個(gè)人去中央戲劇學(xué)院進(jìn)修。鑒于業(yè)務(wù)員們的工作關(guān)系,總經(jīng)理建議:

 。1)如果選鞏俐,那么必須選章子怡并且不能選周迅。

 。2)如果選章子怡或者選周迅,則不能選吳孟達(dá)。

 。3)不能既不選周星馳也不選吳孟達(dá)。

  考慮總經(jīng)理的建議,董事會認(rèn)為,鞏俐必須去中央戲劇學(xué)院進(jìn)修,這樣誰將跟她一起去進(jìn)修?

  A.章子怡和周星馳    B.章子怡和吳孟達(dá)   C.周星馳和周迅    D.吳孟達(dá)和周迅

  解析:邏輯推斷。如果鞏麗必須去,根據(jù)條件1章子儀必須去,而周迅一定不去,從而排除C、D選項(xiàng),根據(jù)條件2,如果選章子儀,那么吳孟達(dá)不能去。排除B 。

  繼續(xù)進(jìn)行推理,根據(jù)條件3必須在周星馳和吳孟達(dá)中選一個(gè),因?yàn)楦鶕?jù)條件2已經(jīng)排除吳孟達(dá),所以周星馳必須去。選擇A。

  10.20世紀(jì)60年代早期之前,挪威的斯塔溫格是一個(gè)安靜而和平的小鎮(zhèn)。進(jìn)入60年代以來,這里成為了挪威近海石油勘探中心。從此,暴力犯罪和毀壞公物的現(xiàn)象在斯塔溫格小鎮(zhèn)急劇增加。顯然這些社會問題是是石油繁榮的副產(chǎn)品。

  下列哪一個(gè)選項(xiàng)最強(qiáng)烈的支持上面的論述。

  A.暴力犯罪和毀壞公物在沒有石油繁榮的挪威城鎮(zhèn)保持低水平。

  B.斯塔溫格居民很少對這里是近海石油勘探中心感到遺憾。

  C.挪威社會學(xué)家對斯塔溫格的暴力犯罪和毀壞公物現(xiàn)象持續(xù)增加表示關(guān)切。

  D.非暴力犯罪、毒品、離婚,在這個(gè)小鎮(zhèn)也以同樣的速度增加著。

  解析:題目把社會問題的原因歸結(jié)為石油繁榮,最強(qiáng)烈支持的論據(jù)為在沒有石油繁榮的時(shí)期,社會問題處于低水平。因此選A。

11.認(rèn)為只傷害自己而沒有傷害到別人的行為并沒有什么錯(cuò)誤的想法,通常伴隨著對人與人之間相互依賴的人際關(guān)系的忽視。毀壞一個(gè)人自己的健康或者生命意味著不能你將不能為家庭或者社會提供幫助,相反意味著你將額外享有那些本來就有限的諸如食物、健康服務(wù)和教育等社會資源,而不是相反將它們回報(bào)給社會。

  以下哪個(gè)選項(xiàng)如果正確,最強(qiáng)烈的地支持題干所表達(dá)的觀點(diǎn)。

  A.本來可以避免的疾病和意外事故所造成的花銷增加了每個(gè)人的健康保險(xiǎn)金。

  B.傷害一個(gè)人能夠?qū)е麻g接的利益,諸如可以使別人獲得與健康領(lǐng)域相關(guān)聯(lián)的工作。

  C.一個(gè)人對社會所作的貢獻(xiàn)可以通過他的健康程度來衡量。

  D.由喝酒、吸煙和吸食毒品所導(dǎo)致的主要傷害是由使用那些東西的人來承受的。

  解析:題目中的結(jié)論是傷害自己的行為與傷害別人的行為同樣都是錯(cuò)誤的。原因是傷害自己將不能為家庭和社會提供幫助,同時(shí)會向社會索取的更多。這道題的關(guān)鍵是大家要讀懂題干,這樣從選項(xiàng)中我們不難選出A,本來可以避免的個(gè)人傷害回增加健康保險(xiǎn)金,從而增加社會的負(fù)擔(dān)。

  12.大多數(shù)道路的修理比預(yù)算的要花費(fèi)更多的時(shí)間和金錢,但是去年夜間修理京津塘高速公路和類似的道路并未比預(yù)算花費(fèi)更多的時(shí)間或金錢。因此,在夏季,夜間修理主要道路可能省時(shí)省錢。

  下列哪一個(gè),假如正確,最支持上面得到的結(jié)論?

  A.修理京津塘高速公路的預(yù)算足夠用,所以不可能超過預(yù)算。

  B.夏季,夜間路上通行車輛較少,而且溫度較舒適,允許修路工人工作得更快。

  C.愿意在晚上工作的修路工人較容易找到工作機(jī)會,因?yàn)榇蠖鄶?shù)人寧愿白天工作。

  D.用于道路修理的瀝青在較高溫度下膨脹,在溫度降低的時(shí)候收縮。

  解析:補(bǔ)充論據(jù)。從題干可以看出從特殊樣本推出結(jié)論明顯論據(jù)不足。夏天夜間施工本來和省時(shí)省錢本來沒有關(guān)系,屬于不相關(guān)關(guān)系,因此需要補(bǔ)充適當(dāng)?shù)恼摀?jù),在這兩個(gè)問題中間搭起橋梁,建立關(guān)系,所以選擇B。

  13.偏頭痛一直被認(rèn)為是由食物過敏引起的。但是,如果讓患者停止食用那些已經(jīng)證明會不斷引起過敏性偏頭痛的食物,他們的偏頭痛并沒有停止,因此,顯然存在別的某種原因引起偏頭痛。下列哪項(xiàng)如果為真,最能削弱上面的結(jié)論?

  A.許多患者說誘發(fā)偏頭痛病的那些食物往往是他們最喜歡吃的食物。

  B.許多普通食物只在食用幾天后才誘發(fā)偏頭痛,因此,不容易觀察患者的過敏反應(yīng)和他們食用的食物之間的關(guān)系。

  C.很少有食物過敏會引起像偏頭痛那樣嚴(yán)重的癥狀。

  D.許多不患偏頭痛的人同樣有食物過敏反應(yīng)。

  解析:題目結(jié)論是過敏引發(fā)偏頭痛,但是引發(fā)過敏的問題不僅僅是食物引起的。結(jié)論指明食物過敏與偏頭痛之間不存在這因果關(guān)系,原因是停用了這種食物之后,偏頭痛的病情仍然存在,若要削弱這個(gè)結(jié)論,需要針對上述論據(jù)提出反對。本著這個(gè)思路我們看選項(xiàng),結(jié)論B。表明了停用食物之后仍然存在偏頭痛的原因是因?yàn)樵S多普通食物在食用幾天后才發(fā)作,因此雖然停用了食物,仍會發(fā)作一段時(shí)間。

  14.世界衛(wèi)生組織在全球范圍內(nèi)進(jìn)行了一項(xiàng)有關(guān)獻(xiàn)血對健康的影響的跟蹤調(diào)查。調(diào)查對象分為三組。第一組對象中均有二次以上的獻(xiàn)血記錄,其中最多的達(dá)數(shù)十次;第二組中的對象均僅有一次獻(xiàn)血記錄;第三組對象均從未獻(xiàn)過血。調(diào)查結(jié)果顯示,被調(diào)查對象中癌癥和心臟病的發(fā)病率,第一組分別為0.3%和0.5%,第二組分別為0.7%和0.9%,第三組分別為1.2%和2.7%。一些專家依此得出結(jié)論,獻(xiàn)血有利于減少患癌癥和心臟病的風(fēng)險(xiǎn)。這兩種病已經(jīng)不僅在發(fā)達(dá)國家而且在發(fā)展中國家成為威脅中老年人生命的主要?dú)⑹。因此,獻(xiàn)血利己利人,一舉兩得。以下哪項(xiàng)如果為真,將削弱以上結(jié)論?

  I 60歲以上的調(diào)查對象,在第一組中占60%,在第二中占70%,在第三組中占80%。

  II 獻(xiàn)血者在獻(xiàn)血前要經(jīng)過嚴(yán)格的體檢,一般具有較好的體質(zhì)。

  III 調(diào)查對象的人數(shù),第一組為1700人,第二組為3000人,第三組為7000人。

  A.只有I和II。    B.只有II和III。    C.只有I和III    D.只有I

  解析:題目中對獻(xiàn)血與癌癥、心臟病的發(fā)病幾率建立了正相關(guān)關(guān)系。但是樣本仍然存在問題,如果鮮血的人本身身體素質(zhì)就比沒有獻(xiàn)血的身體素質(zhì)好,就不能退出上述結(jié)論。這是最簡單容易想出的悖論。因此淘汰C、D。下面在條件1和3中選一個(gè),條件3說明樣本容量不同,但是題目中使用的是百分?jǐn)?shù)這個(gè)相對指標(biāo),因此不能形成反對結(jié)論的悖論。選A。

  15.許多孕婦都出現(xiàn)了維生素缺乏的癥狀,但這通常不是由于孕婦的飲食中缺乏維生素,而是由于腹內(nèi)嬰兒的生長使她們比其他人對維生素有更高的需求。為了評價(jià)上述結(jié)論的確切程度,以下哪項(xiàng)操作最為重要?

  A.對某個(gè)不缺乏維生素的孕婦的日常飲食進(jìn)行檢測,確定其中維生素的含量。

  B.對孕婦的科學(xué)食譜進(jìn)行研究,以確定有利于孕婦攝入足量維生素的最佳選擇。

  C.對日常飲食中維生素足量的一個(gè)孕婦和一個(gè)非孕婦進(jìn)行檢測,并分別確定她們是否缺乏維生素。

  D.對日常飲食中維生素不足量的一個(gè)孕婦和另一個(gè)非孕婦進(jìn)行檢測,并分別確定她們是否缺乏維生素。

  解析:支持論據(jù)。題目需要根據(jù)結(jié)論找出支持論據(jù),顯然對同量的維生素給孕婦和非孕婦服用,觀察他們是否缺乏維生素,對得出結(jié)論最重要。選C

  16.經(jīng)A省的防疫部門檢測,在該省境內(nèi)接受檢疫的長尾猴中,有1%感染上了狂犬病。但是只有與人及其寵物有接觸的長尾猴才接受檢疫。防疫部門的專家因此推測,該省長尾猴中感染有狂犬病的比例,將大大小于1%。

  以下哪項(xiàng)如果為真,將最有力地支持專家的推測?

  A.在A省境內(nèi),與人及其寵物有接觸的長尾猴,只占長尾猴總數(shù)的不到10%。

  B.在A省,感染有狂犬病的寵物,約占寵物總數(shù)的0.1%。

  C.在與A省毗鄰的B省境內(nèi),至今沒有關(guān)于長尾猴感染狂犬病的疫情報(bào)告。

  D.與健康的長尾猴相比,感染有狂犬病的長尾猴更愿意與人及其寵物接觸。

  解析:選擇D,其他選項(xiàng)均不能支持題干給出的結(jié)論。在與人和動物接觸的長尾猴進(jìn)行檢驗(yàn),發(fā)現(xiàn)1%感染了狂犬病,推斷出該省長尾猴感染狂犬病的比例小于1%,那么只有感染狂犬病的長尾猴更喜歡與人和動物接觸能夠有效地支持這個(gè)結(jié)論。

  17.作為市電視臺的攝像師,最近國內(nèi)電池市場的突然變化讓我非常頭疼。進(jìn)口電池缺貨,我只能用國產(chǎn)電池來代替作為攝像的主要電源。盡管每單位的國產(chǎn)電池要比進(jìn)口電池便宜,但我估計(jì)如果持續(xù)用國產(chǎn)電池替代進(jìn)口電池來提供同樣的電源供應(yīng)的話,我在能源上的支付將會提高。

  說這番話的人在上面這段話中隱含了以下哪項(xiàng)假設(shè)?

  A. 持續(xù)使用國產(chǎn)電池,攝像的質(zhì)量將無法得到保障。

  B. 每單位的進(jìn)口電池要比國產(chǎn)電池價(jià)格貴。

  C. 生產(chǎn)國產(chǎn)電池要比生產(chǎn)進(jìn)口電池成本低。

  D. 以每單位電池提供的電能來計(jì)算,國產(chǎn)電池要比進(jìn)口電池提供得少。

  解析:這是日常生活中經(jīng)常會遇到的問題,答案一目了然選擇D。得出結(jié)論的原因是盡管國產(chǎn)電池比進(jìn)口電池單價(jià)便宜,但是就單位電能來講,國產(chǎn)電池的價(jià)格卻比進(jìn)口電池高。

  18.最近由于氣候的異常變化,使得山東省的大蒜產(chǎn)量受到較大影響而減產(chǎn)。山東大蒜的價(jià)格比平時(shí)同期上漲了兩倍,這就大大提高了大蒜素生產(chǎn)的成本,估計(jì)未來大蒜素的市場價(jià)格將有大幅度的提高。以下哪項(xiàng)如果是真的,最能削弱上述結(jié)論?

  A.去年大蒜的價(jià)格是歷年最低的。

  B.其他替代原料可以用來生產(chǎn)人工大蒜素。

  C.除了山東省外,其他省份也可以提供大蒜。

  D.最近的天氣異常不如專家們估計(jì)的那么嚴(yán)重。

  解析:題目中的推斷是沒有問題的,成本的提高會促使價(jià)格的提高,但是其他省也提供大蒜,而成本并沒有提高,價(jià)格不會上升,在這樣的市場上,山東省的大蒜盡管成本提高了,但是價(jià)格將會保持市場水平,因此C有效地消弱了結(jié)論。

  19.某學(xué)校開設(shè)選修課,規(guī)定:同學(xué)只有選了文化史課的同學(xué)才能選古典音樂課。如果選古典音樂課,那他就不能選文學(xué)欣賞課。如果選文學(xué)欣賞課,就不能選古典音樂課。

  如果以上陳述為真,以下哪項(xiàng)陳述不可能假?

  A.同學(xué)不選文學(xué)欣賞課 或者不選古典音樂課。

  B.同學(xué)選文學(xué)欣賞課 或者不選古典音樂課。

  C.同學(xué)不選古典音樂課,或者不選文化史課。

  D.同學(xué)選古典音樂課,或者不選文學(xué)欣賞課

  解析:邏輯推斷題。不可能假=一定真,因此題目要求在選項(xiàng)中找出一定真的項(xiàng)。選了文化史,才能選古典音樂,這是個(gè)必要條件;但是選了古典音樂就不能選文學(xué)欣賞,二選其一;選項(xiàng)A的論述與題干一樣。

  20.20年前,幾乎所有的公司首腦在選擇重新設(shè)置公司總部的時(shí)候,主要關(guān)心的是土地價(jià)格。今天一個(gè)高級執(zhí)行官計(jì)劃重設(shè)總部時(shí)主要關(guān)心的東西更廣泛了,經(jīng)常包括當(dāng)?shù)氐膶W(xué)校和住房質(zhì)量。

  假如上述信息可靠,下面哪一項(xiàng)最好地解釋了公司首腦們主要關(guān)心問題的變化?

  A.20年前高質(zhì)量的住房和學(xué)校像今天一樣難以發(fā)現(xiàn)。

  B.近年來優(yōu)秀專業(yè)辦公人員缺乏的問題迫使公司找到盡可能多的方法來留住老員工,并且吸引優(yōu)秀新員工的加盟。

  C.公司執(zhí)行官總是考慮自己的決定將怎樣影響公司的利潤。

  D.在過去20年,一些地區(qū)比其他地區(qū)土地價(jià)格變化少。

  解析:選項(xiàng)A屬于強(qiáng)拉因果,不存在相關(guān)關(guān)系;選項(xiàng)B說明了高級執(zhí)行官在公司選址時(shí)關(guān)注學(xué)校和住房質(zhì)量的原因,分別是吸引優(yōu)秀新員工加盟和留住老員工;選項(xiàng)C與題干沒有關(guān)系,并未涉及到利潤這個(gè)因素;選項(xiàng)D,最多可以說明高級執(zhí)行官不在關(guān)心土地價(jià)格的原因,但是并不能說明關(guān)注學(xué)校和住房質(zhì)量的原因。

21.用蒸餾麥芽渣提取的酒精作為汽油的替代品進(jìn)入市場,使得糧食市場和能源市場發(fā)生了前所未有的直接聯(lián)系。到1995年,谷物作為酒精的價(jià)值己經(jīng)超過了作為糧食的價(jià)值。西方國家已經(jīng)或正在考慮用從谷物提取的酒精來替代一部分進(jìn)口石油。

  如果上述斷定為真,對于那些己經(jīng)用從谷物提取的酒精來替代一部分進(jìn)口石油的西方國家。以下哪項(xiàng),最可能是1995年后進(jìn)口石油價(jià)格下跌的后果?

  A.一些谷物從能源市場轉(zhuǎn)入糧食市場。

  B.一些谷物從糧食市場轉(zhuǎn)入能源市場。

  C.谷物的價(jià)格面臨下跌的壓力。

  D.谷物的價(jià)格出現(xiàn)上浮。

  解析:谷物可以提取出酒精來代替石油出口,因此可以判斷谷物和石油成為了相互競爭的替代品,因此如果石油價(jià)格下跌,那么谷物的價(jià)格也必然會下調(diào),否則進(jìn)口國將會進(jìn)口價(jià)格低廉的石油取代谷物。

  22.有八個(gè)教授參加學(xué)術(shù)會議后,互留通信地址,以保持聯(lián)絡(luò)。一年后,統(tǒng)計(jì)他們之間的通信情況如下:有一人給其他三個(gè)人寫過信,給其他人寫過兩封信的有三人,給其他人寫過一封信的有四個(gè)人。若以上統(tǒng)計(jì)屬實(shí),則最能得出以下哪項(xiàng)結(jié)論?

  A.通過書信來往,他們遞進(jìn)了友誼,也加強(qiáng)了學(xué)術(shù)聯(lián)系。

  B.八個(gè)教授都收到過其他教授的來信。

  C.至少有一個(gè)人收到信件后沒有都回復(fù)。

  D.雖然參加學(xué)術(shù)會議的時(shí)候大家不熟悉,但現(xiàn)在他們已經(jīng)成為至交。

  解析:選項(xiàng)首先排除A、D,沒有任何證據(jù)表明通過書信加強(qiáng)了學(xué)術(shù)聯(lián)系,也沒有證據(jù)表明他們成為了至交。下面看選項(xiàng)B、C,我們可以通過畫圖的方法來解決這個(gè)問題相對比較簡單,從而得出正確答案為C。

  23.金鳳扒雞是一個(gè)享有盛譽(yù)的百年扒雞品牌,在石家莊銷路很好,深受當(dāng)?shù)匕傩障矏郏刻煲簧鲜芯蜁粨屬徱豢。為了打開北京市場,金鳳扒雞改進(jìn)了工藝,用真空包裝,延長了保質(zhì)期,試圖在北京各大超市打開銷路,但是金鳳扒雞在北京卻很難銷售出去。

  下面除了哪一項(xiàng),都有助于解決上述現(xiàn)象?

  A.北京人和石家莊人的飲食習(xí)慣不同。他們更喜歡吃烤鴨而不是扒雞。

  B.恒慧通燒雞也采用真空包裝,保質(zhì)期一樣長,在北京的銷路卻很好。

  C.人們喜歡吃剛出鍋的新鮮扒雞,保質(zhì)期短的反而更受消費(fèi)者青睞。

  D.雖然金鳳扒雞是百年品牌,但是它的銷售地點(diǎn)僅在石家莊市區(qū),北京人對這個(gè)品牌不熟悉。

  解析:這是一道頗具爭議的題目,選項(xiàng)A、B、C、D分別從消費(fèi)者的喜好,競爭者,消費(fèi)偏好和品牌對金鳳扒雞在北京銷路受阻提供了良好的解釋。但是選項(xiàng)B可以從另外一個(gè)角度思考,恒惠通燒雞同樣采用真空包裝,保質(zhì)期一樣長,什么都和金鳳扒雞一樣,為什么銷路很好,而金鳳扒雞為什么銷路不好,這就不能解釋為什么金鳳扒雞在北京銷路不好的原因。

  24.中國自1978年改革開放以來28年間,從人口數(shù)量上說:農(nóng)民的數(shù)量并沒有減少,而是持續(xù)增加的,只不過這種增長速度相對于總?cè)丝诘脑鲩L速度來說,是比較低的。

  下列那一句話直接與上述信息矛盾?

  A.在總?cè)丝谥修r(nóng)民的數(shù)量在1978年到2006年間略微增長了。

  B.農(nóng)民在總?cè)丝谥械谋嚷蕪?978年的81.4%增加到了2006年的85.3%。

  C.中國勞動力的增長率和總?cè)丝诘脑鲩L率在1978年至今的28年里同時(shí)增加了。

  D.中國的勞動力中,農(nóng)民的比率從1978年的78%下降到了75.6%。

  解析:選項(xiàng)B可以看出,農(nóng)民相對總?cè)丝诘脑鲩L速度并不低,增長速度也是增加的。

  25.有三戶人家,每家有一孩子,他們的名字是:小萍(女)、小紅(女)、小虎。孩子的爸爸是老王、老張和老陳;媽媽是劉蓉、李玲和方麗。對于這三家人,已知:

 。1)老王家和李玲家的孩子都參加了少女舞蹈隊(duì)。

 。2)老張的女兒不是小紅。

 。3)老陳和方麗不是一家人。

  根據(jù)以上條件,確定以下哪項(xiàng)是正確的?

  A.老王、劉蓉和小萍是一家。

  B.老張、李玲和小紅是一家。

  C.老王、方麗和小紅是一家。

  D.老陳、方麗和小虎是一家。

  解析:邏輯推斷題。從條件1可以看出小虎不是老王和李玲的孩子,因?yàn)樾』⑹悄泻ⅰD敲纯梢酝瞥鲂∑己托〖t的爸爸可能是老王。根據(jù)條件2,老張的女兒不是小紅,所以可以確定老王的女兒是小萍。鎖定選項(xiàng)A,如果A正確,那么根據(jù)條件3老陳和李玲是一家,根據(jù)條件1,女兒是小紅;得出老陳、方麗和小虎是一家。那么選項(xiàng)D也正確。因此同時(shí)排除A、D。選項(xiàng)B與條件2不符,因此選擇C。

  26.5名犯罪嫌疑人被警察詢問,事后得知其中一名是罪犯。下面是五個(gè)嫌疑人的供述,其中只有3句真話。問,誰是罪犯?

  甲:丁是罪犯。

  乙:我是無辜的。

  丙:戊不是罪犯。

  。杭自谌鲋e。

  戊:乙說的是實(shí)話。

  A.戊    B.甲和丙    C.丁    D. 乙和戊

  解析:從選項(xiàng)入手,選項(xiàng)B,甲和丙是罪犯,可以推出乙、戊、丁、丙都為真,而題目表明只有3項(xiàng)為真,所以放棄B;選項(xiàng)C,丁是罪犯,可以推出甲、乙、丙、戊為真,同樣與題目要求不符;選項(xiàng)D,乙和戊是罪犯,只能推出丁為真。因此這道題選擇A。

  27.學(xué)生上完體育課后回到教室,有15人喝了飲水機(jī)里的純凈水,其中5人很快產(chǎn)生了腹瀉。飲水機(jī)里的純凈水馬上被送去檢驗(yàn),檢驗(yàn)的結(jié)果不能肯定其中有造成腹瀉的有害物質(zhì)。因此,喝了飲水機(jī)里的純凈水不是造成腹瀉的原因。

  如果上述檢驗(yàn)結(jié)果是正確的,則以下哪項(xiàng)對上述論證的評價(jià)最為恰當(dāng)?

  A.題干的論證有漏洞,因?yàn)樗鼪]有考慮到另一個(gè)事實(shí):哪些沒有喝了飲水機(jī)里的純凈水的人沒有造成腹瀉。

  B.題干的論證有漏洞,因?yàn)樗讶鄙僮C據(jù)證明某種情況存在,當(dāng)作有充分證據(jù)證明某種情況不存在。

  C.題干的論證有漏洞,因?yàn)樗鼪]有利用一個(gè)有力的論據(jù):為什么有更多人喝了飲水機(jī)里的純凈水沒有造成腹瀉。

  D.題干的論證有漏洞,因?yàn)樗鼪]有指出造成腹瀉的真正原因。

  解析:理解這道題非常簡單,舉個(gè)例子說明,如果一個(gè)人確實(shí)殺了人,但是法官沒有證據(jù)表明他是兇手,那么可能無法定罪,但終不能得出他不是兇手的結(jié)論,因?yàn)橥瑯記]有證據(jù)表明他不是兇手。題目中給出了同樣的問題,檢驗(yàn)結(jié)果不能肯定有造成腹瀉的有害物質(zhì),但是并不能說明純凈水不是造成腹瀉的原因。

  28.一項(xiàng)全球范圍的調(diào)查顯示,近 10年來:吸煙者的總數(shù)基本保持不變;每年只有10%的吸煙者改變自己的品牌,即放棄原有的品牌而改吸其他品牌;煙草制造商用在廣告上的支出占其毛收入的10%。在Z煙草公司的年終董事會上,懂事A認(rèn)為,上述統(tǒng)計(jì)表明,煙草業(yè)在廣告上的收益正好等于其支出,因此,此類廣告完全可以不做。以下哪項(xiàng),構(gòu)成對懂事A的結(jié)論的最有力質(zhì)疑?

  A.懂事A的結(jié)論忽視了:今年來各種品牌的香煙的價(jià)格都有了很大的變動。

  B.懂事A的結(jié)論基于一個(gè)錯(cuò)誤的假設(shè):每個(gè)吸煙者在某個(gè)時(shí)候只喜歡一種品牌。

  C.懂事A的結(jié)論基于一個(gè)錯(cuò)誤的假設(shè):每個(gè)煙草制造商只生產(chǎn)一種品牌。

  D.懂事A的結(jié)論忽視了:世界煙草業(yè)是一個(gè)由處于競爭狀態(tài)的眾多經(jīng)濟(jì)實(shí)體組成的。

  解析:董事A的判斷為廣告的支出正好與收益相等,因此得出廣告可以不做的結(jié)論。但是忽略了如果不做廣告是否能夠獲得與做廣告時(shí)除去廣告費(fèi)用后相同的收益,因?yàn)檫@部分收入也是由廣告產(chǎn)生的效益。因?yàn)橥袠I(yè)的其他競爭者會因?yàn)閺V告,增加10%的顧客,而不做廣告的煙草公司將有減少10%顧客的風(fēng)險(xiǎn)。因此選擇D

  29.在收款局的5個(gè)賬單收款員中,楊先生收款的不成功率最高。然而楊先生是這個(gè)局的職員中最好的賬單收款員。

  下面哪一項(xiàng),如果正確,最有助于解決上述短文中的明顯分歧?

  A.在加入收款局之前,楊先生是一個(gè)大百貨公司的信貸部的一名職員。

  B.收款局中的其他四個(gè)收款員都認(rèn)為楊先生是一個(gè)非常能干的帳單收款員。

  C.楊先生在過去的幾年內(nèi),每年收款成功的比率都保持相當(dāng)?shù)胤(wěn)定。

  D.這個(gè)收款局的大多數(shù)最困難的事情都是派楊先生去做的。

  解析:從題干入手分析,楊先生是最好的賬單收款員,但是收款不成功率最高。題目要求找到解釋這一矛盾的原因。顯然D最合適。

  30.在一次工程碩士入學(xué)考試中,據(jù)統(tǒng)計(jì),在英語、數(shù)學(xué)、語文和邏輯四個(gè)科目中:70%的考生英語及格了,75%的考生數(shù)學(xué)及格了,80%的考生語文及格了,85%的考生邏輯及格了。如果以上情況屬實(shí),那么,以下哪個(gè)選項(xiàng)一定正確:

  A.在此次考試中,大部分考生四門課都及格了。

  B.在此次考試中,大部分考生四門課沒有都及格。

  C.在此次考試中,有不少于10%的考生,四門課程都及格了。

  D.在此次考試中,至少有10%的考生,四門課程都沒及格。

  解析:不少于=大于等于,至少=大于等于。選線排除A、B,選項(xiàng)過于籠統(tǒng),并且與C、D存在重復(fù)關(guān)系。選項(xiàng)D與題干明顯不符合,選擇C。

 31.某游戲的游戲規(guī)則有這樣一條規(guī)定:任何在游戲中拒絕成為選手的人將在拒絕的時(shí)候被減掉10分。

  下列哪一個(gè)是此條規(guī)則所暗含的?

  A.同意參加游戲的所有人的得分將高于那些因?yàn)榇艘?guī)則而被判罰分的人。

  B.一個(gè)人在拒絕玩這個(gè)游戲的同時(shí),依然是這個(gè)游戲的參與者。

  C.最初同意成為游戲參加者然后在游戲進(jìn)行中退出的人可以避免被罰10分。

  D.一個(gè)拒絕玩這個(gè)游戲的人不能在游戲中被宣布為失敗者。

  解析:由于拒絕玩這個(gè)游戲,會被剪掉20分,說明依舊是游戲的參與者,選B。

  32.受到廣泛贊揚(yáng)的斯坦尼斯拉夫斯基演員訓(xùn)練法來自他做年輕演員時(shí)對戲劇不熟練和易受影響的老套做法。對該方法的理解必須基于斯坦尼斯拉夫斯基個(gè)人的演藝研究,從而從典型化的姿勢、反復(fù)嘗試的嗓音語調(diào)和標(biāo)準(zhǔn)的感情模式的誘惑中解脫出來。盡管他的美國門徒有過要求,但是這位俄國導(dǎo)演從未打算寫一本對表演問題提出嚴(yán)格解決方法的教科書。

  關(guān)于表演這個(gè)問題,上文作者對斯坦尼斯拉夫斯基的演員訓(xùn)練法持有怎樣的觀點(diǎn)?

  A.斯坦尼斯拉夫斯基的美國門徒,在使用他的方法訓(xùn)練時(shí),使得這種方法喪失了原有的靈活性和探索性。

  B.表演本質(zhì)上是自發(fā)的感情表達(dá),系統(tǒng)的訓(xùn)練經(jīng)常干涉它。

  C.實(shí)際上唯一需要給予年輕演員的建議是他們必須系統(tǒng)地抵制他們在戲劇表演中所表現(xiàn)出來的那些老套的模式。

  D.斯坦尼斯拉夫斯基的方法主要是針對那些必須克服有做作和不成熟表演行為的年輕演員。

  解析:斯坦尼斯拉夫斯基之所以未打算對表演問題提出嚴(yán)格解決方法的教科書,本意是不想使該方法失去靈活性和探索性。因此選擇A。

  33.已發(fā)現(xiàn)有國產(chǎn)嬰幼兒奶粉三聚氰胺含量超標(biāo)。

  如果上述斷定是真的,那么在下述三個(gè)斷定中不能確定真假的是:

 、、國產(chǎn)嬰幼兒奶粉沒有三聚氰胺含量不超標(biāo)的。

  Ⅱ、有國產(chǎn)嬰幼兒奶粉三聚氰胺含量沒超標(biāo)。

 、蟆⑺袊a(chǎn)嬰幼兒奶粉三聚氰胺含量都未超標(biāo)。

  A.只有Ⅰ和Ⅱ。    B.Ⅰ、Ⅱ和Ⅲ。    C.只有Ⅰ和Ⅲ。    D.只有Ⅱ。

  解析:選項(xiàng)1不能確定真假,從部分不能推出整體;選項(xiàng)2不能確定真假,從一部分不能退出另一部分具有相反的結(jié)論。條件3明顯可以判定為假,選擇A。

  34.股民購買股票的目的就是盈利。股民購買股票后,股票價(jià)格上漲,就會盈利,否則,就會虧損。一般情況下,盈利或者虧損達(dá)到10%就應(yīng)該拋出,以避免那些不可預(yù)測的突然變化所引起的損失。在股市上,有經(jīng)驗(yàn)的股民往往會選擇跟莊戰(zhàn)術(shù)來買賣股票。即大股東買進(jìn)的時(shí)候,他們也買進(jìn),大股東拋出的時(shí)候,他們就拋出。因?yàn)榇蠊蓶|有可能掌握更多上市公司和市場行情方面的信息。這些天,天虹公司的股票一直在下跌,但是大股東們卻一直在大量購進(jìn)這只股票。

  以上事實(shí)最能支持以下哪種預(yù)測?

  A.天虹公司的股票價(jià)格會繼續(xù)下降,但速度會放慢。

  B.天虹公司的股票價(jià)格會在短期內(nèi)大幅度上漲。

  C.股民盲目跟莊不是一個(gè)好的盈利方法。

  D.最近大股東對天虹公司股價(jià)變動判斷失誤,有可能造成巨大經(jīng)濟(jì)損失。

  解析:題目結(jié)論如下:大股東買進(jìn)的股票會漲,因?yàn)樗麄冋莆樟烁嗌鲜泄竞褪袌鲂星榉矫娴男畔。所以選擇B。

  35.作為身體的精密平衡系統(tǒng)的一部分,人類心臟分泌某種激素,這些激素能夠控制血液中的含鹽量和體內(nèi)參與循環(huán)的血量。人體僅需要極少的這種激素。它在控制血壓時(shí)極其重要,并且在患心臟病的病人血中經(jīng)常檢出。

  基于上面的研究成果,下面的哪個(gè)論點(diǎn)一定正確?

  A.假如心臟分泌的這種激素缺乏,將導(dǎo)致低血壓。

  B.一個(gè)僅僅由機(jī)械閥制作的設(shè)備被用作人工心臟,它將不能執(zhí)行人類心臟的所有功能。

  C.少量這種激素的分泌對人體的影響是長期性的。

  D.任何控制血壓的藥將通過影響心臟此種激素分泌量達(dá)到效果。

  解析:選擇B,因?yàn)橛蓹C(jī)械閥制作的人工心臟,顯然不具有分泌激素的功能,因此就不能執(zhí)行人類心臟的所有功能,題干反復(fù)說明就是為了得出這樣的結(jié)論。

  36.某出版社近年來出版物的錯(cuò)字率較前幾年有明顯的增加,引起了讀者的不滿和有關(guān)部門的批評,這主要是由于該出版社大量引進(jìn)非專業(yè)編輯所致。當(dāng)然,近年來該社出版物的大量增加也是一個(gè)重要原因。

  上述議論的漏洞,也類似地出現(xiàn)在以下哪項(xiàng)中?

  I 美國航空公司近兩年來的投訴率比前幾年有明顯的下降。這主要是由于該航空公司在裁員整頓的基礎(chǔ)上,有效地提高了服務(wù)質(zhì)量。當(dāng)然,“9.11”事件后航班乘客數(shù)量的銳減也是一個(gè)重要原因。

  II 統(tǒng)計(jì)數(shù)字表明:近年來我國心血管病的死亡率,即由心血管病導(dǎo)致的死亡率在整個(gè)死亡人數(shù)中的比例,較前有明顯增加,這主要是由于隨著經(jīng)濟(jì)的發(fā)展,我國民眾的飲食結(jié)構(gòu)和生活方式發(fā)生了容易誘發(fā)心血管病的不良變化。當(dāng)然,由于心血管病主要是老年病,因此,我國人口的老齡化,即人口中老年人比例的增加也是一個(gè)重要原因。

  III 某市去年的高考錄取率比前年增加15%,這主要是由于各中學(xué)狠抓了教育質(zhì)量。當(dāng)然,另一個(gè)重要原因是,該市今年參加高考的人數(shù)比去年增加了20%。

  A.僅僅I和II。   B.僅僅II和III。   C.僅僅I和III。   D.僅僅I。

  解析:題目中的邏輯錯(cuò)誤是強(qiáng)加因果,出版物數(shù)量的增加與錯(cuò)字率的增加沒有因果關(guān)系。根據(jù)這一思路對選項(xiàng)進(jìn)行判斷,選項(xiàng)I航班數(shù)銳減與投訴率的下降沒有因果關(guān)系,與題目中犯了同樣的錯(cuò)誤;選項(xiàng)II,心血管病是老年病,我國人口老齡化,因此我國心血管死亡率增加,這是一個(gè)有效地推理過程;選項(xiàng)III,參加高考的人數(shù)增加,并不能推出錄取比例增加的這一結(jié)論。也是屬于沒有因果關(guān)系。另外從絕對和相對概念這個(gè)角度分析,同樣可以得到相似的結(jié)論。選C

  37.從前,一個(gè)孤島上有一個(gè)奇怪的風(fēng)俗:凡是漂流到這個(gè)島上的外鄉(xiāng)人都要作為祭品被殺掉,但允許被殺的人在臨死前說一句話,然后由這個(gè)島上的長老判定這句話是真的還是假的。如果說的是真話,則將這個(gè)外鄉(xiāng)人在真理之神面前殺掉;如果說的是假話,則將他在錯(cuò)誤之神面前殺掉。有一天,一位哲學(xué)家漂流到了這個(gè)島上,他說了一句話,使得島上的人沒有辦法殺掉他。

  該哲學(xué)家必定說了下面哪一句話?

  A.你們這樣做不合乎理性。    B.我將死在真理之神面前。

  C.無論如何我都會死。      D.我將死在錯(cuò)誤之神面前。

  解析:A選項(xiàng),可以判斷為真,會在真理之神面前殺掉;B選項(xiàng),可以判定為真,因?yàn)樗麜涝谡胬碇衩媲埃贿x項(xiàng)C,同樣可以判定為真,選項(xiàng)D我將死在錯(cuò)誤之神面前,如果為真,那么說明他說的是假話,表明他將死在真理之神面前為真,這本身與這句話矛盾了,所以這句話自背反命題,選D。

  38.有的地質(zhì)學(xué)家認(rèn)為,如果地球的未勘探地區(qū)中單位面積的平均石油儲藏量能和已勘探地區(qū)一樣的話,那么,目前關(guān)于地下未開采的能源含量的正確估計(jì)因此要乘上一萬倍,由此可得出結(jié)論,全球的石油需求,至少可以在未來五個(gè)世紀(jì)中得到滿足,即便此種需求每年呈加速上升的趨勢。為使上述論證成立,以下哪項(xiàng)是必須假設(shè)的?

  A. 地球上未勘探地區(qū)的總面積是已勘探地區(qū)的一萬倍。

  B. 在未來至少五個(gè)世紀(jì)中,世界人口的增長率不會超過對石油需求的增長率。

  C. 新技術(shù)將使未來對石油的勘探和開采比現(xiàn)在更為可行。

  D. 地球上未勘探地區(qū)中儲藏的石油可以被勘測和開采出來。

  解析:答案為D,題目中通過假設(shè)得出了未來五個(gè)世紀(jì)的石油需求相會得到滿足的結(jié)論,假設(shè)條件為地球上未勘探地區(qū)的單位面積石油儲量與已勘探地區(qū)一樣多。但僅有這個(gè)前提還是不夠的,還需要這些為勘探的時(shí)候都是可以被開采出來的這一假設(shè)。

  39.某礦山發(fā)生了一起嚴(yán)重的安全事故。關(guān)于事故原因,甲乙丙丁四位負(fù)責(zé)人有如下斷定:

  甲:如果造成事故的直接原因是設(shè)備故障,那么肯定有人違反操作規(guī)程。

  乙:確實(shí)有人違反操作規(guī)程,但造成事故的直接原因不是設(shè)備故障

  丙:造成事故的直接原因確實(shí)是設(shè)備故障,但并沒有人違反操作規(guī)程。

  丁:造成事故的直接原因是設(shè)備故障。

  如果上述斷定只有一個(gè)人的斷定為真,那么以下斷定都不可能為真,除了:

  A.甲的斷定為真,有人違反了操作規(guī)程。

  B.甲的斷定為真,但沒有人違反操作規(guī)程。

  C.乙的斷定為真。

  D.丙的斷定為真。

  解析:這道題屬于邏輯推理題,由于只有一個(gè)人判斷為真,因此丙、丁的判斷首先排除,因?yàn)檫@兩個(gè)判定并不沖突。甲、乙的推斷有矛盾,因此兩項(xiàng)必有一個(gè)是正確的。甲的推斷結(jié)構(gòu)是如果B推出A,那么 C必然推出B;乙的推斷結(jié)構(gòu)是C推出B,但B不能退出A,是甲判定的一個(gè)逆向命題。選項(xiàng)B是正確答案,因?yàn)檫x項(xiàng)B肯定了甲的推斷,同時(shí)否定了乙的發(fā)生,因此是真命題。其他選項(xiàng)均不正確。

  40.心臟的搏動引起血液循環(huán)。對同一個(gè)人,心率越快,單位時(shí)間進(jìn)入循環(huán)的血液量越多。血液中的紅血球運(yùn)輸氧氣。一般地說,一個(gè)人單位時(shí)間通過血液循環(huán)獲得的氧氣越多,他的體能及其發(fā)揮就越佳。因此,為了提高運(yùn)動員在體育比賽中的競技水平,應(yīng)該加強(qiáng)他們在高海拔地區(qū)的訓(xùn)練,因?yàn)樵诟吆0蔚貐^(qū),人體內(nèi)每單位體積血液中含有的紅血球數(shù)量,要高于在低海拔地區(qū)。

  以下哪項(xiàng)是題干的論證必須假設(shè)的?

  A.運(yùn)動員在高海拔地區(qū)的心率不低于在低海拔地區(qū)。

  B.不同運(yùn)動員的心率基本相同。

  C.運(yùn)動員的心率比普通人慢。

  D.在高海拔地區(qū)訓(xùn)練能使運(yùn)動員的心率加快。

  解析:此題論證說明運(yùn)動員在高海拔地區(qū)訓(xùn)練體內(nèi)的紅血球數(shù)量高于低海拔地區(qū),從而達(dá)到最佳體能的目的。B和C與題意不符,選A。

 41~45題基于以下題干:

  七個(gè)學(xué)生R、S、T、V、W、X、Y,被分成兩個(gè)學(xué)習(xí)小組。第一組有三名成員,第二組有四名成員。學(xué)生們的分組必須符合以下要求:

  R和T不能在同一個(gè)小組;

  如果S在第一組,那么V必須在第一組;

  如果W在第一組,那么T必須在第二組;

  X必須在第二組。

  41.如果W在第一組,那么以下哪項(xiàng)也一定在第一組?

  A.R    B.S    C.T    D.V

  解析:根據(jù)條件第一組有三名成員。因?yàn)镽和T不能在一組,又因?yàn)闂l件:如果W在第一組,那么T必須在第二組,則R可以在第一組

  42.如果T和Y都在第一組,那么以下哪項(xiàng)一定是真的?

  A.S和V在同一組。   B.S和W在同一組。   C.V和R在同一組。   D.W和T在同一組。

  解析:如果T和Y在第一組,R和T又不能在一組,可知R只能在第二組,而X必須在第二組。如果S在第一組,根據(jù)條件,V也需在第一組,而第一組只有三個(gè)成員,那么S和V就不能在同一組。而如果W在第一組,那么T必須在第二組。但是T已經(jīng)在第一組,則W必須在第二組,加之第二組有四人,那么S也需在第二組,那么S和W為一組。

  43.如果W和T在同一組,那么以下哪項(xiàng)可能在同一組,除了:

  A.R和S    B.S和Y    C.T和Y    D.V和Y

  答案:B

  解析:

  條件分析:

  1、R1且T2;或者 R2且T1

  2、S1→V1;即S2的時(shí)候,V可以自由分配。

  3、W1→T2;即W2的時(shí)候,T可以自由分配。

  4、X2

  43題的臨時(shí)題干要求WT在同一組,根據(jù)條件3,他們不可能在第一組,不然就違背了題設(shè)。所以,這個(gè)時(shí)候,WT都在都在第二組。第二組已經(jīng)有了WTX三個(gè)成員,只有一個(gè)空位了。根據(jù)條件1,可以知道,R在第一組。因此,除了跟WTX搭伴的元素外,任何二人組合都必須在第一組了。

  選項(xiàng)A,第一組RSV;第二組WTXY,有可能。

  選項(xiàng)B,第一組RSY;則第一組還必須有V,顯然超編了。因?yàn)榈谝唤M只能有三個(gè)人。因此,此項(xiàng)為答案。

  選項(xiàng)C,依然是第一組RSV;第二組WTXY這樣的組合。

  選項(xiàng)D,第一組RVY;第二組WTXS,不矛盾。

  44.如果V和Y在同一組,那么以下哪項(xiàng)一定是真的?

  A.R在第一組。    B.S在第一組。    C.W在第二組。    D.Y在第二組。

  解析:如果V和Y在第一組,那么根據(jù)條件如果S在第一組,那么V必須在第一組;說明S可以在第一組,又因?yàn)槿绻鸚在第一組,那么T必須在第二組,但是R和T又不能在一組,則R必須在第一組,W和T可以在第二組。則答案C正確。

  45.如果S在第一組,那么以下哪項(xiàng)一定是真的?

  A.T在第一組。    B.T在第二組。    C.Y在第一組。    D.Y在第二組。

  解析:如果S在第一組,根據(jù)條件V必須在第一組,如果W在第一組,那么T必須在第二組;R和T不能在同一個(gè)小組;那么W或R可以在第一組,X必須在第二組。因此在第一組的可以是S或V或R或W,因此答案D正確。

  46~50題基于以下題干:

  六位教授F、G、H、J、K、L,將評審馬、任、孫、吳博士的論文4篇。評審需遵守以下原則:

  (1)每位教授只評審一篇博士論文;

 。2)每篇博士論文至少有一位教授評審;

  (3)H與F評審?fù)黄┦空撐模?/p>

 。4)L只與其他教授中的一位同評一篇博士論文;

 。5)G評審馬博士的論文;

  (6)J評審馬博士或吳博士的論文;

  (7)H不評審吳博士的論文。

  46.如果K不評審孫博士的論文,那么以下哪項(xiàng)一定是真的?

  A.L評審馬博士的論文           B.L評審孫博士的論文

  C.F和H評審任博士的論文        D.F和H評審孫博士的論文

  解析:H不評審吳博士論文,H與F又評審?fù)黄撐;那么H和F有可能評審馬或任或?qū)O博士的論文,但是G評審馬博士的論文,J評審馬博士或者吳博士的論文,那么H與F就有可能評審任或?qū)O博士的論文,K不評審孫博士的論文,K就有可能評審任博士的論文,那么H和F就會評審孫博士的論文。

  47.以下哪項(xiàng)完整地列出了L可能評審的博士論文?

  A.馬、任      B.馬、孫、吳     C.馬、任、吳       D.馬、任、孫、吳

  解析:根據(jù)條件四,L只與其他教授中的一位同評一篇博士論文,而每篇博士論文至少有一位教授評審,也就是說一篇論文有可能是兩個(gè)教授或者三個(gè)教授一起評論。因此,L可以評論馬、任、孫、吳的博士論文。

  48.以下哪項(xiàng)可能是真的?

  A.F和G評審馬博士的論文。

  B. F和L評審吳博士的論文。

  C. K評審吳博士的論文并且L評審馬博士的論文。

  D. L評審任博士的論文并且F評審孫博士的論文。

  解析:G評審馬博士的論文,J評審馬博士或吳博士的論文,H不評審吳博士的論文,H與F又評審?fù)黄撐;那么H和F有可能評審馬或任或?qū)O博士的論文,那么H與F就有可能評審任或?qū)O博士的論文。加之題一,K不評審孫博士的論文,K就有可能評審任博士的論文,條件三H與F評審?fù)黄┦空撐,那么H和F就會評審孫博士的論文。條件四:L只與其他教授中的一位同評一篇博士論文,L可以與K評審任博士論文,故D正確

  49.以下哪項(xiàng)不可能是真的?

  A. L和G評審馬博士的論文        B. L和K評審馬博士的論文

  C. L和K評審任博士的論文        D. L和K評審孫博士的論文

  解析:根據(jù)條件(4)L只與其他教授中的一位同評一篇博士論文,而條件(5)G評審馬博士的論文,那么可知L和G可評審馬博士論文,故A正確。根據(jù)條件(6)J評審馬博士或吳博士的論文,可以得知L可以和J可以評審吳博士或者馬博士的論文,H不評審吳博士的論文,那么H可以評論任博士和孫博士的論文。K可以評論吳博士、任博士、孫博士的論文。加之條件(4),得知答案C和D正確,故B錯(cuò)誤

  50.如下哪位教授不可能審閱任博士的論文?

  A.F   B.K    C.J   D. L

  解析:條件(6)J評審馬博士或吳博士的論文;而條件(1)每位教授只評審一篇博士論文,根據(jù)兩個(gè)條件可以推斷J不可能審閱任或?qū)O博士的論文,故選C

  模擬題1參考答案:

01. C 02. A 03. C 04. D 05. C
06. C 07. A 08. B 09. A 10. A
11. A 12. B 13. B 14. A 15. C
16. D 17. D 18. C 19. A 20. B
21. C 22. C 23. B 24. B 25. C
26. A 27. B 28. D 29. D 30. C
31. B 32. A 33. A 34. B 35. B
36. C 37. D 38. D 39. B 40. A
41. A 42. B 43. B 44. C 45. D
46. D 47. D 48. D 49. B 50. C
 

 

 

 

(50題,每題2分,滿分100分)

  1.任何人都沒有吃過雅各島上的任何水果,所以無法知道雅各島上任何水果的口味。

  為了合乎邏輯的推出上述結(jié)論,需要假設(shè)下面哪項(xiàng)為前提?

  A.如果一種水果有人品嘗過,就可以知道其口味。(充分條件,不是前提)

  B.只憑某些人的品嘗無法真正知道某種水果的口味。

  C.要知道某種水果的口味,需要有人去品嘗。

  D.人們是通過嗅覺來確定水果口味的。

  解析:這道題需要補(bǔ)全假設(shè),結(jié)論是:任何人都無法知道任何水果的味道,前提是:任何人都沒有吃過雅各島上的任何水果。補(bǔ)充前提是:要知道水果的口味,需要有人去品嘗,應(yīng)該選擇C。選項(xiàng)A,是充分條件,但不是構(gòu)成結(jié)論的原因,屬于強(qiáng)拉因果;B、D都為不合理的假設(shè)。

  2.最近的一項(xiàng)研究指出:“經(jīng)常吃沙棘果對兒童的智力發(fā)育有益。”研究人員對560名兒童進(jìn)行調(diào)查,發(fā)現(xiàn)那些經(jīng)常吃沙棘果的兒童,其智力水平較很少吃沙棘果的兒童要高。因此,研究人員發(fā)現(xiàn)了沙棘果與兒童智力發(fā)育之間的聯(lián)系。

  以下哪項(xiàng)如果為真,最不可能削弱上述論證?

  A.對成年人的研究發(fā)現(xiàn),每天吃沙棘果的人智力水平并不比很少吃沙棘果人的高。

  B.調(diào)查顯示:沙棘果價(jià)格非常高,只有富裕家庭的兒童才經(jīng)常吃,同時(shí)這些家庭有條件實(shí)現(xiàn)兒童的早期智力開發(fā)。

  C.這項(xiàng)兒童發(fā)育研究的課題負(fù)責(zé)人是沙棘果生產(chǎn)商,其目的就是要擴(kuò)展沙棘果的銷售渠道。

  D.沙棘果是兒童喜歡的食品,家長經(jīng)常把沙棘果作為禮物獎(jiǎng)給智力表現(xiàn)優(yōu)異的孩子。

  解析:吃沙棘果與兒童智力發(fā)展是否存在必然聯(lián)系。通過樣本調(diào)查判斷出吃沙棘果與兒童智力發(fā)展存在正相關(guān)的關(guān)系。B選項(xiàng)說明家庭條件是否富裕與兒童智力發(fā)展存在關(guān)聯(lián)聯(lián)系;C選項(xiàng)說明題目中的結(jié)論是由于生產(chǎn)商的廣告產(chǎn)生的以偏概全的結(jié)論;D選項(xiàng)屬于因果倒置,成績優(yōu)異是因,吃沙棘果是結(jié)果。

  3.凡金屬都是導(dǎo)電的。銅是導(dǎo)電的,所以銅是金屬。

  下面哪項(xiàng)與上述推理結(jié)構(gòu)最相似?

  A.所有的鳥都是卵生動物,蝙蝠不是卵生動物,所以,蝙蝠不是鳥。

  B.所有的鳥都是卵生動物,天鵝是鳥,所以天鵝是卵生動物。

  C.所有從事工商管理的都要學(xué)習(xí)企業(yè)管理,老陳是學(xué)習(xí)企業(yè)管理的,所以,老陳是從事工商管理工作的。

  D.華山險(xiǎn)于黃山,黃山險(xiǎn)于泰山,所以華山險(xiǎn)于泰山。

  解析:題目結(jié)構(gòu)是所有的A具有某中屬性,B屬于A,因此B具有A的屬性,選項(xiàng)C與上述結(jié)論類似。

  4.為了有助于人們選擇最滿意的城市居住,有關(guān)部門實(shí)施了一項(xiàng)評選“最舒適城市”的活動。方法是,選擇十個(gè)方面,包括社會治安、商業(yè)設(shè)施、清潔程度、綠化程度、教育設(shè)施、旅游文化景點(diǎn)等等,每個(gè)方面按實(shí)際質(zhì)量的高低,評以1分至10分之間的某一分值,然后求得十個(gè)分值的平均數(shù)即是這個(gè)城市的舒適性指數(shù)。

  以下哪項(xiàng)是實(shí)施上述活動需要預(yù)設(shè)的前提?

  Ⅰ、城市的各種舒適性質(zhì)量程度都可以用準(zhǔn)確的數(shù)字表達(dá)。

 、颉⒊鞘械母鞣N舒適性對于居民來說都是同等重要的。

  Ⅲ、居民有自由選擇居住城市的權(quán)利并且大都樂于這樣做。

  A.僅Ⅰ。      B.僅Ⅲ。      C.僅Ⅰ和Ⅱ。      D.Ⅰ、Ⅱ和Ⅲ。

  解析:這道題是補(bǔ)充假設(shè)的問題,由于每個(gè)人對“舒適程度”的感覺是不一樣的,因此題干結(jié)論若要正確,條件Ⅱ是必須的前提;若題目中結(jié)論正確,需要假設(shè)題目中的十個(gè)指標(biāo)存在量化標(biāo)準(zhǔn),因此條件Ⅰ為必選項(xiàng);條件Ⅲ存在爭議,不過可以從另外一個(gè)角度理解,便于理解題目的結(jié)論,如果居民沒有選擇居住地的權(quán)利,那么題目中的滿意度調(diào)查就沒有意義,所以也得不到上述結(jié)論。

  5.衛(wèi)生部的官員們對牟定縣狂犬病疫情有以下斷定:

  (1)該縣所有的狗都得了狂犬病。

  (2)該縣有些斑點(diǎn)狗得了狂犬病。(真)

  (3)該縣有些狗得了狂犬病。

  (4)該縣有些狗沒得狂犬病。(真)

  其實(shí)上述斷定中只有兩個(gè)與事實(shí)相符。根據(jù)如上的情況,以下哪項(xiàng)結(jié)論是可能成立的?

  I.該縣的狗都是斑點(diǎn)狗。

  II. 該縣沒有斑點(diǎn)狗可能得狂犬病。

  III. 該縣的斑點(diǎn)狗并非可能沒有得狂犬病。

  IV. 該縣的狗不可能都沒得狂犬病。

  A.僅I      B.僅III      C.僅II和IV      D.僅IV。

  解析:這道題屬于邏輯推理中真假話題的問題。解析這種問題按照1、先找矛盾;2、繞開矛盾;3、推出答案的步驟進(jìn)行。首先先找矛盾,首先1和2是一對矛盾,3和4是一對矛盾,只有一個(gè)正確,2、繞開矛盾,如果1正確,那么3和4都假,因此只能2為真,如果2為真,只能判斷4為真,3為假,3、推出答案,條件2有些斑點(diǎn)夠得了狂犬病,可推出該縣沒有斑點(diǎn)狗可能得狂犬病,大家注意必然=沒有……必然;同時(shí)根據(jù)條件4可以得出該縣的狗不可能都沒得狂犬病。

  6.如果一個(gè)兒童的體重與身高的比值超過本地區(qū)80%兒童的水平,就稱其為肥胖兒。根據(jù)歷年的調(diào)查結(jié)果,15年來,臨江市的肥胖兒的數(shù)量一直在穩(wěn)定增長。

  如果以上斷定為真,則以下哪項(xiàng)也必為真?

  A.臨江市每一個(gè)肥胖兒的體重都超過全市兒童的平均體重。

  B.15年來,臨江市的兒童的體育鍛煉越來越不足。

  C.臨江市的非肥胖兒的數(shù)量15年來不斷增長。

  D.15年來,臨江市體重不足標(biāo)準(zhǔn)體重的兒童數(shù)量不斷下降。

  解析:這道題屬于迷惑推理題,大家務(wù)必要小心。首先大家可以看出臨江市兒童數(shù)量是在增長的;兒童由肥胖和非肥胖組成;肥胖兒童的數(shù)量穩(wěn)定增長,說明非肥胖兒童的數(shù)量也在穩(wěn)定增長;沒有證據(jù)表明非肥胖兒童的數(shù)量在不斷下降。

  7.孩子出生后的第一年在托兒所里度過,會引發(fā)孩子的緊張不安。在我們的研究中,有464名12~13歲的兒童接受了特異情景測試法的測驗(yàn),該項(xiàng)測驗(yàn)意在測試兒童1歲時(shí)的狀況與對母親的依附心理之間的關(guān)系。其結(jié)果:有41.5%曾在托兒所看護(hù)的兒童和25.7%曾在家看護(hù)的兒童被認(rèn)為緊張不安,過于依附母親。

  以下哪項(xiàng)如果為真,最沒有可能對上述研究的推斷提出質(zhì)疑?

  A.出生后第一年在家看護(hù)的孩子多數(shù)是由祖父母或外祖父母看護(hù)的,并形成濃厚的親情。

  B.這項(xiàng)研究的主持者被證實(shí)曾經(jīng)在自己的幼兒時(shí)期受到過長時(shí)間來自托兒所阿姨的冷落。

  C.針對孩子母親的另一部分研究發(fā)現(xiàn):由于孩子在家里表現(xiàn)出過度的依附心理,父母因此希望將其送入托兒所予以矯正。

  D.研究中所測試的孩子并不是從托兒所看護(hù)和在家看護(hù)兩種情況下隨機(jī)抽取的。因此,這兩組樣本兒童的家庭很可能有系統(tǒng)性的差異存在。

  解析:這種題最好的解決辦法是否定推斷,把你認(rèn)為可能的答案否定,帶入題目中判斷。如果能夠提出質(zhì)疑,說明你的判斷是正確的。A選項(xiàng)。

  8.在玉米地中套種的小麥有可能得小麥黃葉病,這種病是由于光照不足造成的,一旦光照充足,比如減少玉米植株數(shù)量,或者玉米收割后,癥狀很快就會消失,且不會影響產(chǎn)量。而小麥銹病是由花斑銹菌引起,如果在病癥初起時(shí)不及時(shí)恰當(dāng)應(yīng)對,就會使小麥植株很快枯死。由于小麥銹病和小麥黃葉病初期癥狀很難區(qū)分,所以,在玉米地中套種小麥,必須高度注意小麥銹病的定期篩查和預(yù)防。

  以下哪項(xiàng)最可能是上述論證所假設(shè)的?

  麥黃葉病不會誘發(fā)小麥銹病。

  B.花斑銹菌不會因?yàn)槌渥愎庹斩π←湹那趾αΑ?/p>

  C.小麥銹病如果及時(shí)恰當(dāng)處理,就不會使小麥減產(chǎn)。

  D.科學(xué)家一直沒有找到有效遏制小麥銹病的藥物。

  解析:假設(shè)推斷。陽光充足,小麥黃葉病會消失,產(chǎn)量不影響;花斑銹菌,使小麥植株枯死。因此需要假設(shè)陽光充足不會抑制花斑銹菌的發(fā)病,才能得出小麥黃葉不發(fā)病時(shí),花斑銹菌仍然會發(fā)病,導(dǎo)致兩種病情難以區(qū)分。換一種角度,如果小麥黃葉病不會和花斑銹菌同時(shí)發(fā)病,那么就不存在難以區(qū)分病情的問題了。

  9.公司準(zhǔn)備從五個(gè)業(yè)務(wù)骨干中選幾個(gè)人去中央戲劇學(xué)院進(jìn)修。鑒于業(yè)務(wù)員們的工作關(guān)系,總經(jīng)理建議:

 。1)如果選鞏俐,那么必須選章子怡并且不能選周迅。

 。2)如果選章子怡或者選周迅,則不能選吳孟達(dá)。

  (3)不能既不選周星馳也不選吳孟達(dá)。

  考慮總經(jīng)理的建議,董事會認(rèn)為,鞏俐必須去中央戲劇學(xué)院進(jìn)修,這樣誰將跟她一起去進(jìn)修?

  A.章子怡和周星馳    B.章子怡和吳孟達(dá)   C.周星馳和周迅    D.吳孟達(dá)和周迅

  解析:邏輯推斷。如果鞏麗必須去,根據(jù)條件1章子儀必須去,而周迅一定不去,從而排除C、D選項(xiàng),根據(jù)條件2,如果選章子儀,那么吳孟達(dá)不能去。排除B 。

  繼續(xù)進(jìn)行推理,根據(jù)條件3必須在周星馳和吳孟達(dá)中選一個(gè),因?yàn)楦鶕?jù)條件2已經(jīng)排除吳孟達(dá),所以周星馳必須去。選擇A。

  10.20世紀(jì)60年代早期之前,挪威的斯塔溫格是一個(gè)安靜而和平的小鎮(zhèn)。進(jìn)入60年代以來,這里成為了挪威近海石油勘探中心。從此,暴力犯罪和毀壞公物的現(xiàn)象在斯塔溫格小鎮(zhèn)急劇增加。顯然這些社會問題是是石油繁榮的副產(chǎn)品。

  下列哪一個(gè)選項(xiàng)最強(qiáng)烈的支持上面的論述。

  A.暴力犯罪和毀壞公物在沒有石油繁榮的挪威城鎮(zhèn)保持低水平。

  B.斯塔溫格居民很少對這里是近海石油勘探中心感到遺憾。

  C.挪威社會學(xué)家對斯塔溫格的暴力犯罪和毀壞公物現(xiàn)象持續(xù)增加表示關(guān)切。

  D.非暴力犯罪、毒品、離婚,在這個(gè)小鎮(zhèn)也以同樣的速度增加著。

  解析:題目把社會問題的原因歸結(jié)為石油繁榮,最強(qiáng)烈支持的論據(jù)為在沒有石油繁榮的時(shí)期,社會問題處于低水平。因此選A。

11.認(rèn)為只傷害自己而沒有傷害到別人的行為并沒有什么錯(cuò)誤的想法,通常伴隨著對人與人之間相互依賴的人際關(guān)系的忽視。毀壞一個(gè)人自己的健康或者生命意味著不能你將不能為家庭或者社會提供幫助,相反意味著你將額外享有那些本來就有限的諸如食物、健康服務(wù)和教育等社會資源,而不是相反將它們回報(bào)給社會。

  以下哪個(gè)選項(xiàng)如果正確,最強(qiáng)烈的地支持題干所表達(dá)的觀點(diǎn)。

  A.本來可以避免的疾病和意外事故所造成的花銷增加了每個(gè)人的健康保險(xiǎn)金。

  B.傷害一個(gè)人能夠?qū)е麻g接的利益,諸如可以使別人獲得與健康領(lǐng)域相關(guān)聯(lián)的工作。

  C.一個(gè)人對社會所作的貢獻(xiàn)可以通過他的健康程度來衡量。

  D.由喝酒、吸煙和吸食毒品所導(dǎo)致的主要傷害是由使用那些東西的人來承受的。

  解析:題目中的結(jié)論是傷害自己的行為與傷害別人的行為同樣都是錯(cuò)誤的。原因是傷害自己將不能為家庭和社會提供幫助,同時(shí)會向社會索取的更多。這道題的關(guān)鍵是大家要讀懂題干,這樣從選項(xiàng)中我們不難選出A,本來可以避免的個(gè)人傷害回增加健康保險(xiǎn)金,從而增加社會的負(fù)擔(dān)。

  12.大多數(shù)道路的修理比預(yù)算的要花費(fèi)更多的時(shí)間和金錢,但是去年夜間修理京津塘高速公路和類似的道路并未比預(yù)算花費(fèi)更多的時(shí)間或金錢。因此,在夏季,夜間修理主要道路可能省時(shí)省錢。

  下列哪一個(gè),假如正確,最支持上面得到的結(jié)論?

  A.修理京津塘高速公路的預(yù)算足夠用,所以不可能超過預(yù)算。

  B.夏季,夜間路上通行車輛較少,而且溫度較舒適,允許修路工人工作得更快。

  C.愿意在晚上工作的修路工人較容易找到工作機(jī)會,因?yàn)榇蠖鄶?shù)人寧愿白天工作。

  D.用于道路修理的瀝青在較高溫度下膨脹,在溫度降低的時(shí)候收縮。

  解析:補(bǔ)充論據(jù)。從題干可以看出從特殊樣本推出結(jié)論明顯論據(jù)不足。夏天夜間施工本來和省時(shí)省錢本來沒有關(guān)系,屬于不相關(guān)關(guān)系,因此需要補(bǔ)充適當(dāng)?shù)恼摀?jù),在這兩個(gè)問題中間搭起橋梁,建立關(guān)系,所以選擇B。

  13.偏頭痛一直被認(rèn)為是由食物過敏引起的。但是,如果讓患者停止食用那些已經(jīng)證明會不斷引起過敏性偏頭痛的食物,他們的偏頭痛并沒有停止,因此,顯然存在別的某種原因引起偏頭痛。下列哪項(xiàng)如果為真,最能削弱上面的結(jié)論?

  A.許多患者說誘發(fā)偏頭痛病的那些食物往往是他們最喜歡吃的食物。

  B.許多普通食物只在食用幾天后才誘發(fā)偏頭痛,因此,不容易觀察患者的過敏反應(yīng)和他們食用的食物之間的關(guān)系。

  C.很少有食物過敏會引起像偏頭痛那樣嚴(yán)重的癥狀。

  D.許多不患偏頭痛的人同樣有食物過敏反應(yīng)。

  解析:題目結(jié)論是過敏引發(fā)偏頭痛,但是引發(fā)過敏的問題不僅僅是食物引起的。結(jié)論指明食物過敏與偏頭痛之間不存在這因果關(guān)系,原因是停用了這種食物之后,偏頭痛的病情仍然存在,若要削弱這個(gè)結(jié)論,需要針對上述論據(jù)提出反對。本著這個(gè)思路我們看選項(xiàng),結(jié)論B。表明了停用食物之后仍然存在偏頭痛的原因是因?yàn)樵S多普通食物在食用幾天后才發(fā)作,因此雖然停用了食物,仍會發(fā)作一段時(shí)間。

  14.世界衛(wèi)生組織在全球范圍內(nèi)進(jìn)行了一項(xiàng)有關(guān)獻(xiàn)血對健康的影響的跟蹤調(diào)查。調(diào)查對象分為三組。第一組對象中均有二次以上的獻(xiàn)血記錄,其中最多的達(dá)數(shù)十次;第二組中的對象均僅有一次獻(xiàn)血記錄;第三組對象均從未獻(xiàn)過血。調(diào)查結(jié)果顯示,被調(diào)查對象中癌癥和心臟病的發(fā)病率,第一組分別為0.3%和0.5%,第二組分別為0.7%和0.9%,第三組分別為1.2%和2.7%。一些專家依此得出結(jié)論,獻(xiàn)血有利于減少患癌癥和心臟病的風(fēng)險(xiǎn)。這兩種病已經(jīng)不僅在發(fā)達(dá)國家而且在發(fā)展中國家成為威脅中老年人生命的主要?dú)⑹。因此,獻(xiàn)血利己利人,一舉兩得。以下哪項(xiàng)如果為真,將削弱以上結(jié)論?

  I 60歲以上的調(diào)查對象,在第一組中占60%,在第二中占70%,在第三組中占80%。

  II 獻(xiàn)血者在獻(xiàn)血前要經(jīng)過嚴(yán)格的體檢,一般具有較好的體質(zhì)。

  III 調(diào)查對象的人數(shù),第一組為1700人,第二組為3000人,第三組為7000人。

  A.只有I和II。    B.只有II和III。    C.只有I和III    D.只有I

  解析:題目中對獻(xiàn)血與癌癥、心臟病的發(fā)病幾率建立了正相關(guān)關(guān)系。但是樣本仍然存在問題,如果鮮血的人本身身體素質(zhì)就比沒有獻(xiàn)血的身體素質(zhì)好,就不能退出上述結(jié)論。這是最簡單容易想出的悖論。因此淘汰C、D。下面在條件1和3中選一個(gè),條件3說明樣本容量不同,但是題目中使用的是百分?jǐn)?shù)這個(gè)相對指標(biāo),因此不能形成反對結(jié)論的悖論。選A。

  15.許多孕婦都出現(xiàn)了維生素缺乏的癥狀,但這通常不是由于孕婦的飲食中缺乏維生素,而是由于腹內(nèi)嬰兒的生長使她們比其他人對維生素有更高的需求。為了評價(jià)上述結(jié)論的確切程度,以下哪項(xiàng)操作最為重要?

  A.對某個(gè)不缺乏維生素的孕婦的日常飲食進(jìn)行檢測,確定其中維生素的含量。

  B.對孕婦的科學(xué)食譜進(jìn)行研究,以確定有利于孕婦攝入足量維生素的最佳選擇。

  C.對日常飲食中維生素足量的一個(gè)孕婦和一個(gè)非孕婦進(jìn)行檢測,并分別確定她們是否缺乏維生素。

  D.對日常飲食中維生素不足量的一個(gè)孕婦和另一個(gè)非孕婦進(jìn)行檢測,并分別確定她們是否缺乏維生素。

  解析:支持論據(jù)。題目需要根據(jù)結(jié)論找出支持論據(jù),顯然對同量的維生素給孕婦和非孕婦服用,觀察他們是否缺乏維生素,對得出結(jié)論最重要。選C

  16.經(jīng)A省的防疫部門檢測,在該省境內(nèi)接受檢疫的長尾猴中,有1%感染上了狂犬病。但是只有與人及其寵物有接觸的長尾猴才接受檢疫。防疫部門的專家因此推測,該省長尾猴中感染有狂犬病的比例,將大大小于1%。

  以下哪項(xiàng)如果為真,將最有力地支持專家的推測?

  A.在A省境內(nèi),與人及其寵物有接觸的長尾猴,只占長尾猴總數(shù)的不到10%。

  B.在A省,感染有狂犬病的寵物,約占寵物總數(shù)的0.1%。

  C.在與A省毗鄰的B省境內(nèi),至今沒有關(guān)于長尾猴感染狂犬病的疫情報(bào)告。

  D.與健康的長尾猴相比,感染有狂犬病的長尾猴更愿意與人及其寵物接觸。

  解析:選擇D,其他選項(xiàng)均不能支持題干給出的結(jié)論。在與人和動物接觸的長尾猴進(jìn)行檢驗(yàn),發(fā)現(xiàn)1%感染了狂犬病,推斷出該省長尾猴感染狂犬病的比例小于1%,那么只有感染狂犬病的長尾猴更喜歡與人和動物接觸能夠有效地支持這個(gè)結(jié)論。

  17.作為市電視臺的攝像師,最近國內(nèi)電池市場的突然變化讓我非常頭疼。進(jìn)口電池缺貨,我只能用國產(chǎn)電池來代替作為攝像的主要電源。盡管每單位的國產(chǎn)電池要比進(jìn)口電池便宜,但我估計(jì)如果持續(xù)用國產(chǎn)電池替代進(jìn)口電池來提供同樣的電源供應(yīng)的話,我在能源上的支付將會提高。

  說這番話的人在上面這段話中隱含了以下哪項(xiàng)假設(shè)?

  A. 持續(xù)使用國產(chǎn)電池,攝像的質(zhì)量將無法得到保障。

  B. 每單位的進(jìn)口電池要比國產(chǎn)電池價(jià)格貴。

  C. 生產(chǎn)國產(chǎn)電池要比生產(chǎn)進(jìn)口電池成本低。

  D. 以每單位電池提供的電能來計(jì)算,國產(chǎn)電池要比進(jìn)口電池提供得少。

  解析:這是日常生活中經(jīng)常會遇到的問題,答案一目了然選擇D。得出結(jié)論的原因是盡管國產(chǎn)電池比進(jìn)口電池單價(jià)便宜,但是就單位電能來講,國產(chǎn)電池的價(jià)格卻比進(jìn)口電池高。

  18.最近由于氣候的異常變化,使得山東省的大蒜產(chǎn)量受到較大影響而減產(chǎn)。山東大蒜的價(jià)格比平時(shí)同期上漲了兩倍,這就大大提高了大蒜素生產(chǎn)的成本,估計(jì)未來大蒜素的市場價(jià)格將有大幅度的提高。以下哪項(xiàng)如果是真的,最能削弱上述結(jié)論?

  A.去年大蒜的價(jià)格是歷年最低的。

  B.其他替代原料可以用來生產(chǎn)人工大蒜素。

  C.除了山東省外,其他省份也可以提供大蒜。

  D.最近的天氣異常不如專家們估計(jì)的那么嚴(yán)重。

  解析:題目中的推斷是沒有問題的,成本的提高會促使價(jià)格的提高,但是其他省也提供大蒜,而成本并沒有提高,價(jià)格不會上升,在這樣的市場上,山東省的大蒜盡管成本提高了,但是價(jià)格將會保持市場水平,因此C有效地消弱了結(jié)論。

  19.某學(xué)校開設(shè)選修課,規(guī)定:同學(xué)只有選了文化史課的同學(xué)才能選古典音樂課。如果選古典音樂課,那他就不能選文學(xué)欣賞課。如果選文學(xué)欣賞課,就不能選古典音樂課。

  如果以上陳述為真,以下哪項(xiàng)陳述不可能假?

  A.同學(xué)不選文學(xué)欣賞課 或者不選古典音樂課。

  B.同學(xué)選文學(xué)欣賞課 或者不選古典音樂課。

  C.同學(xué)不選古典音樂課,或者不選文化史課。

  D.同學(xué)選古典音樂課,或者不選文學(xué)欣賞課

  解析:邏輯推斷題。不可能假=一定真,因此題目要求在選項(xiàng)中找出一定真的項(xiàng)。選了文化史,才能選古典音樂,這是個(gè)必要條件;但是選了古典音樂就不能選文學(xué)欣賞,二選其一;選項(xiàng)A的論述與題干一樣。

  20.20年前,幾乎所有的公司首腦在選擇重新設(shè)置公司總部的時(shí)候,主要關(guān)心的是土地價(jià)格。今天一個(gè)高級執(zhí)行官計(jì)劃重設(shè)總部時(shí)主要關(guān)心的東西更廣泛了,經(jīng)常包括當(dāng)?shù)氐膶W(xué)校和住房質(zhì)量。

  假如上述信息可靠,下面哪一項(xiàng)最好地解釋了公司首腦們主要關(guān)心問題的變化?

  A.20年前高質(zhì)量的住房和學(xué)校像今天一樣難以發(fā)現(xiàn)。

  B.近年來優(yōu)秀專業(yè)辦公人員缺乏的問題迫使公司找到盡可能多的方法來留住老員工,并且吸引優(yōu)秀新員工的加盟。

  C.公司執(zhí)行官總是考慮自己的決定將怎樣影響公司的利潤。

  D.在過去20年,一些地區(qū)比其他地區(qū)土地價(jià)格變化少。

  解析:選項(xiàng)A屬于強(qiáng)拉因果,不存在相關(guān)關(guān)系;選項(xiàng)B說明了高級執(zhí)行官在公司選址時(shí)關(guān)注學(xué)校和住房質(zhì)量的原因,分別是吸引優(yōu)秀新員工加盟和留住老員工;選項(xiàng)C與題干沒有關(guān)系,并未涉及到利潤這個(gè)因素;選項(xiàng)D,最多可以說明高級執(zhí)行官不在關(guān)心土地價(jià)格的原因,但是并不能說明關(guān)注學(xué)校和住房質(zhì)量的原因。

21.用蒸餾麥芽渣提取的酒精作為汽油的替代品進(jìn)入市場,使得糧食市場和能源市場發(fā)生了前所未有的直接聯(lián)系。到1995年,谷物作為酒精的價(jià)值己經(jīng)超過了作為糧食的價(jià)值。西方國家已經(jīng)或正在考慮用從谷物提取的酒精來替代一部分進(jìn)口石油。

  如果上述斷定為真,對于那些己經(jīng)用從谷物提取的酒精來替代一部分進(jìn)口石油的西方國家。以下哪項(xiàng),最可能是1995年后進(jìn)口石油價(jià)格下跌的后果?

  A.一些谷物從能源市場轉(zhuǎn)入糧食市場。

  B.一些谷物從糧食市場轉(zhuǎn)入能源市場。

  C.谷物的價(jià)格面臨下跌的壓力。

  D.谷物的價(jià)格出現(xiàn)上浮。

  解析:谷物可以提取出酒精來代替石油出口,因此可以判斷谷物和石油成為了相互競爭的替代品,因此如果石油價(jià)格下跌,那么谷物的價(jià)格也必然會下調(diào),否則進(jìn)口國將會進(jìn)口價(jià)格低廉的石油取代谷物。

  22.有八個(gè)教授參加學(xué)術(shù)會議后,互留通信地址,以保持聯(lián)絡(luò)。一年后,統(tǒng)計(jì)他們之間的通信情況如下:有一人給其他三個(gè)人寫過信,給其他人寫過兩封信的有三人,給其他人寫過一封信的有四個(gè)人。若以上統(tǒng)計(jì)屬實(shí),則最能得出以下哪項(xiàng)結(jié)論?

  A.通過書信來往,他們遞進(jìn)了友誼,也加強(qiáng)了學(xué)術(shù)聯(lián)系。

  B.八個(gè)教授都收到過其他教授的來信。

  C.至少有一個(gè)人收到信件后沒有都回復(fù)。

  D.雖然參加學(xué)術(shù)會議的時(shí)候大家不熟悉,但現(xiàn)在他們已經(jīng)成為至交。

  解析:選項(xiàng)首先排除A、D,沒有任何證據(jù)表明通過書信加強(qiáng)了學(xué)術(shù)聯(lián)系,也沒有證據(jù)表明他們成為了至交。下面看選項(xiàng)B、C,我們可以通過畫圖的方法來解決這個(gè)問題相對比較簡單,從而得出正確答案為C。

  23.金鳳扒雞是一個(gè)享有盛譽(yù)的百年扒雞品牌,在石家莊銷路很好,深受當(dāng)?shù)匕傩障矏郏刻煲簧鲜芯蜁粨屬徱豢。為了打開北京市場,金鳳扒雞改進(jìn)了工藝,用真空包裝,延長了保質(zhì)期,試圖在北京各大超市打開銷路,但是金鳳扒雞在北京卻很難銷售出去。

  下面除了哪一項(xiàng),都有助于解決上述現(xiàn)象?

  A.北京人和石家莊人的飲食習(xí)慣不同。他們更喜歡吃烤鴨而不是扒雞。

  B.恒慧通燒雞也采用真空包裝,保質(zhì)期一樣長,在北京的銷路卻很好。

  C.人們喜歡吃剛出鍋的新鮮扒雞,保質(zhì)期短的反而更受消費(fèi)者青睞。

  D.雖然金鳳扒雞是百年品牌,但是它的銷售地點(diǎn)僅在石家莊市區(qū),北京人對這個(gè)品牌不熟悉。

  解析:這是一道頗具爭議的題目,選項(xiàng)A、B、C、D分別從消費(fèi)者的喜好,競爭者,消費(fèi)偏好和品牌對金鳳扒雞在北京銷路受阻提供了良好的解釋。但是選項(xiàng)B可以從另外一個(gè)角度思考,恒惠通燒雞同樣采用真空包裝,保質(zhì)期一樣長,什么都和金鳳扒雞一樣,為什么銷路很好,而金鳳扒雞為什么銷路不好,這就不能解釋為什么金鳳扒雞在北京銷路不好的原因。

  24.中國自1978年改革開放以來28年間,從人口數(shù)量上說:農(nóng)民的數(shù)量并沒有減少,而是持續(xù)增加的,只不過這種增長速度相對于總?cè)丝诘脑鲩L速度來說,是比較低的。

  下列那一句話直接與上述信息矛盾?

  A.在總?cè)丝谥修r(nóng)民的數(shù)量在1978年到2006年間略微增長了。

  B.農(nóng)民在總?cè)丝谥械谋嚷蕪?978年的81.4%增加到了2006年的85.3%。

  C.中國勞動力的增長率和總?cè)丝诘脑鲩L率在1978年至今的28年里同時(shí)增加了。

  D.中國的勞動力中,農(nóng)民的比率從1978年的78%下降到了75.6%。

  解析:選項(xiàng)B可以看出,農(nóng)民相對總?cè)丝诘脑鲩L速度并不低,增長速度也是增加的。

  25.有三戶人家,每家有一孩子,他們的名字是:小萍(女)、小紅(女)、小虎。孩子的爸爸是老王、老張和老陳;媽媽是劉蓉、李玲和方麗。對于這三家人,已知:

 。1)老王家和李玲家的孩子都參加了少女舞蹈隊(duì)。

 。2)老張的女兒不是小紅。

 。3)老陳和方麗不是一家人。

  根據(jù)以上條件,確定以下哪項(xiàng)是正確的?

  A.老王、劉蓉和小萍是一家。

  B.老張、李玲和小紅是一家。

  C.老王、方麗和小紅是一家。

  D.老陳、方麗和小虎是一家。

  解析:邏輯推斷題。從條件1可以看出小虎不是老王和李玲的孩子,因?yàn)樾』⑹悄泻ⅰD敲纯梢酝瞥鲂∑己托〖t的爸爸可能是老王。根據(jù)條件2,老張的女兒不是小紅,所以可以確定老王的女兒是小萍。鎖定選項(xiàng)A,如果A正確,那么根據(jù)條件3老陳和李玲是一家,根據(jù)條件1,女兒是小紅;得出老陳、方麗和小虎是一家。那么選項(xiàng)D也正確。因此同時(shí)排除A、D。選項(xiàng)B與條件2不符,因此選擇C。

  26.5名犯罪嫌疑人被警察詢問,事后得知其中一名是罪犯。下面是五個(gè)嫌疑人的供述,其中只有3句真話。問,誰是罪犯?

  甲:丁是罪犯。

  乙:我是無辜的。

  丙:戊不是罪犯。

  丁:甲在撒謊。

  戊:乙說的是實(shí)話。

  A.戊    B.甲和丙    C.丁    D. 乙和戊

  解析:從選項(xiàng)入手,選項(xiàng)B,甲和丙是罪犯,可以推出乙、戊、丁、丙都為真,而題目表明只有3項(xiàng)為真,所以放棄B;選項(xiàng)C,丁是罪犯,可以推出甲、乙、丙、戊為真,同樣與題目要求不符;選項(xiàng)D,乙和戊是罪犯,只能推出丁為真。因此這道題選擇A。

  27.學(xué)生上完體育課后回到教室,有15人喝了飲水機(jī)里的純凈水,其中5人很快產(chǎn)生了腹瀉。飲水機(jī)里的純凈水馬上被送去檢驗(yàn),檢驗(yàn)的結(jié)果不能肯定其中有造成腹瀉的有害物質(zhì)。因此,喝了飲水機(jī)里的純凈水不是造成腹瀉的原因。

  如果上述檢驗(yàn)結(jié)果是正確的,則以下哪項(xiàng)對上述論證的評價(jià)最為恰當(dāng)?

  A.題干的論證有漏洞,因?yàn)樗鼪]有考慮到另一個(gè)事實(shí):哪些沒有喝了飲水機(jī)里的純凈水的人沒有造成腹瀉。

  B.題干的論證有漏洞,因?yàn)樗讶鄙僮C據(jù)證明某種情況存在,當(dāng)作有充分證據(jù)證明某種情況不存在。

  C.題干的論證有漏洞,因?yàn)樗鼪]有利用一個(gè)有力的論據(jù):為什么有更多人喝了飲水機(jī)里的純凈水沒有造成腹瀉。

  D.題干的論證有漏洞,因?yàn)樗鼪]有指出造成腹瀉的真正原因。

  解析:理解這道題非常簡單,舉個(gè)例子說明,如果一個(gè)人確實(shí)殺了人,但是法官沒有證據(jù)表明他是兇手,那么可能無法定罪,但終不能得出他不是兇手的結(jié)論,因?yàn)橥瑯記]有證據(jù)表明他不是兇手。題目中給出了同樣的問題,檢驗(yàn)結(jié)果不能肯定有造成腹瀉的有害物質(zhì),但是并不能說明純凈水不是造成腹瀉的原因。

  28.一項(xiàng)全球范圍的調(diào)查顯示,近 10年來:吸煙者的總數(shù)基本保持不變;每年只有10%的吸煙者改變自己的品牌,即放棄原有的品牌而改吸其他品牌;煙草制造商用在廣告上的支出占其毛收入的10%。在Z煙草公司的年終董事會上,懂事A認(rèn)為,上述統(tǒng)計(jì)表明,煙草業(yè)在廣告上的收益正好等于其支出,因此,此類廣告完全可以不做。以下哪項(xiàng),構(gòu)成對懂事A的結(jié)論的最有力質(zhì)疑?

  A.懂事A的結(jié)論忽視了:今年來各種品牌的香煙的價(jià)格都有了很大的變動。

  B.懂事A的結(jié)論基于一個(gè)錯(cuò)誤的假設(shè):每個(gè)吸煙者在某個(gè)時(shí)候只喜歡一種品牌。

  C.懂事A的結(jié)論基于一個(gè)錯(cuò)誤的假設(shè):每個(gè)煙草制造商只生產(chǎn)一種品牌。

  D.懂事A的結(jié)論忽視了:世界煙草業(yè)是一個(gè)由處于競爭狀態(tài)的眾多經(jīng)濟(jì)實(shí)體組成的。

  解析:董事A的判斷為廣告的支出正好與收益相等,因此得出廣告可以不做的結(jié)論。但是忽略了如果不做廣告是否能夠獲得與做廣告時(shí)除去廣告費(fèi)用后相同的收益,因?yàn)檫@部分收入也是由廣告產(chǎn)生的效益。因?yàn)橥袠I(yè)的其他競爭者會因?yàn)閺V告,增加10%的顧客,而不做廣告的煙草公司將有減少10%顧客的風(fēng)險(xiǎn)。因此選擇D

  29.在收款局的5個(gè)賬單收款員中,楊先生收款的不成功率最高。然而楊先生是這個(gè)局的職員中最好的賬單收款員。

  下面哪一項(xiàng),如果正確,最有助于解決上述短文中的明顯分歧?

  A.在加入收款局之前,楊先生是一個(gè)大百貨公司的信貸部的一名職員。

  B.收款局中的其他四個(gè)收款員都認(rèn)為楊先生是一個(gè)非常能干的帳單收款員。

  C.楊先生在過去的幾年內(nèi),每年收款成功的比率都保持相當(dāng)?shù)胤(wěn)定。

  D.這個(gè)收款局的大多數(shù)最困難的事情都是派楊先生去做的。

  解析:從題干入手分析,楊先生是最好的賬單收款員,但是收款不成功率最高。題目要求找到解釋這一矛盾的原因。顯然D最合適。

  30.在一次工程碩士入學(xué)考試中,據(jù)統(tǒng)計(jì),在英語、數(shù)學(xué)、語文和邏輯四個(gè)科目中:70%的考生英語及格了,75%的考生數(shù)學(xué)及格了,80%的考生語文及格了,85%的考生邏輯及格了。如果以上情況屬實(shí),那么,以下哪個(gè)選項(xiàng)一定正確:

  A.在此次考試中,大部分考生四門課都及格了。

  B.在此次考試中,大部分考生四門課沒有都及格。

  C.在此次考試中,有不少于10%的考生,四門課程都及格了。

  D.在此次考試中,至少有10%的考生,四門課程都沒及格。

  解析:不少于=大于等于,至少=大于等于。選線排除A、B,選項(xiàng)過于籠統(tǒng),并且與C、D存在重復(fù)關(guān)系。選項(xiàng)D與題干明顯不符合,選擇C。

 31.某游戲的游戲規(guī)則有這樣一條規(guī)定:任何在游戲中拒絕成為選手的人將在拒絕的時(shí)候被減掉10分。

  下列哪一個(gè)是此條規(guī)則所暗含的?

  A.同意參加游戲的所有人的得分將高于那些因?yàn)榇艘?guī)則而被判罰分的人。

  B.一個(gè)人在拒絕玩這個(gè)游戲的同時(shí),依然是這個(gè)游戲的參與者。

  C.最初同意成為游戲參加者然后在游戲進(jìn)行中退出的人可以避免被罰10分。

  D.一個(gè)拒絕玩這個(gè)游戲的人不能在游戲中被宣布為失敗者。

  解析:由于拒絕玩這個(gè)游戲,會被剪掉20分,說明依舊是游戲的參與者,選B。

  32.受到廣泛贊揚(yáng)的斯坦尼斯拉夫斯基演員訓(xùn)練法來自他做年輕演員時(shí)對戲劇不熟練和易受影響的老套做法。對該方法的理解必須基于斯坦尼斯拉夫斯基個(gè)人的演藝研究,從而從典型化的姿勢、反復(fù)嘗試的嗓音語調(diào)和標(biāo)準(zhǔn)的感情模式的誘惑中解脫出來。盡管他的美國門徒有過要求,但是這位俄國導(dǎo)演從未打算寫一本對表演問題提出嚴(yán)格解決方法的教科書。

  關(guān)于表演這個(gè)問題,上文作者對斯坦尼斯拉夫斯基的演員訓(xùn)練法持有怎樣的觀點(diǎn)?

  A.斯坦尼斯拉夫斯基的美國門徒,在使用他的方法訓(xùn)練時(shí),使得這種方法喪失了原有的靈活性和探索性。

  B.表演本質(zhì)上是自發(fā)的感情表達(dá),系統(tǒng)的訓(xùn)練經(jīng)常干涉它。

  C.實(shí)際上唯一需要給予年輕演員的建議是他們必須系統(tǒng)地抵制他們在戲劇表演中所表現(xiàn)出來的那些老套的模式。

  D.斯坦尼斯拉夫斯基的方法主要是針對那些必須克服有做作和不成熟表演行為的年輕演員。

  解析:斯坦尼斯拉夫斯基之所以未打算對表演問題提出嚴(yán)格解決方法的教科書,本意是不想使該方法失去靈活性和探索性。因此選擇A。

  33.已發(fā)現(xiàn)有國產(chǎn)嬰幼兒奶粉三聚氰胺含量超標(biāo)。

  如果上述斷定是真的,那么在下述三個(gè)斷定中不能確定真假的是:

  Ⅰ、國產(chǎn)嬰幼兒奶粉沒有三聚氰胺含量不超標(biāo)的。

 、、有國產(chǎn)嬰幼兒奶粉三聚氰胺含量沒超標(biāo)。

 、、所有國產(chǎn)嬰幼兒奶粉三聚氰胺含量都未超標(biāo)。

  A.只有Ⅰ和Ⅱ。    B.Ⅰ、Ⅱ和Ⅲ。    C.只有Ⅰ和Ⅲ。    D.只有Ⅱ。

  解析:選項(xiàng)1不能確定真假,從部分不能推出整體;選項(xiàng)2不能確定真假,從一部分不能退出另一部分具有相反的結(jié)論。條件3明顯可以判定為假,選擇A。

  34.股民購買股票的目的就是盈利。股民購買股票后,股票價(jià)格上漲,就會盈利,否則,就會虧損。一般情況下,盈利或者虧損達(dá)到10%就應(yīng)該拋出,以避免那些不可預(yù)測的突然變化所引起的損失。在股市上,有經(jīng)驗(yàn)的股民往往會選擇跟莊戰(zhàn)術(shù)來買賣股票。即大股東買進(jìn)的時(shí)候,他們也買進(jìn),大股東拋出的時(shí)候,他們就拋出。因?yàn)榇蠊蓶|有可能掌握更多上市公司和市場行情方面的信息。這些天,天虹公司的股票一直在下跌,但是大股東們卻一直在大量購進(jìn)這只股票。

  以上事實(shí)最能支持以下哪種預(yù)測?

  A.天虹公司的股票價(jià)格會繼續(xù)下降,但速度會放慢。

  B.天虹公司的股票價(jià)格會在短期內(nèi)大幅度上漲。

  C.股民盲目跟莊不是一個(gè)好的盈利方法。

  D.最近大股東對天虹公司股價(jià)變動判斷失誤,有可能造成巨大經(jīng)濟(jì)損失。

  解析:題目結(jié)論如下:大股東買進(jìn)的股票會漲,因?yàn)樗麄冋莆樟烁嗌鲜泄竞褪袌鲂星榉矫娴男畔ⅰK赃x擇B。

  35.作為身體的精密平衡系統(tǒng)的一部分,人類心臟分泌某種激素,這些激素能夠控制血液中的含鹽量和體內(nèi)參與循環(huán)的血量。人體僅需要極少的這種激素。它在控制血壓時(shí)極其重要,并且在患心臟病的病人血中經(jīng)常檢出。

  基于上面的研究成果,下面的哪個(gè)論點(diǎn)一定正確?

  A.假如心臟分泌的這種激素缺乏,將導(dǎo)致低血壓。

  B.一個(gè)僅僅由機(jī)械閥制作的設(shè)備被用作人工心臟,它將不能執(zhí)行人類心臟的所有功能。

  C.少量這種激素的分泌對人體的影響是長期性的。

  D.任何控制血壓的藥將通過影響心臟此種激素分泌量達(dá)到效果。

  解析:選擇B,因?yàn)橛蓹C(jī)械閥制作的人工心臟,顯然不具有分泌激素的功能,因此就不能執(zhí)行人類心臟的所有功能,題干反復(fù)說明就是為了得出這樣的結(jié)論。

  36.某出版社近年來出版物的錯(cuò)字率較前幾年有明顯的增加,引起了讀者的不滿和有關(guān)部門的批評,這主要是由于該出版社大量引進(jìn)非專業(yè)編輯所致。當(dāng)然,近年來該社出版物的大量增加也是一個(gè)重要原因。

  上述議論的漏洞,也類似地出現(xiàn)在以下哪項(xiàng)中?

  I 美國航空公司近兩年來的投訴率比前幾年有明顯的下降。這主要是由于該航空公司在裁員整頓的基礎(chǔ)上,有效地提高了服務(wù)質(zhì)量。當(dāng)然,“9.11”事件后航班乘客數(shù)量的銳減也是一個(gè)重要原因。

  II 統(tǒng)計(jì)數(shù)字表明:近年來我國心血管病的死亡率,即由心血管病導(dǎo)致的死亡率在整個(gè)死亡人數(shù)中的比例,較前有明顯增加,這主要是由于隨著經(jīng)濟(jì)的發(fā)展,我國民眾的飲食結(jié)構(gòu)和生活方式發(fā)生了容易誘發(fā)心血管病的不良變化。當(dāng)然,由于心血管病主要是老年病,因此,我國人口的老齡化,即人口中老年人比例的增加也是一個(gè)重要原因。

  III 某市去年的高考錄取率比前年增加15%,這主要是由于各中學(xué)狠抓了教育質(zhì)量。當(dāng)然,另一個(gè)重要原因是,該市今年參加高考的人數(shù)比去年增加了20%。

  A.僅僅I和II。   B.僅僅II和III。   C.僅僅I和III。   D.僅僅I。

  解析:題目中的邏輯錯(cuò)誤是強(qiáng)加因果,出版物數(shù)量的增加與錯(cuò)字率的增加沒有因果關(guān)系。根據(jù)這一思路對選項(xiàng)進(jìn)行判斷,選項(xiàng)I航班數(shù)銳減與投訴率的下降沒有因果關(guān)系,與題目中犯了同樣的錯(cuò)誤;選項(xiàng)II,心血管病是老年病,我國人口老齡化,因此我國心血管死亡率增加,這是一個(gè)有效地推理過程;選項(xiàng)III,參加高考的人數(shù)增加,并不能推出錄取比例增加的這一結(jié)論。也是屬于沒有因果關(guān)系。另外從絕對和相對概念這個(gè)角度分析,同樣可以得到相似的結(jié)論。選C

  37.從前,一個(gè)孤島上有一個(gè)奇怪的風(fēng)俗:凡是漂流到這個(gè)島上的外鄉(xiāng)人都要作為祭品被殺掉,但允許被殺的人在臨死前說一句話,然后由這個(gè)島上的長老判定這句話是真的還是假的。如果說的是真話,則將這個(gè)外鄉(xiāng)人在真理之神面前殺掉;如果說的是假話,則將他在錯(cuò)誤之神面前殺掉。有一天,一位哲學(xué)家漂流到了這個(gè)島上,他說了一句話,使得島上的人沒有辦法殺掉他。

  該哲學(xué)家必定說了下面哪一句話?

  A.你們這樣做不合乎理性。    B.我將死在真理之神面前。

  C.無論如何我都會死。      D.我將死在錯(cuò)誤之神面前。

  解析:A選項(xiàng),可以判斷為真,會在真理之神面前殺掉;B選項(xiàng),可以判定為真,因?yàn)樗麜涝谡胬碇衩媲;選項(xiàng)C,同樣可以判定為真,選項(xiàng)D我將死在錯(cuò)誤之神面前,如果為真,那么說明他說的是假話,表明他將死在真理之神面前為真,這本身與這句話矛盾了,所以這句話自背反命題,選D。

  38.有的地質(zhì)學(xué)家認(rèn)為,如果地球的未勘探地區(qū)中單位面積的平均石油儲藏量能和已勘探地區(qū)一樣的話,那么,目前關(guān)于地下未開采的能源含量的正確估計(jì)因此要乘上一萬倍,由此可得出結(jié)論,全球的石油需求,至少可以在未來五個(gè)世紀(jì)中得到滿足,即便此種需求每年呈加速上升的趨勢。為使上述論證成立,以下哪項(xiàng)是必須假設(shè)的?

  A. 地球上未勘探地區(qū)的總面積是已勘探地區(qū)的一萬倍。

  B. 在未來至少五個(gè)世紀(jì)中,世界人口的增長率不會超過對石油需求的增長率。

  C. 新技術(shù)將使未來對石油的勘探和開采比現(xiàn)在更為可行。

  D. 地球上未勘探地區(qū)中儲藏的石油可以被勘測和開采出來。

  解析:答案為D,題目中通過假設(shè)得出了未來五個(gè)世紀(jì)的石油需求相會得到滿足的結(jié)論,假設(shè)條件為地球上未勘探地區(qū)的單位面積石油儲量與已勘探地區(qū)一樣多。但僅有這個(gè)前提還是不夠的,還需要這些為勘探的時(shí)候都是可以被開采出來的這一假設(shè)。

  39.某礦山發(fā)生了一起嚴(yán)重的安全事故。關(guān)于事故原因,甲乙丙丁四位負(fù)責(zé)人有如下斷定:

  甲:如果造成事故的直接原因是設(shè)備故障,那么肯定有人違反操作規(guī)程。

  乙:確實(shí)有人違反操作規(guī)程,但造成事故的直接原因不是設(shè)備故障

  丙:造成事故的直接原因確實(shí)是設(shè)備故障,但并沒有人違反操作規(guī)程。

  。涸斐墒鹿实闹苯釉蚴窃O(shè)備故障。

  如果上述斷定只有一個(gè)人的斷定為真,那么以下斷定都不可能為真,除了:

  A.甲的斷定為真,有人違反了操作規(guī)程。

  B.甲的斷定為真,但沒有人違反操作規(guī)程。

  C.乙的斷定為真。

  D.丙的斷定為真。

  解析:這道題屬于邏輯推理題,由于只有一個(gè)人判斷為真,因此丙、丁的判斷首先排除,因?yàn)檫@兩個(gè)判定并不沖突。甲、乙的推斷有矛盾,因此兩項(xiàng)必有一個(gè)是正確的。甲的推斷結(jié)構(gòu)是如果B推出A,那么 C必然推出B;乙的推斷結(jié)構(gòu)是C推出B,但B不能退出A,是甲判定的一個(gè)逆向命題。選項(xiàng)B是正確答案,因?yàn)檫x項(xiàng)B肯定了甲的推斷,同時(shí)否定了乙的發(fā)生,因此是真命題。其他選項(xiàng)均不正確。

  40.心臟的搏動引起血液循環(huán)。對同一個(gè)人,心率越快,單位時(shí)間進(jìn)入循環(huán)的血液量越多。血液中的紅血球運(yùn)輸氧氣。一般地說,一個(gè)人單位時(shí)間通過血液循環(huán)獲得的氧氣越多,他的體能及其發(fā)揮就越佳。因此,為了提高運(yùn)動員在體育比賽中的競技水平,應(yīng)該加強(qiáng)他們在高海拔地區(qū)的訓(xùn)練,因?yàn)樵诟吆0蔚貐^(qū),人體內(nèi)每單位體積血液中含有的紅血球數(shù)量,要高于在低海拔地區(qū)。

  以下哪項(xiàng)是題干的論證必須假設(shè)的?

  A.運(yùn)動員在高海拔地區(qū)的心率不低于在低海拔地區(qū)。

  B.不同運(yùn)動員的心率基本相同。

  C.運(yùn)動員的心率比普通人慢。

  D.在高海拔地區(qū)訓(xùn)練能使運(yùn)動員的心率加快。

  解析:此題論證說明運(yùn)動員在高海拔地區(qū)訓(xùn)練體內(nèi)的紅血球數(shù)量高于低海拔地區(qū),從而達(dá)到最佳體能的目的。B和C與題意不符,選A。

 41~45題基于以下題干:

  七個(gè)學(xué)生R、S、T、V、W、X、Y,被分成兩個(gè)學(xué)習(xí)小組。第一組有三名成員,第二組有四名成員。學(xué)生們的分組必須符合以下要求:

  R和T不能在同一個(gè)小組;

  如果S在第一組,那么V必須在第一組;

  如果W在第一組,那么T必須在第二組;

  X必須在第二組。

  41.如果W在第一組,那么以下哪項(xiàng)也一定在第一組?

  A.R    B.S    C.T    D.V

  解析:根據(jù)條件第一組有三名成員。因?yàn)镽和T不能在一組,又因?yàn)闂l件:如果W在第一組,那么T必須在第二組,則R可以在第一組

  42.如果T和Y都在第一組,那么以下哪項(xiàng)一定是真的?

  A.S和V在同一組。   B.S和W在同一組。   C.V和R在同一組。   D.W和T在同一組。

  解析:如果T和Y在第一組,R和T又不能在一組,可知R只能在第二組,而X必須在第二組。如果S在第一組,根據(jù)條件,V也需在第一組,而第一組只有三個(gè)成員,那么S和V就不能在同一組。而如果W在第一組,那么T必須在第二組。但是T已經(jīng)在第一組,則W必須在第二組,加之第二組有四人,那么S也需在第二組,那么S和W為一組。

  43.如果W和T在同一組,那么以下哪項(xiàng)可能在同一組,除了:

  A.R和S    B.S和Y    C.T和Y    D.V和Y

  答案:B

  解析:

  條件分析:

  1、R1且T2;或者 R2且T1

  2、S1→V1;即S2的時(shí)候,V可以自由分配。

  3、W1→T2;即W2的時(shí)候,T可以自由分配。

  4、X2

  43題的臨時(shí)題干要求WT在同一組,根據(jù)條件3,他們不可能在第一組,不然就違背了題設(shè)。所以,這個(gè)時(shí)候,WT都在都在第二組。第二組已經(jīng)有了WTX三個(gè)成員,只有一個(gè)空位了。根據(jù)條件1,可以知道,R在第一組。因此,除了跟WTX搭伴的元素外,任何二人組合都必須在第一組了。

  選項(xiàng)A,第一組RSV;第二組WTXY,有可能。

  選項(xiàng)B,第一組RSY;則第一組還必須有V,顯然超編了。因?yàn)榈谝唤M只能有三個(gè)人。因此,此項(xiàng)為答案。

  選項(xiàng)C,依然是第一組RSV;第二組WTXY這樣的組合。

  選項(xiàng)D,第一組RVY;第二組WTXS,不矛盾。

  44.如果V和Y在同一組,那么以下哪項(xiàng)一定是真的?

  A.R在第一組。    B.S在第一組。    C.W在第二組。    D.Y在第二組。

  解析:如果V和Y在第一組,那么根據(jù)條件如果S在第一組,那么V必須在第一組;說明S可以在第一組,又因?yàn)槿绻鸚在第一組,那么T必須在第二組,但是R和T又不能在一組,則R必須在第一組,W和T可以在第二組。則答案C正確。

  45.如果S在第一組,那么以下哪項(xiàng)一定是真的?

  A.T在第一組。    B.T在第二組。    C.Y在第一組。    D.Y在第二組。

  解析:如果S在第一組,根據(jù)條件V必須在第一組,如果W在第一組,那么T必須在第二組;R和T不能在同一個(gè)小組;那么W或R可以在第一組,X必須在第二組。因此在第一組的可以是S或V或R或W,因此答案D正確。

  46~50題基于以下題干:

  六位教授F、G、H、J、K、L,將評審馬、任、孫、吳博士的論文4篇。評審需遵守以下原則:

  (1)每位教授只評審一篇博士論文;

  (2)每篇博士論文至少有一位教授評審;

 。3)H與F評審?fù)黄┦空撐模?/p>

 。4)L只與其他教授中的一位同評一篇博士論文;

 。5)G評審馬博士的論文;

  (6)J評審馬博士或吳博士的論文;

 。7)H不評審吳博士的論文。

  46.如果K不評審孫博士的論文,那么以下哪項(xiàng)一定是真的?

  A.L評審馬博士的論文           B.L評審孫博士的論文

  C.F和H評審任博士的論文        D.F和H評審孫博士的論文

  解析:H不評審吳博士論文,H與F又評審?fù)黄撐模荒敲碒和F有可能評審馬或任或?qū)O博士的論文,但是G評審馬博士的論文,J評審馬博士或者吳博士的論文,那么H與F就有可能評審任或?qū)O博士的論文,K不評審孫博士的論文,K就有可能評審任博士的論文,那么H和F就會評審孫博士的論文。

  47.以下哪項(xiàng)完整地列出了L可能評審的博士論文?

  A.馬、任      B.馬、孫、吳     C.馬、任、吳       D.馬、任、孫、吳

  解析:根據(jù)條件四,L只與其他教授中的一位同評一篇博士論文,而每篇博士論文至少有一位教授評審,也就是說一篇論文有可能是兩個(gè)教授或者三個(gè)教授一起評論。因此,L可以評論馬、任、孫、吳的博士論文。

  48.以下哪項(xiàng)可能是真的?

  A.F和G評審馬博士的論文。

  B. F和L評審吳博士的論文。

  C. K評審吳博士的論文并且L評審馬博士的論文。

  D. L評審任博士的論文并且F評審孫博士的論文。

  解析:G評審馬博士的論文,J評審馬博士或吳博士的論文,H不評審吳博士的論文,H與F又評審?fù)黄撐;那么H和F有可能評審馬或任或?qū)O博士的論文,那么H與F就有可能評審任或?qū)O博士的論文。加之題一,K不評審孫博士的論文,K就有可能評審任博士的論文,條件三H與F評審?fù)黄┦空撐,那么H和F就會評審孫博士的論文。條件四:L只與其他教授中的一位同評一篇博士論文,L可以與K評審任博士論文,故D正確

  49.以下哪項(xiàng)不可能是真的?

  A. L和G評審馬博士的論文        B. L和K評審馬博士的論文

  C. L和K評審任博士的論文        D. L和K評審孫博士的論文

  解析:根據(jù)條件(4)L只與其他教授中的一位同評一篇博士論文,而條件(5)G評審馬博士的論文,那么可知L和G可評審馬博士論文,故A正確。根據(jù)條件(6)J評審馬博士或吳博士的論文,可以得知L可以和J可以評審吳博士或者馬博士的論文,H不評審吳博士的論文,那么H可以評論任博士和孫博士的論文。K可以評論吳博士、任博士、孫博士的論文。加之條件(4),得知答案C和D正確,故B錯(cuò)誤

  50.如下哪位教授不可能審閱任博士的論文?

  A.F   B.K    C.J   D. L

  解析:條件(6)J評審馬博士或吳博士的論文;而條件(1)每位教授只評審一篇博士論文,根據(jù)兩個(gè)條件可以推斷J不可能審閱任或?qū)O博士的論文,故選C

  模擬題1參考答案:

01. C 02. A 03. C 04. D 05. C
06. C 07. A 08. B 09. A 10. A
11. A 12. B 13. B 14. A 15. C
16. D 17. D 18. C 19. A 20. B
21. C 22. C 23. B 24. B 25. C
26. A 27. B 28. D 29. D 30. C
31. B 32. A 33. A 34. B 35. B
36. C 37. D 38. D 39. B 40. A
41. A 42. B 43. B 44. C 45. D
46. D 47. D 48. D 49. B 50. C
 

 

 

 

(50題,每題2分,滿分100分)

  1.任何人都沒有吃過雅各島上的任何水果,所以無法知道雅各島上任何水果的口味。

  為了合乎邏輯的推出上述結(jié)論,需要假設(shè)下面哪項(xiàng)為前提?

  A.如果一種水果有人品嘗過,就可以知道其口味。(充分條件,不是前提)

  B.只憑某些人的品嘗無法真正知道某種水果的口味。

  C.要知道某種水果的口味,需要有人去品嘗。

  D.人們是通過嗅覺來確定水果口味的。

  解析:這道題需要補(bǔ)全假設(shè),結(jié)論是:任何人都無法知道任何水果的味道,前提是:任何人都沒有吃過雅各島上的任何水果。補(bǔ)充前提是:要知道水果的口味,需要有人去品嘗,應(yīng)該選擇C。選項(xiàng)A,是充分條件,但不是構(gòu)成結(jié)論的原因,屬于強(qiáng)拉因果;B、D都為不合理的假設(shè)。

  2.最近的一項(xiàng)研究指出:“經(jīng)常吃沙棘果對兒童的智力發(fā)育有益。”研究人員對560名兒童進(jìn)行調(diào)查,發(fā)現(xiàn)那些經(jīng)常吃沙棘果的兒童,其智力水平較很少吃沙棘果的兒童要高。因此,研究人員發(fā)現(xiàn)了沙棘果與兒童智力發(fā)育之間的聯(lián)系。

  以下哪項(xiàng)如果為真,最不可能削弱上述論證?

  A.對成年人的研究發(fā)現(xiàn),每天吃沙棘果的人智力水平并不比很少吃沙棘果人的高。

  B.調(diào)查顯示:沙棘果價(jià)格非常高,只有富裕家庭的兒童才經(jīng)常吃,同時(shí)這些家庭有條件實(shí)現(xiàn)兒童的早期智力開發(fā)。

  C.這項(xiàng)兒童發(fā)育研究的課題負(fù)責(zé)人是沙棘果生產(chǎn)商,其目的就是要擴(kuò)展沙棘果的銷售渠道。

  D.沙棘果是兒童喜歡的食品,家長經(jīng)常把沙棘果作為禮物獎(jiǎng)給智力表現(xiàn)優(yōu)異的孩子。

  解析:吃沙棘果與兒童智力發(fā)展是否存在必然聯(lián)系。通過樣本調(diào)查判斷出吃沙棘果與兒童智力發(fā)展存在正相關(guān)的關(guān)系。B選項(xiàng)說明家庭條件是否富裕與兒童智力發(fā)展存在關(guān)聯(lián)聯(lián)系;C選項(xiàng)說明題目中的結(jié)論是由于生產(chǎn)商的廣告產(chǎn)生的以偏概全的結(jié)論;D選項(xiàng)屬于因果倒置,成績優(yōu)異是因,吃沙棘果是結(jié)果。

  3.凡金屬都是導(dǎo)電的。銅是導(dǎo)電的,所以銅是金屬。

  下面哪項(xiàng)與上述推理結(jié)構(gòu)最相似?

  A.所有的鳥都是卵生動物,蝙蝠不是卵生動物,所以,蝙蝠不是鳥。

  B.所有的鳥都是卵生動物,天鵝是鳥,所以天鵝是卵生動物。

  C.所有從事工商管理的都要學(xué)習(xí)企業(yè)管理,老陳是學(xué)習(xí)企業(yè)管理的,所以,老陳是從事工商管理工作的。

  D.華山險(xiǎn)于黃山,黃山險(xiǎn)于泰山,所以華山險(xiǎn)于泰山。

  解析:題目結(jié)構(gòu)是所有的A具有某中屬性,B屬于A,因此B具有A的屬性,選項(xiàng)C與上述結(jié)論類似。

  4.為了有助于人們選擇最滿意的城市居住,有關(guān)部門實(shí)施了一項(xiàng)評選“最舒適城市”的活動。方法是,選擇十個(gè)方面,包括社會治安、商業(yè)設(shè)施、清潔程度、綠化程度、教育設(shè)施、旅游文化景點(diǎn)等等,每個(gè)方面按實(shí)際質(zhì)量的高低,評以1分至10分之間的某一分值,然后求得十個(gè)分值的平均數(shù)即是這個(gè)城市的舒適性指數(shù)。

  以下哪項(xiàng)是實(shí)施上述活動需要預(yù)設(shè)的前提?

 、、城市的各種舒適性質(zhì)量程度都可以用準(zhǔn)確的數(shù)字表達(dá)。

 、颉⒊鞘械母鞣N舒適性對于居民來說都是同等重要的。

 、、居民有自由選擇居住城市的權(quán)利并且大都樂于這樣做。

  A.僅Ⅰ。      B.僅Ⅲ。      C.僅Ⅰ和Ⅱ。      D.Ⅰ、Ⅱ和Ⅲ。

  解析:這道題是補(bǔ)充假設(shè)的問題,由于每個(gè)人對“舒適程度”的感覺是不一樣的,因此題干結(jié)論若要正確,條件Ⅱ是必須的前提;若題目中結(jié)論正確,需要假設(shè)題目中的十個(gè)指標(biāo)存在量化標(biāo)準(zhǔn),因此條件Ⅰ為必選項(xiàng);條件Ⅲ存在爭議,不過可以從另外一個(gè)角度理解,便于理解題目的結(jié)論,如果居民沒有選擇居住地的權(quán)利,那么題目中的滿意度調(diào)查就沒有意義,所以也得不到上述結(jié)論。

  5.衛(wèi)生部的官員們對牟定縣狂犬病疫情有以下斷定:

  (1)該縣所有的狗都得了狂犬病。

  (2)該縣有些斑點(diǎn)狗得了狂犬病。(真)

  (3)該縣有些狗得了狂犬病。

  (4)該縣有些狗沒得狂犬病。(真)

  其實(shí)上述斷定中只有兩個(gè)與事實(shí)相符。根據(jù)如上的情況,以下哪項(xiàng)結(jié)論是可能成立的?

  I.該縣的狗都是斑點(diǎn)狗。

  II. 該縣沒有斑點(diǎn)狗可能得狂犬病。

  III. 該縣的斑點(diǎn)狗并非可能沒有得狂犬病。

  IV. 該縣的狗不可能都沒得狂犬病。

  A.僅I      B.僅III      C.僅II和IV      D.僅IV。

  解析:這道題屬于邏輯推理中真假話題的問題。解析這種問題按照1、先找矛盾;2、繞開矛盾;3、推出答案的步驟進(jìn)行。首先先找矛盾,首先1和2是一對矛盾,3和4是一對矛盾,只有一個(gè)正確,2、繞開矛盾,如果1正確,那么3和4都假,因此只能2為真,如果2為真,只能判斷4為真,3為假,3、推出答案,條件2有些斑點(diǎn)夠得了狂犬病,可推出該縣沒有斑點(diǎn)狗可能得狂犬病,大家注意必然=沒有……必然;同時(shí)根據(jù)條件4可以得出該縣的狗不可能都沒得狂犬病。

  6.如果一個(gè)兒童的體重與身高的比值超過本地區(qū)80%兒童的水平,就稱其為肥胖兒。根據(jù)歷年的調(diào)查結(jié)果,15年來,臨江市的肥胖兒的數(shù)量一直在穩(wěn)定增長。

  如果以上斷定為真,則以下哪項(xiàng)也必為真?

  A.臨江市每一個(gè)肥胖兒的體重都超過全市兒童的平均體重。

  B.15年來,臨江市的兒童的體育鍛煉越來越不足。

  C.臨江市的非肥胖兒的數(shù)量15年來不斷增長。

  D.15年來,臨江市體重不足標(biāo)準(zhǔn)體重的兒童數(shù)量不斷下降。

  解析:這道題屬于迷惑推理題,大家務(wù)必要小心。首先大家可以看出臨江市兒童數(shù)量是在增長的;兒童由肥胖和非肥胖組成;肥胖兒童的數(shù)量穩(wěn)定增長,說明非肥胖兒童的數(shù)量也在穩(wěn)定增長;沒有證據(jù)表明非肥胖兒童的數(shù)量在不斷下降。

  7.孩子出生后的第一年在托兒所里度過,會引發(fā)孩子的緊張不安。在我們的研究中,有464名12~13歲的兒童接受了特異情景測試法的測驗(yàn),該項(xiàng)測驗(yàn)意在測試兒童1歲時(shí)的狀況與對母親的依附心理之間的關(guān)系。其結(jié)果:有41.5%曾在托兒所看護(hù)的兒童和25.7%曾在家看護(hù)的兒童被認(rèn)為緊張不安,過于依附母親。

  以下哪項(xiàng)如果為真,最沒有可能對上述研究的推斷提出質(zhì)疑?

  A.出生后第一年在家看護(hù)的孩子多數(shù)是由祖父母或外祖父母看護(hù)的,并形成濃厚的親情。

  B.這項(xiàng)研究的主持者被證實(shí)曾經(jīng)在自己的幼兒時(shí)期受到過長時(shí)間來自托兒所阿姨的冷落。

  C.針對孩子母親的另一部分研究發(fā)現(xiàn):由于孩子在家里表現(xiàn)出過度的依附心理,父母因此希望將其送入托兒所予以矯正。

  D.研究中所測試的孩子并不是從托兒所看護(hù)和在家看護(hù)兩種情況下隨機(jī)抽取的。因此,這兩組樣本兒童的家庭很可能有系統(tǒng)性的差異存在。

  解析:這種題最好的解決辦法是否定推斷,把你認(rèn)為可能的答案否定,帶入題目中判斷。如果能夠提出質(zhì)疑,說明你的判斷是正確的。A選項(xiàng)。

  8.在玉米地中套種的小麥有可能得小麥黃葉病,這種病是由于光照不足造成的,一旦光照充足,比如減少玉米植株數(shù)量,或者玉米收割后,癥狀很快就會消失,且不會影響產(chǎn)量。而小麥銹病是由花斑銹菌引起,如果在病癥初起時(shí)不及時(shí)恰當(dāng)應(yīng)對,就會使小麥植株很快枯死。由于小麥銹病和小麥黃葉病初期癥狀很難區(qū)分,所以,在玉米地中套種小麥,必須高度注意小麥銹病的定期篩查和預(yù)防。

  以下哪項(xiàng)最可能是上述論證所假設(shè)的?

  麥黃葉病不會誘發(fā)小麥銹病。

  B.花斑銹菌不會因?yàn)槌渥愎庹斩π←湹那趾αΑ?/p>

  C.小麥銹病如果及時(shí)恰當(dāng)處理,就不會使小麥減產(chǎn)。

  D.科學(xué)家一直沒有找到有效遏制小麥銹病的藥物。

  解析:假設(shè)推斷。陽光充足,小麥黃葉病會消失,產(chǎn)量不影響;花斑銹菌,使小麥植株枯死。因此需要假設(shè)陽光充足不會抑制花斑銹菌的發(fā)病,才能得出小麥黃葉不發(fā)病時(shí),花斑銹菌仍然會發(fā)病,導(dǎo)致兩種病情難以區(qū)分。換一種角度,如果小麥黃葉病不會和花斑銹菌同時(shí)發(fā)病,那么就不存在難以區(qū)分病情的問題了。

  9.公司準(zhǔn)備從五個(gè)業(yè)務(wù)骨干中選幾個(gè)人去中央戲劇學(xué)院進(jìn)修。鑒于業(yè)務(wù)員們的工作關(guān)系,總經(jīng)理建議:

 。1)如果選鞏俐,那么必須選章子怡并且不能選周迅。

 。2)如果選章子怡或者選周迅,則不能選吳孟達(dá)。

 。3)不能既不選周星馳也不選吳孟達(dá)。

  考慮總經(jīng)理的建議,董事會認(rèn)為,鞏俐必須去中央戲劇學(xué)院進(jìn)修,這樣誰將跟她一起去進(jìn)修?

  A.章子怡和周星馳    B.章子怡和吳孟達(dá)   C.周星馳和周迅    D.吳孟達(dá)和周迅

  解析:邏輯推斷。如果鞏麗必須去,根據(jù)條件1章子儀必須去,而周迅一定不去,從而排除C、D選項(xiàng),根據(jù)條件2,如果選章子儀,那么吳孟達(dá)不能去。排除B 。

  繼續(xù)進(jìn)行推理,根據(jù)條件3必須在周星馳和吳孟達(dá)中選一個(gè),因?yàn)楦鶕?jù)條件2已經(jīng)排除吳孟達(dá),所以周星馳必須去。選擇A。

  10.20世紀(jì)60年代早期之前,挪威的斯塔溫格是一個(gè)安靜而和平的小鎮(zhèn)。進(jìn)入60年代以來,這里成為了挪威近海石油勘探中心。從此,暴力犯罪和毀壞公物的現(xiàn)象在斯塔溫格小鎮(zhèn)急劇增加。顯然這些社會問題是是石油繁榮的副產(chǎn)品。

  下列哪一個(gè)選項(xiàng)最強(qiáng)烈的支持上面的論述。

  A.暴力犯罪和毀壞公物在沒有石油繁榮的挪威城鎮(zhèn)保持低水平。

  B.斯塔溫格居民很少對這里是近海石油勘探中心感到遺憾。

  C.挪威社會學(xué)家對斯塔溫格的暴力犯罪和毀壞公物現(xiàn)象持續(xù)增加表示關(guān)切。

  D.非暴力犯罪、毒品、離婚,在這個(gè)小鎮(zhèn)也以同樣的速度增加著。

  解析:題目把社會問題的原因歸結(jié)為石油繁榮,最強(qiáng)烈支持的論據(jù)為在沒有石油繁榮的時(shí)期,社會問題處于低水平。因此選A。

11.認(rèn)為只傷害自己而沒有傷害到別人的行為并沒有什么錯(cuò)誤的想法,通常伴隨著對人與人之間相互依賴的人際關(guān)系的忽視。毀壞一個(gè)人自己的健康或者生命意味著不能你將不能為家庭或者社會提供幫助,相反意味著你將額外享有那些本來就有限的諸如食物、健康服務(wù)和教育等社會資源,而不是相反將它們回報(bào)給社會。

  以下哪個(gè)選項(xiàng)如果正確,最強(qiáng)烈的地支持題干所表達(dá)的觀點(diǎn)。

  A.本來可以避免的疾病和意外事故所造成的花銷增加了每個(gè)人的健康保險(xiǎn)金。

  B.傷害一個(gè)人能夠?qū)е麻g接的利益,諸如可以使別人獲得與健康領(lǐng)域相關(guān)聯(lián)的工作。

  C.一個(gè)人對社會所作的貢獻(xiàn)可以通過他的健康程度來衡量。

  D.由喝酒、吸煙和吸食毒品所導(dǎo)致的主要傷害是由使用那些東西的人來承受的。

  解析:題目中的結(jié)論是傷害自己的行為與傷害別人的行為同樣都是錯(cuò)誤的。原因是傷害自己將不能為家庭和社會提供幫助,同時(shí)會向社會索取的更多。這道題的關(guān)鍵是大家要讀懂題干,這樣從選項(xiàng)中我們不難選出A,本來可以避免的個(gè)人傷害回增加健康保險(xiǎn)金,從而增加社會的負(fù)擔(dān)。

  12.大多數(shù)道路的修理比預(yù)算的要花費(fèi)更多的時(shí)間和金錢,但是去年夜間修理京津塘高速公路和類似的道路并未比預(yù)算花費(fèi)更多的時(shí)間或金錢。因此,在夏季,夜間修理主要道路可能省時(shí)省錢。

  下列哪一個(gè),假如正確,最支持上面得到的結(jié)論?

  A.修理京津塘高速公路的預(yù)算足夠用,所以不可能超過預(yù)算。

  B.夏季,夜間路上通行車輛較少,而且溫度較舒適,允許修路工人工作得更快。

  C.愿意在晚上工作的修路工人較容易找到工作機(jī)會,因?yàn)榇蠖鄶?shù)人寧愿白天工作。

  D.用于道路修理的瀝青在較高溫度下膨脹,在溫度降低的時(shí)候收縮。

  解析:補(bǔ)充論據(jù)。從題干可以看出從特殊樣本推出結(jié)論明顯論據(jù)不足。夏天夜間施工本來和省時(shí)省錢本來沒有關(guān)系,屬于不相關(guān)關(guān)系,因此需要補(bǔ)充適當(dāng)?shù)恼摀?jù),在這兩個(gè)問題中間搭起橋梁,建立關(guān)系,所以選擇B。

  13.偏頭痛一直被認(rèn)為是由食物過敏引起的。但是,如果讓患者停止食用那些已經(jīng)證明會不斷引起過敏性偏頭痛的食物,他們的偏頭痛并沒有停止,因此,顯然存在別的某種原因引起偏頭痛。下列哪項(xiàng)如果為真,最能削弱上面的結(jié)論?

  A.許多患者說誘發(fā)偏頭痛病的那些食物往往是他們最喜歡吃的食物。

  B.許多普通食物只在食用幾天后才誘發(fā)偏頭痛,因此,不容易觀察患者的過敏反應(yīng)和他們食用的食物之間的關(guān)系。

  C.很少有食物過敏會引起像偏頭痛那樣嚴(yán)重的癥狀。

  D.許多不患偏頭痛的人同樣有食物過敏反應(yīng)。

  解析:題目結(jié)論是過敏引發(fā)偏頭痛,但是引發(fā)過敏的問題不僅僅是食物引起的。結(jié)論指明食物過敏與偏頭痛之間不存在這因果關(guān)系,原因是停用了這種食物之后,偏頭痛的病情仍然存在,若要削弱這個(gè)結(jié)論,需要針對上述論據(jù)提出反對。本著這個(gè)思路我們看選項(xiàng),結(jié)論B。表明了停用食物之后仍然存在偏頭痛的原因是因?yàn)樵S多普通食物在食用幾天后才發(fā)作,因此雖然停用了食物,仍會發(fā)作一段時(shí)間。

  14.世界衛(wèi)生組織在全球范圍內(nèi)進(jìn)行了一項(xiàng)有關(guān)獻(xiàn)血對健康的影響的跟蹤調(diào)查。調(diào)查對象分為三組。第一組對象中均有二次以上的獻(xiàn)血記錄,其中最多的達(dá)數(shù)十次;第二組中的對象均僅有一次獻(xiàn)血記錄;第三組對象均從未獻(xiàn)過血。調(diào)查結(jié)果顯示,被調(diào)查對象中癌癥和心臟病的發(fā)病率,第一組分別為0.3%和0.5%,第二組分別為0.7%和0.9%,第三組分別為1.2%和2.7%。一些專家依此得出結(jié)論,獻(xiàn)血有利于減少患癌癥和心臟病的風(fēng)險(xiǎn)。這兩種病已經(jīng)不僅在發(fā)達(dá)國家而且在發(fā)展中國家成為威脅中老年人生命的主要?dú)⑹。因此,獻(xiàn)血利己利人,一舉兩得。以下哪項(xiàng)如果為真,將削弱以上結(jié)論?

  I 60歲以上的調(diào)查對象,在第一組中占60%,在第二中占70%,在第三組中占80%。

  II 獻(xiàn)血者在獻(xiàn)血前要經(jīng)過嚴(yán)格的體檢,一般具有較好的體質(zhì)。

  III 調(diào)查對象的人數(shù),第一組為1700人,第二組為3000人,第三組為7000人。

  A.只有I和II。    B.只有II和III。    C.只有I和III    D.只有I

  解析:題目中對獻(xiàn)血與癌癥、心臟病的發(fā)病幾率建立了正相關(guān)關(guān)系。但是樣本仍然存在問題,如果鮮血的人本身身體素質(zhì)就比沒有獻(xiàn)血的身體素質(zhì)好,就不能退出上述結(jié)論。這是最簡單容易想出的悖論。因此淘汰C、D。下面在條件1和3中選一個(gè),條件3說明樣本容量不同,但是題目中使用的是百分?jǐn)?shù)這個(gè)相對指標(biāo),因此不能形成反對結(jié)論的悖論。選A。

  15.許多孕婦都出現(xiàn)了維生素缺乏的癥狀,但這通常不是由于孕婦的飲食中缺乏維生素,而是由于腹內(nèi)嬰兒的生長使她們比其他人對維生素有更高的需求。為了評價(jià)上述結(jié)論的確切程度,以下哪項(xiàng)操作最為重要?

  A.對某個(gè)不缺乏維生素的孕婦的日常飲食進(jìn)行檢測,確定其中維生素的含量。

  B.對孕婦的科學(xué)食譜進(jìn)行研究,以確定有利于孕婦攝入足量維生素的最佳選擇。

  C.對日常飲食中維生素足量的一個(gè)孕婦和一個(gè)非孕婦進(jìn)行檢測,并分別確定她們是否缺乏維生素。

  D.對日常飲食中維生素不足量的一個(gè)孕婦和另一個(gè)非孕婦進(jìn)行檢測,并分別確定她們是否缺乏維生素。

  解析:支持論據(jù)。題目需要根據(jù)結(jié)論找出支持論據(jù),顯然對同量的維生素給孕婦和非孕婦服用,觀察他們是否缺乏維生素,對得出結(jié)論最重要。選C

  16.經(jīng)A省的防疫部門檢測,在該省境內(nèi)接受檢疫的長尾猴中,有1%感染上了狂犬病。但是只有與人及其寵物有接觸的長尾猴才接受檢疫。防疫部門的專家因此推測,該省長尾猴中感染有狂犬病的比例,將大大小于1%。

  以下哪項(xiàng)如果為真,將最有力地支持專家的推測?

  A.在A省境內(nèi),與人及其寵物有接觸的長尾猴,只占長尾猴總數(shù)的不到10%。

  B.在A省,感染有狂犬病的寵物,約占寵物總數(shù)的0.1%。

  C.在與A省毗鄰的B省境內(nèi),至今沒有關(guān)于長尾猴感染狂犬病的疫情報(bào)告。

  D.與健康的長尾猴相比,感染有狂犬病的長尾猴更愿意與人及其寵物接觸。

  解析:選擇D,其他選項(xiàng)均不能支持題干給出的結(jié)論。在與人和動物接觸的長尾猴進(jìn)行檢驗(yàn),發(fā)現(xiàn)1%感染了狂犬病,推斷出該省長尾猴感染狂犬病的比例小于1%,那么只有感染狂犬病的長尾猴更喜歡與人和動物接觸能夠有效地支持這個(gè)結(jié)論。

  17.作為市電視臺的攝像師,最近國內(nèi)電池市場的突然變化讓我非常頭疼。進(jìn)口電池缺貨,我只能用國產(chǎn)電池來代替作為攝像的主要電源。盡管每單位的國產(chǎn)電池要比進(jìn)口電池便宜,但我估計(jì)如果持續(xù)用國產(chǎn)電池替代進(jìn)口電池來提供同樣的電源供應(yīng)的話,我在能源上的支付將會提高。

  說這番話的人在上面這段話中隱含了以下哪項(xiàng)假設(shè)?

  A. 持續(xù)使用國產(chǎn)電池,攝像的質(zhì)量將無法得到保障。

  B. 每單位的進(jìn)口電池要比國產(chǎn)電池價(jià)格貴。

  C. 生產(chǎn)國產(chǎn)電池要比生產(chǎn)進(jìn)口電池成本低。

  D. 以每單位電池提供的電能來計(jì)算,國產(chǎn)電池要比進(jìn)口電池提供得少。

  解析:這是日常生活中經(jīng)常會遇到的問題,答案一目了然選擇D。得出結(jié)論的原因是盡管國產(chǎn)電池比進(jìn)口電池單價(jià)便宜,但是就單位電能來講,國產(chǎn)電池的價(jià)格卻比進(jìn)口電池高。

  18.最近由于氣候的異常變化,使得山東省的大蒜產(chǎn)量受到較大影響而減產(chǎn)。山東大蒜的價(jià)格比平時(shí)同期上漲了兩倍,這就大大提高了大蒜素生產(chǎn)的成本,估計(jì)未來大蒜素的市場價(jià)格將有大幅度的提高。以下哪項(xiàng)如果是真的,最能削弱上述結(jié)論?

  A.去年大蒜的價(jià)格是歷年最低的。

  B.其他替代原料可以用來生產(chǎn)人工大蒜素。

  C.除了山東省外,其他省份也可以提供大蒜。

  D.最近的天氣異常不如專家們估計(jì)的那么嚴(yán)重。

  解析:題目中的推斷是沒有問題的,成本的提高會促使價(jià)格的提高,但是其他省也提供大蒜,而成本并沒有提高,價(jià)格不會上升,在這樣的市場上,山東省的大蒜盡管成本提高了,但是價(jià)格將會保持市場水平,因此C有效地消弱了結(jié)論。

  19.某學(xué)校開設(shè)選修課,規(guī)定:同學(xué)只有選了文化史課的同學(xué)才能選古典音樂課。如果選古典音樂課,那他就不能選文學(xué)欣賞課。如果選文學(xué)欣賞課,就不能選古典音樂課。

  如果以上陳述為真,以下哪項(xiàng)陳述不可能假?

  A.同學(xué)不選文學(xué)欣賞課 或者不選古典音樂課。

  B.同學(xué)選文學(xué)欣賞課 或者不選古典音樂課。

  C.同學(xué)不選古典音樂課,或者不選文化史課。

  D.同學(xué)選古典音樂課,或者不選文學(xué)欣賞課

  解析:邏輯推斷題。不可能假=一定真,因此題目要求在選項(xiàng)中找出一定真的項(xiàng)。選了文化史,才能選古典音樂,這是個(gè)必要條件;但是選了古典音樂就不能選文學(xué)欣賞,二選其一;選項(xiàng)A的論述與題干一樣。

  20.20年前,幾乎所有的公司首腦在選擇重新設(shè)置公司總部的時(shí)候,主要關(guān)心的是土地價(jià)格。今天一個(gè)高級執(zhí)行官計(jì)劃重設(shè)總部時(shí)主要關(guān)心的東西更廣泛了,經(jīng)常包括當(dāng)?shù)氐膶W(xué)校和住房質(zhì)量。

  假如上述信息可靠,下面哪一項(xiàng)最好地解釋了公司首腦們主要關(guān)心問題的變化?

  A.20年前高質(zhì)量的住房和學(xué)校像今天一樣難以發(fā)現(xiàn)。

  B.近年來優(yōu)秀專業(yè)辦公人員缺乏的問題迫使公司找到盡可能多的方法來留住老員工,并且吸引優(yōu)秀新員工的加盟。

  C.公司執(zhí)行官總是考慮自己的決定將怎樣影響公司的利潤。

  D.在過去20年,一些地區(qū)比其他地區(qū)土地價(jià)格變化少。

  解析:選項(xiàng)A屬于強(qiáng)拉因果,不存在相關(guān)關(guān)系;選項(xiàng)B說明了高級執(zhí)行官在公司選址時(shí)關(guān)注學(xué)校和住房質(zhì)量的原因,分別是吸引優(yōu)秀新員工加盟和留住老員工;選項(xiàng)C與題干沒有關(guān)系,并未涉及到利潤這個(gè)因素;選項(xiàng)D,最多可以說明高級執(zhí)行官不在關(guān)心土地價(jià)格的原因,但是并不能說明關(guān)注學(xué)校和住房質(zhì)量的原因。

21.用蒸餾麥芽渣提取的酒精作為汽油的替代品進(jìn)入市場,使得糧食市場和能源市場發(fā)生了前所未有的直接聯(lián)系。到1995年,谷物作為酒精的價(jià)值己經(jīng)超過了作為糧食的價(jià)值。西方國家已經(jīng)或正在考慮用從谷物提取的酒精來替代一部分進(jìn)口石油。

  如果上述斷定為真,對于那些己經(jīng)用從谷物提取的酒精來替代一部分進(jìn)口石油的西方國家。以下哪項(xiàng),最可能是1995年后進(jìn)口石油價(jià)格下跌的后果?

  A.一些谷物從能源市場轉(zhuǎn)入糧食市場。

  B.一些谷物從糧食市場轉(zhuǎn)入能源市場。

  C.谷物的價(jià)格面臨下跌的壓力。

  D.谷物的價(jià)格出現(xiàn)上浮。

  解析:谷物可以提取出酒精來代替石油出口,因此可以判斷谷物和石油成為了相互競爭的替代品,因此如果石油價(jià)格下跌,那么谷物的價(jià)格也必然會下調(diào),否則進(jìn)口國將會進(jìn)口價(jià)格低廉的石油取代谷物。

  22.有八個(gè)教授參加學(xué)術(shù)會議后,互留通信地址,以保持聯(lián)絡(luò)。一年后,統(tǒng)計(jì)他們之間的通信情況如下:有一人給其他三個(gè)人寫過信,給其他人寫過兩封信的有三人,給其他人寫過一封信的有四個(gè)人。若以上統(tǒng)計(jì)屬實(shí),則最能得出以下哪項(xiàng)結(jié)論?

  A.通過書信來往,他們遞進(jìn)了友誼,也加強(qiáng)了學(xué)術(shù)聯(lián)系。

  B.八個(gè)教授都收到過其他教授的來信。

  C.至少有一個(gè)人收到信件后沒有都回復(fù)。

  D.雖然參加學(xué)術(shù)會議的時(shí)候大家不熟悉,但現(xiàn)在他們已經(jīng)成為至交。

  解析:選項(xiàng)首先排除A、D,沒有任何證據(jù)表明通過書信加強(qiáng)了學(xué)術(shù)聯(lián)系,也沒有證據(jù)表明他們成為了至交。下面看選項(xiàng)B、C,我們可以通過畫圖的方法來解決這個(gè)問題相對比較簡單,從而得出正確答案為C。

  23.金鳳扒雞是一個(gè)享有盛譽(yù)的百年扒雞品牌,在石家莊銷路很好,深受當(dāng)?shù)匕傩障矏郏刻煲簧鲜芯蜁粨屬徱豢。為了打開北京市場,金鳳扒雞改進(jìn)了工藝,用真空包裝,延長了保質(zhì)期,試圖在北京各大超市打開銷路,但是金鳳扒雞在北京卻很難銷售出去。

  下面除了哪一項(xiàng),都有助于解決上述現(xiàn)象?

  A.北京人和石家莊人的飲食習(xí)慣不同。他們更喜歡吃烤鴨而不是扒雞。

  B.恒慧通燒雞也采用真空包裝,保質(zhì)期一樣長,在北京的銷路卻很好。

  C.人們喜歡吃剛出鍋的新鮮扒雞,保質(zhì)期短的反而更受消費(fèi)者青睞。

  D.雖然金鳳扒雞是百年品牌,但是它的銷售地點(diǎn)僅在石家莊市區(qū),北京人對這個(gè)品牌不熟悉。

  解析:這是一道頗具爭議的題目,選項(xiàng)A、B、C、D分別從消費(fèi)者的喜好,競爭者,消費(fèi)偏好和品牌對金鳳扒雞在北京銷路受阻提供了良好的解釋。但是選項(xiàng)B可以從另外一個(gè)角度思考,恒惠通燒雞同樣采用真空包裝,保質(zhì)期一樣長,什么都和金鳳扒雞一樣,為什么銷路很好,而金鳳扒雞為什么銷路不好,這就不能解釋為什么金鳳扒雞在北京銷路不好的原因。

  24.中國自1978年改革開放以來28年間,從人口數(shù)量上說:農(nóng)民的數(shù)量并沒有減少,而是持續(xù)增加的,只不過這種增長速度相對于總?cè)丝诘脑鲩L速度來說,是比較低的。

  下列那一句話直接與上述信息矛盾?

  A.在總?cè)丝谥修r(nóng)民的數(shù)量在1978年到2006年間略微增長了。

  B.農(nóng)民在總?cè)丝谥械谋嚷蕪?978年的81.4%增加到了2006年的85.3%。

  C.中國勞動力的增長率和總?cè)丝诘脑鲩L率在1978年至今的28年里同時(shí)增加了。

  D.中國的勞動力中,農(nóng)民的比率從1978年的78%下降到了75.6%。

  解析:選項(xiàng)B可以看出,農(nóng)民相對總?cè)丝诘脑鲩L速度并不低,增長速度也是增加的。

  25.有三戶人家,每家有一孩子,他們的名字是:小萍(女)、小紅(女)、小虎。孩子的爸爸是老王、老張和老陳;媽媽是劉蓉、李玲和方麗。對于這三家人,已知:

 。1)老王家和李玲家的孩子都參加了少女舞蹈隊(duì)。

 。2)老張的女兒不是小紅。

 。3)老陳和方麗不是一家人。

  根據(jù)以上條件,確定以下哪項(xiàng)是正確的?

  A.老王、劉蓉和小萍是一家。

  B.老張、李玲和小紅是一家。

  C.老王、方麗和小紅是一家。

  D.老陳、方麗和小虎是一家。

  解析:邏輯推斷題。從條件1可以看出小虎不是老王和李玲的孩子,因?yàn)樾』⑹悄泻ⅰD敲纯梢酝瞥鲂∑己托〖t的爸爸可能是老王。根據(jù)條件2,老張的女兒不是小紅,所以可以確定老王的女兒是小萍。鎖定選項(xiàng)A,如果A正確,那么根據(jù)條件3老陳和李玲是一家,根據(jù)條件1,女兒是小紅;得出老陳、方麗和小虎是一家。那么選項(xiàng)D也正確。因此同時(shí)排除A、D。選項(xiàng)B與條件2不符,因此選擇C。

  26.5名犯罪嫌疑人被警察詢問,事后得知其中一名是罪犯。下面是五個(gè)嫌疑人的供述,其中只有3句真話。問,誰是罪犯?

  甲:丁是罪犯。

  乙:我是無辜的。

  丙:戊不是罪犯。

  。杭自谌鲋e。

  戊:乙說的是實(shí)話。

  A.戊    B.甲和丙    C.丁    D. 乙和戊

  解析:從選項(xiàng)入手,選項(xiàng)B,甲和丙是罪犯,可以推出乙、戊、丁、丙都為真,而題目表明只有3項(xiàng)為真,所以放棄B;選項(xiàng)C,丁是罪犯,可以推出甲、乙、丙、戊為真,同樣與題目要求不符;選項(xiàng)D,乙和戊是罪犯,只能推出丁為真。因此這道題選擇A。

  27.學(xué)生上完體育課后回到教室,有15人喝了飲水機(jī)里的純凈水,其中5人很快產(chǎn)生了腹瀉。飲水機(jī)里的純凈水馬上被送去檢驗(yàn),檢驗(yàn)的結(jié)果不能肯定其中有造成腹瀉的有害物質(zhì)。因此,喝了飲水機(jī)里的純凈水不是造成腹瀉的原因。

  如果上述檢驗(yàn)結(jié)果是正確的,則以下哪項(xiàng)對上述論證的評價(jià)最為恰當(dāng)?

  A.題干的論證有漏洞,因?yàn)樗鼪]有考慮到另一個(gè)事實(shí):哪些沒有喝了飲水機(jī)里的純凈水的人沒有造成腹瀉。

  B.題干的論證有漏洞,因?yàn)樗讶鄙僮C據(jù)證明某種情況存在,當(dāng)作有充分證據(jù)證明某種情況不存在。

  C.題干的論證有漏洞,因?yàn)樗鼪]有利用一個(gè)有力的論據(jù):為什么有更多人喝了飲水機(jī)里的純凈水沒有造成腹瀉。

  D.題干的論證有漏洞,因?yàn)樗鼪]有指出造成腹瀉的真正原因。

  解析:理解這道題非常簡單,舉個(gè)例子說明,如果一個(gè)人確實(shí)殺了人,但是法官沒有證據(jù)表明他是兇手,那么可能無法定罪,但終不能得出他不是兇手的結(jié)論,因?yàn)橥瑯記]有證據(jù)表明他不是兇手。題目中給出了同樣的問題,檢驗(yàn)結(jié)果不能肯定有造成腹瀉的有害物質(zhì),但是并不能說明純凈水不是造成腹瀉的原因。

  28.一項(xiàng)全球范圍的調(diào)查顯示,近 10年來:吸煙者的總數(shù)基本保持不變;每年只有10%的吸煙者改變自己的品牌,即放棄原有的品牌而改吸其他品牌;煙草制造商用在廣告上的支出占其毛收入的10%。在Z煙草公司的年終董事會上,懂事A認(rèn)為,上述統(tǒng)計(jì)表明,煙草業(yè)在廣告上的收益正好等于其支出,因此,此類廣告完全可以不做。以下哪項(xiàng),構(gòu)成對懂事A的結(jié)論的最有力質(zhì)疑?

  A.懂事A的結(jié)論忽視了:今年來各種品牌的香煙的價(jià)格都有了很大的變動。

  B.懂事A的結(jié)論基于一個(gè)錯(cuò)誤的假設(shè):每個(gè)吸煙者在某個(gè)時(shí)候只喜歡一種品牌。

  C.懂事A的結(jié)論基于一個(gè)錯(cuò)誤的假設(shè):每個(gè)煙草制造商只生產(chǎn)一種品牌。

  D.懂事A的結(jié)論忽視了:世界煙草業(yè)是一個(gè)由處于競爭狀態(tài)的眾多經(jīng)濟(jì)實(shí)體組成的。

  解析:董事A的判斷為廣告的支出正好與收益相等,因此得出廣告可以不做的結(jié)論。但是忽略了如果不做廣告是否能夠獲得與做廣告時(shí)除去廣告費(fèi)用后相同的收益,因?yàn)檫@部分收入也是由廣告產(chǎn)生的效益。因?yàn)橥袠I(yè)的其他競爭者會因?yàn)閺V告,增加10%的顧客,而不做廣告的煙草公司將有減少10%顧客的風(fēng)險(xiǎn)。因此選擇D

  29.在收款局的5個(gè)賬單收款員中,楊先生收款的不成功率最高。然而楊先生是這個(gè)局的職員中最好的賬單收款員。

  下面哪一項(xiàng),如果正確,最有助于解決上述短文中的明顯分歧?

  A.在加入收款局之前,楊先生是一個(gè)大百貨公司的信貸部的一名職員。

  B.收款局中的其他四個(gè)收款員都認(rèn)為楊先生是一個(gè)非常能干的帳單收款員。

  C.楊先生在過去的幾年內(nèi),每年收款成功的比率都保持相當(dāng)?shù)胤(wěn)定。

  D.這個(gè)收款局的大多數(shù)最困難的事情都是派楊先生去做的。

  解析:從題干入手分析,楊先生是最好的賬單收款員,但是收款不成功率最高。題目要求找到解釋這一矛盾的原因。顯然D最合適。

  30.在一次工程碩士入學(xué)考試中,據(jù)統(tǒng)計(jì),在英語、數(shù)學(xué)、語文和邏輯四個(gè)科目中:70%的考生英語及格了,75%的考生數(shù)學(xué)及格了,80%的考生語文及格了,85%的考生邏輯及格了。如果以上情況屬實(shí),那么,以下哪個(gè)選項(xiàng)一定正確:

  A.在此次考試中,大部分考生四門課都及格了。

  B.在此次考試中,大部分考生四門課沒有都及格。

  C.在此次考試中,有不少于10%的考生,四門課程都及格了。

  D.在此次考試中,至少有10%的考生,四門課程都沒及格。

  解析:不少于=大于等于,至少=大于等于。選線排除A、B,選項(xiàng)過于籠統(tǒng),并且與C、D存在重復(fù)關(guān)系。選項(xiàng)D與題干明顯不符合,選擇C。

 31.某游戲的游戲規(guī)則有這樣一條規(guī)定:任何在游戲中拒絕成為選手的人將在拒絕的時(shí)候被減掉10分。

  下列哪一個(gè)是此條規(guī)則所暗含的?

  A.同意參加游戲的所有人的得分將高于那些因?yàn)榇艘?guī)則而被判罰分的人。

  B.一個(gè)人在拒絕玩這個(gè)游戲的同時(shí),依然是這個(gè)游戲的參與者。

  C.最初同意成為游戲參加者然后在游戲進(jìn)行中退出的人可以避免被罰10分。

  D.一個(gè)拒絕玩這個(gè)游戲的人不能在游戲中被宣布為失敗者。

  解析:由于拒絕玩這個(gè)游戲,會被剪掉20分,說明依舊是游戲的參與者,選B。

  32.受到廣泛贊揚(yáng)的斯坦尼斯拉夫斯基演員訓(xùn)練法來自他做年輕演員時(shí)對戲劇不熟練和易受影響的老套做法。對該方法的理解必須基于斯坦尼斯拉夫斯基個(gè)人的演藝研究,從而從典型化的姿勢、反復(fù)嘗試的嗓音語調(diào)和標(biāo)準(zhǔn)的感情模式的誘惑中解脫出來。盡管他的美國門徒有過要求,但是這位俄國導(dǎo)演從未打算寫一本對表演問題提出嚴(yán)格解決方法的教科書。

  關(guān)于表演這個(gè)問題,上文作者對斯坦尼斯拉夫斯基的演員訓(xùn)練法持有怎樣的觀點(diǎn)?

  A.斯坦尼斯拉夫斯基的美國門徒,在使用他的方法訓(xùn)練時(shí),使得這種方法喪失了原有的靈活性和探索性。

  B.表演本質(zhì)上是自發(fā)的感情表達(dá),系統(tǒng)的訓(xùn)練經(jīng)常干涉它。

  C.實(shí)際上唯一需要給予年輕演員的建議是他們必須系統(tǒng)地抵制他們在戲劇表演中所表現(xiàn)出來的那些老套的模式。

  D.斯坦尼斯拉夫斯基的方法主要是針對那些必須克服有做作和不成熟表演行為的年輕演員。

  解析:斯坦尼斯拉夫斯基之所以未打算對表演問題提出嚴(yán)格解決方法的教科書,本意是不想使該方法失去靈活性和探索性。因此選擇A。

  33.已發(fā)現(xiàn)有國產(chǎn)嬰幼兒奶粉三聚氰胺含量超標(biāo)。

  如果上述斷定是真的,那么在下述三個(gè)斷定中不能確定真假的是:

 、瘛a(chǎn)嬰幼兒奶粉沒有三聚氰胺含量不超標(biāo)的。

 、、有國產(chǎn)嬰幼兒奶粉三聚氰胺含量沒超標(biāo)。

 、蟆⑺袊a(chǎn)嬰幼兒奶粉三聚氰胺含量都未超標(biāo)。

  A.只有Ⅰ和Ⅱ。    B.Ⅰ、Ⅱ和Ⅲ。    C.只有Ⅰ和Ⅲ。    D.只有Ⅱ。

  解析:選項(xiàng)1不能確定真假,從部分不能推出整體;選項(xiàng)2不能確定真假,從一部分不能退出另一部分具有相反的結(jié)論。條件3明顯可以判定為假,選擇A。

  34.股民購買股票的目的就是盈利。股民購買股票后,股票價(jià)格上漲,就會盈利,否則,就會虧損。一般情況下,盈利或者虧損達(dá)到10%就應(yīng)該拋出,以避免那些不可預(yù)測的突然變化所引起的損失。在股市上,有經(jīng)驗(yàn)的股民往往會選擇跟莊戰(zhàn)術(shù)來買賣股票。即大股東買進(jìn)的時(shí)候,他們也買進(jìn),大股東拋出的時(shí)候,他們就拋出。因?yàn)榇蠊蓶|有可能掌握更多上市公司和市場行情方面的信息。這些天,天虹公司的股票一直在下跌,但是大股東們卻一直在大量購進(jìn)這只股票。

  以上事實(shí)最能支持以下哪種預(yù)測?

  A.天虹公司的股票價(jià)格會繼續(xù)下降,但速度會放慢。

  B.天虹公司的股票價(jià)格會在短期內(nèi)大幅度上漲。

  C.股民盲目跟莊不是一個(gè)好的盈利方法。

  D.最近大股東對天虹公司股價(jià)變動判斷失誤,有可能造成巨大經(jīng)濟(jì)損失。

  解析:題目結(jié)論如下:大股東買進(jìn)的股票會漲,因?yàn)樗麄冋莆樟烁嗌鲜泄竞褪袌鲂星榉矫娴男畔。所以選擇B。

  35.作為身體的精密平衡系統(tǒng)的一部分,人類心臟分泌某種激素,這些激素能夠控制血液中的含鹽量和體內(nèi)參與循環(huán)的血量。人體僅需要極少的這種激素。它在控制血壓時(shí)極其重要,并且在患心臟病的病人血中經(jīng)常檢出。

  基于上面的研究成果,下面的哪個(gè)論點(diǎn)一定正確?

  A.假如心臟分泌的這種激素缺乏,將導(dǎo)致低血壓。

  B.一個(gè)僅僅由機(jī)械閥制作的設(shè)備被用作人工心臟,它將不能執(zhí)行人類心臟的所有功能。

  C.少量這種激素的分泌對人體的影響是長期性的。

  D.任何控制血壓的藥將通過影響心臟此種激素分泌量達(dá)到效果。

  解析:選擇B,因?yàn)橛蓹C(jī)械閥制作的人工心臟,顯然不具有分泌激素的功能,因此就不能執(zhí)行人類心臟的所有功能,題干反復(fù)說明就是為了得出這樣的結(jié)論。

  36.某出版社近年來出版物的錯(cuò)字率較前幾年有明顯的增加,引起了讀者的不滿和有關(guān)部門的批評,這主要是由于該出版社大量引進(jìn)非專業(yè)編輯所致。當(dāng)然,近年來該社出版物的大量增加也是一個(gè)重要原因。

  上述議論的漏洞,也類似地出現(xiàn)在以下哪項(xiàng)中?

  I 美國航空公司近兩年來的投訴率比前幾年有明顯的下降。這主要是由于該航空公司在裁員整頓的基礎(chǔ)上,有效地提高了服務(wù)質(zhì)量。當(dāng)然,“9.11”事件后航班乘客數(shù)量的銳減也是一個(gè)重要原因。

  II 統(tǒng)計(jì)數(shù)字表明:近年來我國心血管病的死亡率,即由心血管病導(dǎo)致的死亡率在整個(gè)死亡人數(shù)中的比例,較前有明顯增加,這主要是由于隨著經(jīng)濟(jì)的發(fā)展,我國民眾的飲食結(jié)構(gòu)和生活方式發(fā)生了容易誘發(fā)心血管病的不良變化。當(dāng)然,由于心血管病主要是老年病,因此,我國人口的老齡化,即人口中老年人比例的增加也是一個(gè)重要原因。

  III 某市去年的高考錄取率比前年增加15%,這主要是由于各中學(xué)狠抓了教育質(zhì)量。當(dāng)然,另一個(gè)重要原因是,該市今年參加高考的人數(shù)比去年增加了20%。

  A.僅僅I和II。   B.僅僅II和III。   C.僅僅I和III。   D.僅僅I。

  解析:題目中的邏輯錯(cuò)誤是強(qiáng)加因果,出版物數(shù)量的增加與錯(cuò)字率的增加沒有因果關(guān)系。根據(jù)這一思路對選項(xiàng)進(jìn)行判斷,選項(xiàng)I航班數(shù)銳減與投訴率的下降沒有因果關(guān)系,與題目中犯了同樣的錯(cuò)誤;選項(xiàng)II,心血管病是老年病,我國人口老齡化,因此我國心血管死亡率增加,這是一個(gè)有效地推理過程;選項(xiàng)III,參加高考的人數(shù)增加,并不能推出錄取比例增加的這一結(jié)論。也是屬于沒有因果關(guān)系。另外從絕對和相對概念這個(gè)角度分析,同樣可以得到相似的結(jié)論。選C

  37.從前,一個(gè)孤島上有一個(gè)奇怪的風(fēng)俗:凡是漂流到這個(gè)島上的外鄉(xiāng)人都要作為祭品被殺掉,但允許被殺的人在臨死前說一句話,然后由這個(gè)島上的長老判定這句話是真的還是假的。如果說的是真話,則將這個(gè)外鄉(xiāng)人在真理之神面前殺掉;如果說的是假話,則將他在錯(cuò)誤之神面前殺掉。有一天,一位哲學(xué)家漂流到了這個(gè)島上,他說了一句話,使得島上的人沒有辦法殺掉他。

  該哲學(xué)家必定說了下面哪一句話?

  A.你們這樣做不合乎理性。    B.我將死在真理之神面前。

  C.無論如何我都會死。      D.我將死在錯(cuò)誤之神面前。

  解析:A選項(xiàng),可以判斷為真,會在真理之神面前殺掉;B選項(xiàng),可以判定為真,因?yàn)樗麜涝谡胬碇衩媲埃贿x項(xiàng)C,同樣可以判定為真,選項(xiàng)D我將死在錯(cuò)誤之神面前,如果為真,那么說明他說的是假話,表明他將死在真理之神面前為真,這本身與這句話矛盾了,所以這句話自背反命題,選D。

  38.有的地質(zhì)學(xué)家認(rèn)為,如果地球的未勘探地區(qū)中單位面積的平均石油儲藏量能和已勘探地區(qū)一樣的話,那么,目前關(guān)于地下未開采的能源含量的正確估計(jì)因此要乘上一萬倍,由此可得出結(jié)論,全球的石油需求,至少可以在未來五個(gè)世紀(jì)中得到滿足,即便此種需求每年呈加速上升的趨勢。為使上述論證成立,以下哪項(xiàng)是必須假設(shè)的?

  A. 地球上未勘探地區(qū)的總面積是已勘探地區(qū)的一萬倍。

  B. 在未來至少五個(gè)世紀(jì)中,世界人口的增長率不會超過對石油需求的增長率。

  C. 新技術(shù)將使未來對石油的勘探和開采比現(xiàn)在更為可行。

  D. 地球上未勘探地區(qū)中儲藏的石油可以被勘測和開采出來。

  解析:答案為D,題目中通過假設(shè)得出了未來五個(gè)世紀(jì)的石油需求相會得到滿足的結(jié)論,假設(shè)條件為地球上未勘探地區(qū)的單位面積石油儲量與已勘探地區(qū)一樣多。但僅有這個(gè)前提還是不夠的,還需要這些為勘探的時(shí)候都是可以被開采出來的這一假設(shè)。

  39.某礦山發(fā)生了一起嚴(yán)重的安全事故。關(guān)于事故原因,甲乙丙丁四位負(fù)責(zé)人有如下斷定:

  甲:如果造成事故的直接原因是設(shè)備故障,那么肯定有人違反操作規(guī)程。

  乙:確實(shí)有人違反操作規(guī)程,但造成事故的直接原因不是設(shè)備故障

  丙:造成事故的直接原因確實(shí)是設(shè)備故障,但并沒有人違反操作規(guī)程。

  。涸斐墒鹿实闹苯釉蚴窃O(shè)備故障。

  如果上述斷定只有一個(gè)人的斷定為真,那么以下斷定都不可能為真,除了:

  A.甲的斷定為真,有人違反了操作規(guī)程。

  B.甲的斷定為真,但沒有人違反操作規(guī)程。

  C.乙的斷定為真。

  D.丙的斷定為真。

  解析:這道題屬于邏輯推理題,由于只有一個(gè)人判斷為真,因此丙、丁的判斷首先排除,因?yàn)檫@兩個(gè)判定并不沖突。甲、乙的推斷有矛盾,因此兩項(xiàng)必有一個(gè)是正確的。甲的推斷結(jié)構(gòu)是如果B推出A,那么 C必然推出B;乙的推斷結(jié)構(gòu)是C推出B,但B不能退出A,是甲判定的一個(gè)逆向命題。選項(xiàng)B是正確答案,因?yàn)檫x項(xiàng)B肯定了甲的推斷,同時(shí)否定了乙的發(fā)生,因此是真命題。其他選項(xiàng)均不正確。

  40.心臟的搏動引起血液循環(huán)。對同一個(gè)人,心率越快,單位時(shí)間進(jìn)入循環(huán)的血液量越多。血液中的紅血球運(yùn)輸氧氣。一般地說,一個(gè)人單位時(shí)間通過血液循環(huán)獲得的氧氣越多,他的體能及其發(fā)揮就越佳。因此,為了提高運(yùn)動員在體育比賽中的競技水平,應(yīng)該加強(qiáng)他們在高海拔地區(qū)的訓(xùn)練,因?yàn)樵诟吆0蔚貐^(qū),人體內(nèi)每單位體積血液中含有的紅血球數(shù)量,要高于在低海拔地區(qū)。

  以下哪項(xiàng)是題干的論證必須假設(shè)的?

  A.運(yùn)動員在高海拔地區(qū)的心率不低于在低海拔地區(qū)。

  B.不同運(yùn)動員的心率基本相同。

  C.運(yùn)動員的心率比普通人慢。

  D.在高海拔地區(qū)訓(xùn)練能使運(yùn)動員的心率加快。

  解析:此題論證說明運(yùn)動員在高海拔地區(qū)訓(xùn)練體內(nèi)的紅血球數(shù)量高于低海拔地區(qū),從而達(dá)到最佳體能的目的。B和C與題意不符,選A。

 41~45題基于以下題干:

  七個(gè)學(xué)生R、S、T、V、W、X、Y,被分成兩個(gè)學(xué)習(xí)小組。第一組有三名成員,第二組有四名成員。學(xué)生們的分組必須符合以下要求:

  R和T不能在同一個(gè)小組;

  如果S在第一組,那么V必須在第一組;

  如果W在第一組,那么T必須在第二組;

  X必須在第二組。

  41.如果W在第一組,那么以下哪項(xiàng)也一定在第一組?

  A.R    B.S    C.T    D.V

  解析:根據(jù)條件第一組有三名成員。因?yàn)镽和T不能在一組,又因?yàn)闂l件:如果W在第一組,那么T必須在第二組,則R可以在第一組

  42.如果T和Y都在第一組,那么以下哪項(xiàng)一定是真的?

  A.S和V在同一組。   B.S和W在同一組。   C.V和R在同一組。   D.W和T在同一組。

  解析:如果T和Y在第一組,R和T又不能在一組,可知R只能在第二組,而X必須在第二組。如果S在第一組,根據(jù)條件,V也需在第一組,而第一組只有三個(gè)成員,那么S和V就不能在同一組。而如果W在第一組,那么T必須在第二組。但是T已經(jīng)在第一組,則W必須在第二組,加之第二組有四人,那么S也需在第二組,那么S和W為一組。

  43.如果W和T在同一組,那么以下哪項(xiàng)可能在同一組,除了:

  A.R和S    B.S和Y    C.T和Y    D.V和Y

  答案:B

  解析:

  條件分析:

  1、R1且T2;或者 R2且T1

  2、S1→V1;即S2的時(shí)候,V可以自由分配。

  3、W1→T2;即W2的時(shí)候,T可以自由分配。

  4、X2

  43題的臨時(shí)題干要求WT在同一組,根據(jù)條件3,他們不可能在第一組,不然就違背了題設(shè)。所以,這個(gè)時(shí)候,WT都在都在第二組。第二組已經(jīng)有了WTX三個(gè)成員,只有一個(gè)空位了。根據(jù)條件1,可以知道,R在第一組。因此,除了跟WTX搭伴的元素外,任何二人組合都必須在第一組了。

  選項(xiàng)A,第一組RSV;第二組WTXY,有可能。

  選項(xiàng)B,第一組RSY;則第一組還必須有V,顯然超編了。因?yàn)榈谝唤M只能有三個(gè)人。因此,此項(xiàng)為答案。

  選項(xiàng)C,依然是第一組RSV;第二組WTXY這樣的組合。

  選項(xiàng)D,第一組RVY;第二組WTXS,不矛盾。

  44.如果V和Y在同一組,那么以下哪項(xiàng)一定是真的?

  A.R在第一組。    B.S在第一組。    C.W在第二組。    D.Y在第二組。

  解析:如果V和Y在第一組,那么根據(jù)條件如果S在第一組,那么V必須在第一組;說明S可以在第一組,又因?yàn)槿绻鸚在第一組,那么T必須在第二組,但是R和T又不能在一組,則R必須在第一組,W和T可以在第二組。則答案C正確。

  45.如果S在第一組,那么以下哪項(xiàng)一定是真的?

  A.T在第一組。    B.T在第二組。    C.Y在第一組。    D.Y在第二組。

  解析:如果S在第一組,根據(jù)條件V必須在第一組,如果W在第一組,那么T必須在第二組;R和T不能在同一個(gè)小組;那么W或R可以在第一組,X必須在第二組。因此在第一組的可以是S或V或R或W,因此答案D正確。

  46~50題基于以下題干:

  六位教授F、G、H、J、K、L,將評審馬、任、孫、吳博士的論文4篇。評審需遵守以下原則:

 。1)每位教授只評審一篇博士論文;

 。2)每篇博士論文至少有一位教授評審;

 。3)H與F評審?fù)黄┦空撐模?/p>

 。4)L只與其他教授中的一位同評一篇博士論文;

 。5)G評審馬博士的論文;

 。6)J評審馬博士或吳博士的論文;

 。7)H不評審吳博士的論文。

  46.如果K不評審孫博士的論文,那么以下哪項(xiàng)一定是真的?

  A.L評審馬博士的論文           B.L評審孫博士的論文

  C.F和H評審任博士的論文        D.F和H評審孫博士的論文

  解析:H不評審吳博士論文,H與F又評審?fù)黄撐模荒敲碒和F有可能評審馬或任或?qū)O博士的論文,但是G評審馬博士的論文,J評審馬博士或者吳博士的論文,那么H與F就有可能評審任或?qū)O博士的論文,K不評審孫博士的論文,K就有可能評審任博士的論文,那么H和F就會評審孫博士的論文。

  47.以下哪項(xiàng)完整地列出了L可能評審的博士論文?

  A.馬、任      B.馬、孫、吳     C.馬、任、吳       D.馬、任、孫、吳

  解析:根據(jù)條件四,L只與其他教授中的一位同評一篇博士論文,而每篇博士論文至少有一位教授評審,也就是說一篇論文有可能是兩個(gè)教授或者三個(gè)教授一起評論。因此,L可以評論馬、任、孫、吳的博士論文。

  48.以下哪項(xiàng)可能是真的?

  A.F和G評審馬博士的論文。

  B. F和L評審吳博士的論文。

  C. K評審吳博士的論文并且L評審馬博士的論文。

  D. L評審任博士的論文并且F評審孫博士的論文。

  解析:G評審馬博士的論文,J評審馬博士或吳博士的論文,H不評審吳博士的論文,H與F又評審?fù)黄撐;那么H和F有可能評審馬或任或?qū)O博士的論文,那么H與F就有可能評審任或?qū)O博士的論文。加之題一,K不評審孫博士的論文,K就有可能評審任博士的論文,條件三H與F評審?fù)黄┦空撐,那么H和F就會評審孫博士的論文。條件四:L只與其他教授中的一位同評一篇博士論文,L可以與K評審任博士論文,故D正確

  49.以下哪項(xiàng)不可能是真的?

  A. L和G評審馬博士的論文        B. L和K評審馬博士的論文

  C. L和K評審任博士的論文        D. L和K評審孫博士的論文

  解析:根據(jù)條件(4)L只與其他教授中的一位同評一篇博士論文,而條件(5)G評審馬博士的論文,那么可知L和G可評審馬博士論文,故A正確。根據(jù)條件(6)J評審馬博士或吳博士的論文,可以得知L可以和J可以評審吳博士或者馬博士的論文,H不評審吳博士的論文,那么H可以評論任博士和孫博士的論文。K可以評論吳博士、任博士、孫博士的論文。加之條件(4),得知答案C和D正確,故B錯(cuò)誤

  50.如下哪位教授不可能審閱任博士的論文?

  A.F   B.K    C.J   D. L

  解析:條件(6)J評審馬博士或吳博士的論文;而條件(1)每位教授只評審一篇博士論文,根據(jù)兩個(gè)條件可以推斷J不可能審閱任或?qū)O博士的論文,故選C

  模擬題1參考答案:

01. C 02. A 03. C 04. D 05. C
06. C 07. A 08. B 09. A 10. A
11. A 12. B 13. B 14. A 15. C
16. D 17. D 18. C 19. A 20. B
21. C 22. C 23. B 24. B 25. C
26. A 27. B 28. D 29. D 30. C
31. B 32. A 33. A 34. B 35. B
36. C 37. D 38. D 39. B 40. A
41. A 42. B 43. B 44. C 45. D
46. D 47. D 48. D 49. B 50. C
 

 

 

 

(50題,每題2分,滿分100分)

  1.任何人都沒有吃過雅各島上的任何水果,所以無法知道雅各島上任何水果的口味。

  為了合乎邏輯的推出上述結(jié)論,需要假設(shè)下面哪項(xiàng)為前提?

  A.如果一種水果有人品嘗過,就可以知道其口味。(充分條件,不是前提)

  B.只憑某些人的品嘗無法真正知道某種水果的口味。

  C.要知道某種水果的口味,需要有人去品嘗。

  D.人們是通過嗅覺來確定水果口味的。

  解析:這道題需要補(bǔ)全假設(shè),結(jié)論是:任何人都無法知道任何水果的味道,前提是:任何人都沒有吃過雅各島上的任何水果。補(bǔ)充前提是:要知道水果的口味,需要有人去品嘗,應(yīng)該選擇C。選項(xiàng)A,是充分條件,但不是構(gòu)成結(jié)論的原因,屬于強(qiáng)拉因果;B、D都為不合理的假設(shè)。

  2.最近的一項(xiàng)研究指出:“經(jīng)常吃沙棘果對兒童的智力發(fā)育有益。”研究人員對560名兒童進(jìn)行調(diào)查,發(fā)現(xiàn)那些經(jīng)常吃沙棘果的兒童,其智力水平較很少吃沙棘果的兒童要高。因此,研究人員發(fā)現(xiàn)了沙棘果與兒童智力發(fā)育之間的聯(lián)系。

  以下哪項(xiàng)如果為真,最不可能削弱上述論證?

  A.對成年人的研究發(fā)現(xiàn),每天吃沙棘果的人智力水平并不比很少吃沙棘果人的高。

  B.調(diào)查顯示:沙棘果價(jià)格非常高,只有富裕家庭的兒童才經(jīng)常吃,同時(shí)這些家庭有條件實(shí)現(xiàn)兒童的早期智力開發(fā)。

  C.這項(xiàng)兒童發(fā)育研究的課題負(fù)責(zé)人是沙棘果生產(chǎn)商,其目的就是要擴(kuò)展沙棘果的銷售渠道。

  D.沙棘果是兒童喜歡的食品,家長經(jīng)常把沙棘果作為禮物獎(jiǎng)給智力表現(xiàn)優(yōu)異的孩子。

  解析:吃沙棘果與兒童智力發(fā)展是否存在必然聯(lián)系。通過樣本調(diào)查判斷出吃沙棘果與兒童智力發(fā)展存在正相關(guān)的關(guān)系。B選項(xiàng)說明家庭條件是否富裕與兒童智力發(fā)展存在關(guān)聯(lián)聯(lián)系;C選項(xiàng)說明題目中的結(jié)論是由于生產(chǎn)商的廣告產(chǎn)生的以偏概全的結(jié)論;D選項(xiàng)屬于因果倒置,成績優(yōu)異是因,吃沙棘果是結(jié)果。

  3.凡金屬都是導(dǎo)電的。銅是導(dǎo)電的,所以銅是金屬。

  下面哪項(xiàng)與上述推理結(jié)構(gòu)最相似?

  A.所有的鳥都是卵生動物,蝙蝠不是卵生動物,所以,蝙蝠不是鳥。

  B.所有的鳥都是卵生動物,天鵝是鳥,所以天鵝是卵生動物。

  C.所有從事工商管理的都要學(xué)習(xí)企業(yè)管理,老陳是學(xué)習(xí)企業(yè)管理的,所以,老陳是從事工商管理工作的。

  D.華山險(xiǎn)于黃山,黃山險(xiǎn)于泰山,所以華山險(xiǎn)于泰山。

  解析:題目結(jié)構(gòu)是所有的A具有某中屬性,B屬于A,因此B具有A的屬性,選項(xiàng)C與上述結(jié)論類似。

  4.為了有助于人們選擇最滿意的城市居住,有關(guān)部門實(shí)施了一項(xiàng)評選“最舒適城市”的活動。方法是,選擇十個(gè)方面,包括社會治安、商業(yè)設(shè)施、清潔程度、綠化程度、教育設(shè)施、旅游文化景點(diǎn)等等,每個(gè)方面按實(shí)際質(zhì)量的高低,評以1分至10分之間的某一分值,然后求得十個(gè)分值的平均數(shù)即是這個(gè)城市的舒適性指數(shù)。

  以下哪項(xiàng)是實(shí)施上述活動需要預(yù)設(shè)的前提?

  Ⅰ、城市的各種舒適性質(zhì)量程度都可以用準(zhǔn)確的數(shù)字表達(dá)。

 、、城市的各種舒適性對于居民來說都是同等重要的。

 、蟆⒕用裼凶杂蛇x擇居住城市的權(quán)利并且大都樂于這樣做。

  A.僅Ⅰ。      B.僅Ⅲ。      C.僅Ⅰ和Ⅱ。      D.Ⅰ、Ⅱ和Ⅲ。

  解析:這道題是補(bǔ)充假設(shè)的問題,由于每個(gè)人對“舒適程度”的感覺是不一樣的,因此題干結(jié)論若要正確,條件Ⅱ是必須的前提;若題目中結(jié)論正確,需要假設(shè)題目中的十個(gè)指標(biāo)存在量化標(biāo)準(zhǔn),因此條件Ⅰ為必選項(xiàng);條件Ⅲ存在爭議,不過可以從另外一個(gè)角度理解,便于理解題目的結(jié)論,如果居民沒有選擇居住地的權(quán)利,那么題目中的滿意度調(diào)查就沒有意義,所以也得不到上述結(jié)論。

  5.衛(wèi)生部的官員們對牟定縣狂犬病疫情有以下斷定:

  (1)該縣所有的狗都得了狂犬病。

  (2)該縣有些斑點(diǎn)狗得了狂犬病。(真)

  (3)該縣有些狗得了狂犬病。

  (4)該縣有些狗沒得狂犬病。(真)

  其實(shí)上述斷定中只有兩個(gè)與事實(shí)相符。根據(jù)如上的情況,以下哪項(xiàng)結(jié)論是可能成立的?

  I.該縣的狗都是斑點(diǎn)狗。

  II. 該縣沒有斑點(diǎn)狗可能得狂犬病。

  III. 該縣的斑點(diǎn)狗并非可能沒有得狂犬病。

  IV. 該縣的狗不可能都沒得狂犬病。

  A.僅I      B.僅III      C.僅II和IV      D.僅IV。

  解析:這道題屬于邏輯推理中真假話題的問題。解析這種問題按照1、先找矛盾;2、繞開矛盾;3、推出答案的步驟進(jìn)行。首先先找矛盾,首先1和2是一對矛盾,3和4是一對矛盾,只有一個(gè)正確,2、繞開矛盾,如果1正確,那么3和4都假,因此只能2為真,如果2為真,只能判斷4為真,3為假,3、推出答案,條件2有些斑點(diǎn)夠得了狂犬病,可推出該縣沒有斑點(diǎn)狗可能得狂犬病,大家注意必然=沒有……必然;同時(shí)根據(jù)條件4可以得出該縣的狗不可能都沒得狂犬病。

  6.如果一個(gè)兒童的體重與身高的比值超過本地區(qū)80%兒童的水平,就稱其為肥胖兒。根據(jù)歷年的調(diào)查結(jié)果,15年來,臨江市的肥胖兒的數(shù)量一直在穩(wěn)定增長。

  如果以上斷定為真,則以下哪項(xiàng)也必為真?

  A.臨江市每一個(gè)肥胖兒的體重都超過全市兒童的平均體重。

  B.15年來,臨江市的兒童的體育鍛煉越來越不足。

  C.臨江市的非肥胖兒的數(shù)量15年來不斷增長。

  D.15年來,臨江市體重不足標(biāo)準(zhǔn)體重的兒童數(shù)量不斷下降。

  解析:這道題屬于迷惑推理題,大家務(wù)必要小心。首先大家可以看出臨江市兒童數(shù)量是在增長的;兒童由肥胖和非肥胖組成;肥胖兒童的數(shù)量穩(wěn)定增長,說明非肥胖兒童的數(shù)量也在穩(wěn)定增長;沒有證據(jù)表明非肥胖兒童的數(shù)量在不斷下降。

  7.孩子出生后的第一年在托兒所里度過,會引發(fā)孩子的緊張不安。在我們的研究中,有464名12~13歲的兒童接受了特異情景測試法的測驗(yàn),該項(xiàng)測驗(yàn)意在測試兒童1歲時(shí)的狀況與對母親的依附心理之間的關(guān)系。其結(jié)果:有41.5%曾在托兒所看護(hù)的兒童和25.7%曾在家看護(hù)的兒童被認(rèn)為緊張不安,過于依附母親。

  以下哪項(xiàng)如果為真,最沒有可能對上述研究的推斷提出質(zhì)疑?

  A.出生后第一年在家看護(hù)的孩子多數(shù)是由祖父母或外祖父母看護(hù)的,并形成濃厚的親情。

  B.這項(xiàng)研究的主持者被證實(shí)曾經(jīng)在自己的幼兒時(shí)期受到過長時(shí)間來自托兒所阿姨的冷落。

  C.針對孩子母親的另一部分研究發(fā)現(xiàn):由于孩子在家里表現(xiàn)出過度的依附心理,父母因此希望將其送入托兒所予以矯正。

  D.研究中所測試的孩子并不是從托兒所看護(hù)和在家看護(hù)兩種情況下隨機(jī)抽取的。因此,這兩組樣本兒童的家庭很可能有系統(tǒng)性的差異存在。

  解析:這種題最好的解決辦法是否定推斷,把你認(rèn)為可能的答案否定,帶入題目中判斷。如果能夠提出質(zhì)疑,說明你的判斷是正確的。A選項(xiàng)。

  8.在玉米地中套種的小麥有可能得小麥黃葉病,這種病是由于光照不足造成的,一旦光照充足,比如減少玉米植株數(shù)量,或者玉米收割后,癥狀很快就會消失,且不會影響產(chǎn)量。而小麥銹病是由花斑銹菌引起,如果在病癥初起時(shí)不及時(shí)恰當(dāng)應(yīng)對,就會使小麥植株很快枯死。由于小麥銹病和小麥黃葉病初期癥狀很難區(qū)分,所以,在玉米地中套種小麥,必須高度注意小麥銹病的定期篩查和預(yù)防。

  以下哪項(xiàng)最可能是上述論證所假設(shè)的?

  麥黃葉病不會誘發(fā)小麥銹病。

  B.花斑銹菌不會因?yàn)槌渥愎庹斩π←湹那趾αΑ?/p>

  C.小麥銹病如果及時(shí)恰當(dāng)處理,就不會使小麥減產(chǎn)。

  D.科學(xué)家一直沒有找到有效遏制小麥銹病的藥物。

  解析:假設(shè)推斷。陽光充足,小麥黃葉病會消失,產(chǎn)量不影響;花斑銹菌,使小麥植株枯死。因此需要假設(shè)陽光充足不會抑制花斑銹菌的發(fā)病,才能得出小麥黃葉不發(fā)病時(shí),花斑銹菌仍然會發(fā)病,導(dǎo)致兩種病情難以區(qū)分。換一種角度,如果小麥黃葉病不會和花斑銹菌同時(shí)發(fā)病,那么就不存在難以區(qū)分病情的問題了。

  9.公司準(zhǔn)備從五個(gè)業(yè)務(wù)骨干中選幾個(gè)人去中央戲劇學(xué)院進(jìn)修。鑒于業(yè)務(wù)員們的工作關(guān)系,總經(jīng)理建議:

 。1)如果選鞏俐,那么必須選章子怡并且不能選周迅。

 。2)如果選章子怡或者選周迅,則不能選吳孟達(dá)。

 。3)不能既不選周星馳也不選吳孟達(dá)。

  考慮總經(jīng)理的建議,董事會認(rèn)為,鞏俐必須去中央戲劇學(xué)院進(jìn)修,這樣誰將跟她一起去進(jìn)修?

  A.章子怡和周星馳    B.章子怡和吳孟達(dá)   C.周星馳和周迅    D.吳孟達(dá)和周迅

  解析:邏輯推斷。如果鞏麗必須去,根據(jù)條件1章子儀必須去,而周迅一定不去,從而排除C、D選項(xiàng),根據(jù)條件2,如果選章子儀,那么吳孟達(dá)不能去。排除B 。

  繼續(xù)進(jìn)行推理,根據(jù)條件3必須在周星馳和吳孟達(dá)中選一個(gè),因?yàn)楦鶕?jù)條件2已經(jīng)排除吳孟達(dá),所以周星馳必須去。選擇A。

  10.20世紀(jì)60年代早期之前,挪威的斯塔溫格是一個(gè)安靜而和平的小鎮(zhèn)。進(jìn)入60年代以來,這里成為了挪威近海石油勘探中心。從此,暴力犯罪和毀壞公物的現(xiàn)象在斯塔溫格小鎮(zhèn)急劇增加。顯然這些社會問題是是石油繁榮的副產(chǎn)品。

  下列哪一個(gè)選項(xiàng)最強(qiáng)烈的支持上面的論述。

  A.暴力犯罪和毀壞公物在沒有石油繁榮的挪威城鎮(zhèn)保持低水平。

  B.斯塔溫格居民很少對這里是近海石油勘探中心感到遺憾。

  C.挪威社會學(xué)家對斯塔溫格的暴力犯罪和毀壞公物現(xiàn)象持續(xù)增加表示關(guān)切。

  D.非暴力犯罪、毒品、離婚,在這個(gè)小鎮(zhèn)也以同樣的速度增加著。

  解析:題目把社會問題的原因歸結(jié)為石油繁榮,最強(qiáng)烈支持的論據(jù)為在沒有石油繁榮的時(shí)期,社會問題處于低水平。因此選A。

11.認(rèn)為只傷害自己而沒有傷害到別人的行為并沒有什么錯(cuò)誤的想法,通常伴隨著對人與人之間相互依賴的人際關(guān)系的忽視。毀壞一個(gè)人自己的健康或者生命意味著不能你將不能為家庭或者社會提供幫助,相反意味著你將額外享有那些本來就有限的諸如食物、健康服務(wù)和教育等社會資源,而不是相反將它們回報(bào)給社會。

  以下哪個(gè)選項(xiàng)如果正確,最強(qiáng)烈的地支持題干所表達(dá)的觀點(diǎn)。

  A.本來可以避免的疾病和意外事故所造成的花銷增加了每個(gè)人的健康保險(xiǎn)金。

  B.傷害一個(gè)人能夠?qū)е麻g接的利益,諸如可以使別人獲得與健康領(lǐng)域相關(guān)聯(lián)的工作。

  C.一個(gè)人對社會所作的貢獻(xiàn)可以通過他的健康程度來衡量。

  D.由喝酒、吸煙和吸食毒品所導(dǎo)致的主要傷害是由使用那些東西的人來承受的。

  解析:題目中的結(jié)論是傷害自己的行為與傷害別人的行為同樣都是錯(cuò)誤的。原因是傷害自己將不能為家庭和社會提供幫助,同時(shí)會向社會索取的更多。這道題的關(guān)鍵是大家要讀懂題干,這樣從選項(xiàng)中我們不難選出A,本來可以避免的個(gè)人傷害回增加健康保險(xiǎn)金,從而增加社會的負(fù)擔(dān)。

  12.大多數(shù)道路的修理比預(yù)算的要花費(fèi)更多的時(shí)間和金錢,但是去年夜間修理京津塘高速公路和類似的道路并未比預(yù)算花費(fèi)更多的時(shí)間或金錢。因此,在夏季,夜間修理主要道路可能省時(shí)省錢。

  下列哪一個(gè),假如正確,最支持上面得到的結(jié)論?

  A.修理京津塘高速公路的預(yù)算足夠用,所以不可能超過預(yù)算。

  B.夏季,夜間路上通行車輛較少,而且溫度較舒適,允許修路工人工作得更快。

  C.愿意在晚上工作的修路工人較容易找到工作機(jī)會,因?yàn)榇蠖鄶?shù)人寧愿白天工作。

  D.用于道路修理的瀝青在較高溫度下膨脹,在溫度降低的時(shí)候收縮。

  解析:補(bǔ)充論據(jù)。從題干可以看出從特殊樣本推出結(jié)論明顯論據(jù)不足。夏天夜間施工本來和省時(shí)省錢本來沒有關(guān)系,屬于不相關(guān)關(guān)系,因此需要補(bǔ)充適當(dāng)?shù)恼摀?jù),在這兩個(gè)問題中間搭起橋梁,建立關(guān)系,所以選擇B。

  13.偏頭痛一直被認(rèn)為是由食物過敏引起的。但是,如果讓患者停止食用那些已經(jīng)證明會不斷引起過敏性偏頭痛的食物,他們的偏頭痛并沒有停止,因此,顯然存在別的某種原因引起偏頭痛。下列哪項(xiàng)如果為真,最能削弱上面的結(jié)論?

  A.許多患者說誘發(fā)偏頭痛病的那些食物往往是他們最喜歡吃的食物。

  B.許多普通食物只在食用幾天后才誘發(fā)偏頭痛,因此,不容易觀察患者的過敏反應(yīng)和他們食用的食物之間的關(guān)系。

  C.很少有食物過敏會引起像偏頭痛那樣嚴(yán)重的癥狀。

  D.許多不患偏頭痛的人同樣有食物過敏反應(yīng)。

  解析:題目結(jié)論是過敏引發(fā)偏頭痛,但是引發(fā)過敏的問題不僅僅是食物引起的。結(jié)論指明食物過敏與偏頭痛之間不存在這因果關(guān)系,原因是停用了這種食物之后,偏頭痛的病情仍然存在,若要削弱這個(gè)結(jié)論,需要針對上述論據(jù)提出反對。本著這個(gè)思路我們看選項(xiàng),結(jié)論B。表明了停用食物之后仍然存在偏頭痛的原因是因?yàn)樵S多普通食物在食用幾天后才發(fā)作,因此雖然停用了食物,仍會發(fā)作一段時(shí)間。

  14.世界衛(wèi)生組織在全球范圍內(nèi)進(jìn)行了一項(xiàng)有關(guān)獻(xiàn)血對健康的影響的跟蹤調(diào)查。調(diào)查對象分為三組。第一組對象中均有二次以上的獻(xiàn)血記錄,其中最多的達(dá)數(shù)十次;第二組中的對象均僅有一次獻(xiàn)血記錄;第三組對象均從未獻(xiàn)過血。調(diào)查結(jié)果顯示,被調(diào)查對象中癌癥和心臟病的發(fā)病率,第一組分別為0.3%和0.5%,第二組分別為0.7%和0.9%,第三組分別為1.2%和2.7%。一些專家依此得出結(jié)論,獻(xiàn)血有利于減少患癌癥和心臟病的風(fēng)險(xiǎn)。這兩種病已經(jīng)不僅在發(fā)達(dá)國家而且在發(fā)展中國家成為威脅中老年人生命的主要?dú)⑹。因此,獻(xiàn)血利己利人,一舉兩得。以下哪項(xiàng)如果為真,將削弱以上結(jié)論?

  I 60歲以上的調(diào)查對象,在第一組中占60%,在第二中占70%,在第三組中占80%。

  II 獻(xiàn)血者在獻(xiàn)血前要經(jīng)過嚴(yán)格的體檢,一般具有較好的體質(zhì)。

  III 調(diào)查對象的人數(shù),第一組為1700人,第二組為3000人,第三組為7000人。

  A.只有I和II。    B.只有II和III。    C.只有I和III    D.只有I

  解析:題目中對獻(xiàn)血與癌癥、心臟病的發(fā)病幾率建立了正相關(guān)關(guān)系。但是樣本仍然存在問題,如果鮮血的人本身身體素質(zhì)就比沒有獻(xiàn)血的身體素質(zhì)好,就不能退出上述結(jié)論。這是最簡單容易想出的悖論。因此淘汰C、D。下面在條件1和3中選一個(gè),條件3說明樣本容量不同,但是題目中使用的是百分?jǐn)?shù)這個(gè)相對指標(biāo),因此不能形成反對結(jié)論的悖論。選A。

  15.許多孕婦都出現(xiàn)了維生素缺乏的癥狀,但這通常不是由于孕婦的飲食中缺乏維生素,而是由于腹內(nèi)嬰兒的生長使她們比其他人對維生素有更高的需求。為了評價(jià)上述結(jié)論的確切程度,以下哪項(xiàng)操作最為重要?

  A.對某個(gè)不缺乏維生素的孕婦的日常飲食進(jìn)行檢測,確定其中維生素的含量。

  B.對孕婦的科學(xué)食譜進(jìn)行研究,以確定有利于孕婦攝入足量維生素的最佳選擇。

  C.對日常飲食中維生素足量的一個(gè)孕婦和一個(gè)非孕婦進(jìn)行檢測,并分別確定她們是否缺乏維生素。

  D.對日常飲食中維生素不足量的一個(gè)孕婦和另一個(gè)非孕婦進(jìn)行檢測,并分別確定她們是否缺乏維生素。

  解析:支持論據(jù)。題目需要根據(jù)結(jié)論找出支持論據(jù),顯然對同量的維生素給孕婦和非孕婦服用,觀察他們是否缺乏維生素,對得出結(jié)論最重要。選C

  16.經(jīng)A省的防疫部門檢測,在該省境內(nèi)接受檢疫的長尾猴中,有1%感染上了狂犬病。但是只有與人及其寵物有接觸的長尾猴才接受檢疫。防疫部門的專家因此推測,該省長尾猴中感染有狂犬病的比例,將大大小于1%。

  以下哪項(xiàng)如果為真,將最有力地支持專家的推測?

  A.在A省境內(nèi),與人及其寵物有接觸的長尾猴,只占長尾猴總數(shù)的不到10%。

  B.在A省,感染有狂犬病的寵物,約占寵物總數(shù)的0.1%。

  C.在與A省毗鄰的B省境內(nèi),至今沒有關(guān)于長尾猴感染狂犬病的疫情報(bào)告。

  D.與健康的長尾猴相比,感染有狂犬病的長尾猴更愿意與人及其寵物接觸。

  解析:選擇D,其他選項(xiàng)均不能支持題干給出的結(jié)論。在與人和動物接觸的長尾猴進(jìn)行檢驗(yàn),發(fā)現(xiàn)1%感染了狂犬病,推斷出該省長尾猴感染狂犬病的比例小于1%,那么只有感染狂犬病的長尾猴更喜歡與人和動物接觸能夠有效地支持這個(gè)結(jié)論。

  17.作為市電視臺的攝像師,最近國內(nèi)電池市場的突然變化讓我非常頭疼。進(jìn)口電池缺貨,我只能用國產(chǎn)電池來代替作為攝像的主要電源。盡管每單位的國產(chǎn)電池要比進(jìn)口電池便宜,但我估計(jì)如果持續(xù)用國產(chǎn)電池替代進(jìn)口電池來提供同樣的電源供應(yīng)的話,我在能源上的支付將會提高。

  說這番話的人在上面這段話中隱含了以下哪項(xiàng)假設(shè)?

  A. 持續(xù)使用國產(chǎn)電池,攝像的質(zhì)量將無法得到保障。

  B. 每單位的進(jìn)口電池要比國產(chǎn)電池價(jià)格貴。

  C. 生產(chǎn)國產(chǎn)電池要比生產(chǎn)進(jìn)口電池成本低。

  D. 以每單位電池提供的電能來計(jì)算,國產(chǎn)電池要比進(jìn)口電池提供得少。

  解析:這是日常生活中經(jīng)常會遇到的問題,答案一目了然選擇D。得出結(jié)論的原因是盡管國產(chǎn)電池比進(jìn)口電池單價(jià)便宜,但是就單位電能來講,國產(chǎn)電池的價(jià)格卻比進(jìn)口電池高。

  18.最近由于氣候的異常變化,使得山東省的大蒜產(chǎn)量受到較大影響而減產(chǎn)。山東大蒜的價(jià)格比平時(shí)同期上漲了兩倍,這就大大提高了大蒜素生產(chǎn)的成本,估計(jì)未來大蒜素的市場價(jià)格將有大幅度的提高。以下哪項(xiàng)如果是真的,最能削弱上述結(jié)論?

  A.去年大蒜的價(jià)格是歷年最低的。

  B.其他替代原料可以用來生產(chǎn)人工大蒜素。

  C.除了山東省外,其他省份也可以提供大蒜。

  D.最近的天氣異常不如專家們估計(jì)的那么嚴(yán)重。

  解析:題目中的推斷是沒有問題的,成本的提高會促使價(jià)格的提高,但是其他省也提供大蒜,而成本并沒有提高,價(jià)格不會上升,在這樣的市場上,山東省的大蒜盡管成本提高了,但是價(jià)格將會保持市場水平,因此C有效地消弱了結(jié)論。

  19.某學(xué)校開設(shè)選修課,規(guī)定:同學(xué)只有選了文化史課的同學(xué)才能選古典音樂課。如果選古典音樂課,那他就不能選文學(xué)欣賞課。如果選文學(xué)欣賞課,就不能選古典音樂課。

  如果以上陳述為真,以下哪項(xiàng)陳述不可能假?

  A.同學(xué)不選文學(xué)欣賞課 或者不選古典音樂課。

  B.同學(xué)選文學(xué)欣賞課 或者不選古典音樂課。

  C.同學(xué)不選古典音樂課,或者不選文化史課。

  D.同學(xué)選古典音樂課,或者不選文學(xué)欣賞課

  解析:邏輯推斷題。不可能假=一定真,因此題目要求在選項(xiàng)中找出一定真的項(xiàng)。選了文化史,才能選古典音樂,這是個(gè)必要條件;但是選了古典音樂就不能選文學(xué)欣賞,二選其一;選項(xiàng)A的論述與題干一樣。

  20.20年前,幾乎所有的公司首腦在選擇重新設(shè)置公司總部的時(shí)候,主要關(guān)心的是土地價(jià)格。今天一個(gè)高級執(zhí)行官計(jì)劃重設(shè)總部時(shí)主要關(guān)心的東西更廣泛了,經(jīng)常包括當(dāng)?shù)氐膶W(xué)校和住房質(zhì)量。

  假如上述信息可靠,下面哪一項(xiàng)最好地解釋了公司首腦們主要關(guān)心問題的變化?

  A.20年前高質(zhì)量的住房和學(xué)校像今天一樣難以發(fā)現(xiàn)。

  B.近年來優(yōu)秀專業(yè)辦公人員缺乏的問題迫使公司找到盡可能多的方法來留住老員工,并且吸引優(yōu)秀新員工的加盟。

  C.公司執(zhí)行官總是考慮自己的決定將怎樣影響公司的利潤。

  D.在過去20年,一些地區(qū)比其他地區(qū)土地價(jià)格變化少。

  解析:選項(xiàng)A屬于強(qiáng)拉因果,不存在相關(guān)關(guān)系;選項(xiàng)B說明了高級執(zhí)行官在公司選址時(shí)關(guān)注學(xué)校和住房質(zhì)量的原因,分別是吸引優(yōu)秀新員工加盟和留住老員工;選項(xiàng)C與題干沒有關(guān)系,并未涉及到利潤這個(gè)因素;選項(xiàng)D,最多可以說明高級執(zhí)行官不在關(guān)心土地價(jià)格的原因,但是并不能說明關(guān)注學(xué)校和住房質(zhì)量的原因。

21.用蒸餾麥芽渣提取的酒精作為汽油的替代品進(jìn)入市場,使得糧食市場和能源市場發(fā)生了前所未有的直接聯(lián)系。到1995年,谷物作為酒精的價(jià)值己經(jīng)超過了作為糧食的價(jià)值。西方國家已經(jīng)或正在考慮用從谷物提取的酒精來替代一部分進(jìn)口石油。

  如果上述斷定為真,對于那些己經(jīng)用從谷物提取的酒精來替代一部分進(jìn)口石油的西方國家。以下哪項(xiàng),最可能是1995年后進(jìn)口石油價(jià)格下跌的后果?

  A.一些谷物從能源市場轉(zhuǎn)入糧食市場。

  B.一些谷物從糧食市場轉(zhuǎn)入能源市場。

  C.谷物的價(jià)格面臨下跌的壓力。

  D.谷物的價(jià)格出現(xiàn)上浮。

  解析:谷物可以提取出酒精來代替石油出口,因此可以判斷谷物和石油成為了相互競爭的替代品,因此如果石油價(jià)格下跌,那么谷物的價(jià)格也必然會下調(diào),否則進(jìn)口國將會進(jìn)口價(jià)格低廉的石油取代谷物。

  22.有八個(gè)教授參加學(xué)術(shù)會議后,互留通信地址,以保持聯(lián)絡(luò)。一年后,統(tǒng)計(jì)他們之間的通信情況如下:有一人給其他三個(gè)人寫過信,給其他人寫過兩封信的有三人,給其他人寫過一封信的有四個(gè)人。若以上統(tǒng)計(jì)屬實(shí),則最能得出以下哪項(xiàng)結(jié)論?

  A.通過書信來往,他們遞進(jìn)了友誼,也加強(qiáng)了學(xué)術(shù)聯(lián)系。

  B.八個(gè)教授都收到過其他教授的來信。

  C.至少有一個(gè)人收到信件后沒有都回復(fù)。

  D.雖然參加學(xué)術(shù)會議的時(shí)候大家不熟悉,但現(xiàn)在他們已經(jīng)成為至交。

  解析:選項(xiàng)首先排除A、D,沒有任何證據(jù)表明通過書信加強(qiáng)了學(xué)術(shù)聯(lián)系,也沒有證據(jù)表明他們成為了至交。下面看選項(xiàng)B、C,我們可以通過畫圖的方法來解決這個(gè)問題相對比較簡單,從而得出正確答案為C。

  23.金鳳扒雞是一個(gè)享有盛譽(yù)的百年扒雞品牌,在石家莊銷路很好,深受當(dāng)?shù)匕傩障矏郏刻煲簧鲜芯蜁粨屬徱豢。為了打開北京市場,金鳳扒雞改進(jìn)了工藝,用真空包裝,延長了保質(zhì)期,試圖在北京各大超市打開銷路,但是金鳳扒雞在北京卻很難銷售出去。

  下面除了哪一項(xiàng),都有助于解決上述現(xiàn)象?

  A.北京人和石家莊人的飲食習(xí)慣不同。他們更喜歡吃烤鴨而不是扒雞。

  B.恒慧通燒雞也采用真空包裝,保質(zhì)期一樣長,在北京的銷路卻很好。

  C.人們喜歡吃剛出鍋的新鮮扒雞,保質(zhì)期短的反而更受消費(fèi)者青睞。

  D.雖然金鳳扒雞是百年品牌,但是它的銷售地點(diǎn)僅在石家莊市區(qū),北京人對這個(gè)品牌不熟悉。

  解析:這是一道頗具爭議的題目,選項(xiàng)A、B、C、D分別從消費(fèi)者的喜好,競爭者,消費(fèi)偏好和品牌對金鳳扒雞在北京銷路受阻提供了良好的解釋。但是選項(xiàng)B可以從另外一個(gè)角度思考,恒惠通燒雞同樣采用真空包裝,保質(zhì)期一樣長,什么都和金鳳扒雞一樣,為什么銷路很好,而金鳳扒雞為什么銷路不好,這就不能解釋為什么金鳳扒雞在北京銷路不好的原因。

  24.中國自1978年改革開放以來28年間,從人口數(shù)量上說:農(nóng)民的數(shù)量并沒有減少,而是持續(xù)增加的,只不過這種增長速度相對于總?cè)丝诘脑鲩L速度來說,是比較低的。

  下列那一句話直接與上述信息矛盾?

  A.在總?cè)丝谥修r(nóng)民的數(shù)量在1978年到2006年間略微增長了。

  B.農(nóng)民在總?cè)丝谥械谋嚷蕪?978年的81.4%增加到了2006年的85.3%。

  C.中國勞動力的增長率和總?cè)丝诘脑鲩L率在1978年至今的28年里同時(shí)增加了。

  D.中國的勞動力中,農(nóng)民的比率從1978年的78%下降到了75.6%。

  解析:選項(xiàng)B可以看出,農(nóng)民相對總?cè)丝诘脑鲩L速度并不低,增長速度也是增加的。

  25.有三戶人家,每家有一孩子,他們的名字是:小萍(女)、小紅(女)、小虎。孩子的爸爸是老王、老張和老陳;媽媽是劉蓉、李玲和方麗。對于這三家人,已知:

  (1)老王家和李玲家的孩子都參加了少女舞蹈隊(duì)。

 。2)老張的女兒不是小紅。

  (3)老陳和方麗不是一家人。

  根據(jù)以上條件,確定以下哪項(xiàng)是正確的?

  A.老王、劉蓉和小萍是一家。

  B.老張、李玲和小紅是一家。

  C.老王、方麗和小紅是一家。

  D.老陳、方麗和小虎是一家。

  解析:邏輯推斷題。從條件1可以看出小虎不是老王和李玲的孩子,因?yàn)樾』⑹悄泻。那么可以推出小萍和小紅的爸爸可能是老王。根據(jù)條件2,老張的女兒不是小紅,所以可以確定老王的女兒是小萍。鎖定選項(xiàng)A,如果A正確,那么根據(jù)條件3老陳和李玲是一家,根據(jù)條件1,女兒是小紅;得出老陳、方麗和小虎是一家。那么選項(xiàng)D也正確。因此同時(shí)排除A、D。選項(xiàng)B與條件2不符,因此選擇C。

  26.5名犯罪嫌疑人被警察詢問,事后得知其中一名是罪犯。下面是五個(gè)嫌疑人的供述,其中只有3句真話。問,誰是罪犯?

  甲:丁是罪犯。

  乙:我是無辜的。

  丙:戊不是罪犯。

  。杭自谌鲋e。

  戊:乙說的是實(shí)話。

  A.戊    B.甲和丙    C.丁    D. 乙和戊

  解析:從選項(xiàng)入手,選項(xiàng)B,甲和丙是罪犯,可以推出乙、戊、丁、丙都為真,而題目表明只有3項(xiàng)為真,所以放棄B;選項(xiàng)C,丁是罪犯,可以推出甲、乙、丙、戊為真,同樣與題目要求不符;選項(xiàng)D,乙和戊是罪犯,只能推出丁為真。因此這道題選擇A。

  27.學(xué)生上完體育課后回到教室,有15人喝了飲水機(jī)里的純凈水,其中5人很快產(chǎn)生了腹瀉。飲水機(jī)里的純凈水馬上被送去檢驗(yàn),檢驗(yàn)的結(jié)果不能肯定其中有造成腹瀉的有害物質(zhì)。因此,喝了飲水機(jī)里的純凈水不是造成腹瀉的原因。

  如果上述檢驗(yàn)結(jié)果是正確的,則以下哪項(xiàng)對上述論證的評價(jià)最為恰當(dāng)?

  A.題干的論證有漏洞,因?yàn)樗鼪]有考慮到另一個(gè)事實(shí):哪些沒有喝了飲水機(jī)里的純凈水的人沒有造成腹瀉。

  B.題干的論證有漏洞,因?yàn)樗讶鄙僮C據(jù)證明某種情況存在,當(dāng)作有充分證據(jù)證明某種情況不存在。

  C.題干的論證有漏洞,因?yàn)樗鼪]有利用一個(gè)有力的論據(jù):為什么有更多人喝了飲水機(jī)里的純凈水沒有造成腹瀉。

  D.題干的論證有漏洞,因?yàn)樗鼪]有指出造成腹瀉的真正原因。

  解析:理解這道題非常簡單,舉個(gè)例子說明,如果一個(gè)人確實(shí)殺了人,但是法官沒有證據(jù)表明他是兇手,那么可能無法定罪,但終不能得出他不是兇手的結(jié)論,因?yàn)橥瑯記]有證據(jù)表明他不是兇手。題目中給出了同樣的問題,檢驗(yàn)結(jié)果不能肯定有造成腹瀉的有害物質(zhì),但是并不能說明純凈水不是造成腹瀉的原因。

  28.一項(xiàng)全球范圍的調(diào)查顯示,近 10年來:吸煙者的總數(shù)基本保持不變;每年只有10%的吸煙者改變自己的品牌,即放棄原有的品牌而改吸其他品牌;煙草制造商用在廣告上的支出占其毛收入的10%。在Z煙草公司的年終董事會上,懂事A認(rèn)為,上述統(tǒng)計(jì)表明,煙草業(yè)在廣告上的收益正好等于其支出,因此,此類廣告完全可以不做。以下哪項(xiàng),構(gòu)成對懂事A的結(jié)論的最有力質(zhì)疑?

  A.懂事A的結(jié)論忽視了:今年來各種品牌的香煙的價(jià)格都有了很大的變動。

  B.懂事A的結(jié)論基于一個(gè)錯(cuò)誤的假設(shè):每個(gè)吸煙者在某個(gè)時(shí)候只喜歡一種品牌。

  C.懂事A的結(jié)論基于一個(gè)錯(cuò)誤的假設(shè):每個(gè)煙草制造商只生產(chǎn)一種品牌。

  D.懂事A的結(jié)論忽視了:世界煙草業(yè)是一個(gè)由處于競爭狀態(tài)的眾多經(jīng)濟(jì)實(shí)體組成的。

  解析:董事A的判斷為廣告的支出正好與收益相等,因此得出廣告可以不做的結(jié)論。但是忽略了如果不做廣告是否能夠獲得與做廣告時(shí)除去廣告費(fèi)用后相同的收益,因?yàn)檫@部分收入也是由廣告產(chǎn)生的效益。因?yàn)橥袠I(yè)的其他競爭者會因?yàn)閺V告,增加10%的顧客,而不做廣告的煙草公司將有減少10%顧客的風(fēng)險(xiǎn)。因此選擇D

  29.在收款局的5個(gè)賬單收款員中,楊先生收款的不成功率最高。然而楊先生是這個(gè)局的職員中最好的賬單收款員。

  下面哪一項(xiàng),如果正確,最有助于解決上述短文中的明顯分歧?

  A.在加入收款局之前,楊先生是一個(gè)大百貨公司的信貸部的一名職員。

  B.收款局中的其他四個(gè)收款員都認(rèn)為楊先生是一個(gè)非常能干的帳單收款員。

  C.楊先生在過去的幾年內(nèi),每年收款成功的比率都保持相當(dāng)?shù)胤(wěn)定。

  D.這個(gè)收款局的大多數(shù)最困難的事情都是派楊先生去做的。

  解析:從題干入手分析,楊先生是最好的賬單收款員,但是收款不成功率最高。題目要求找到解釋這一矛盾的原因。顯然D最合適。

  30.在一次工程碩士入學(xué)考試中,據(jù)統(tǒng)計(jì),在英語、數(shù)學(xué)、語文和邏輯四個(gè)科目中:70%的考生英語及格了,75%的考生數(shù)學(xué)及格了,80%的考生語文及格了,85%的考生邏輯及格了。如果以上情況屬實(shí),那么,以下哪個(gè)選項(xiàng)一定正確:

  A.在此次考試中,大部分考生四門課都及格了。

  B.在此次考試中,大部分考生四門課沒有都及格。

  C.在此次考試中,有不少于10%的考生,四門課程都及格了。

  D.在此次考試中,至少有10%的考生,四門課程都沒及格。

  解析:不少于=大于等于,至少=大于等于。選線排除A、B,選項(xiàng)過于籠統(tǒng),并且與C、D存在重復(fù)關(guān)系。選項(xiàng)D與題干明顯不符合,選擇C。

 31.某游戲的游戲規(guī)則有這樣一條規(guī)定:任何在游戲中拒絕成為選手的人將在拒絕的時(shí)候被減掉10分。

  下列哪一個(gè)是此條規(guī)則所暗含的?

  A.同意參加游戲的所有人的得分將高于那些因?yàn)榇艘?guī)則而被判罰分的人。

  B.一個(gè)人在拒絕玩這個(gè)游戲的同時(shí),依然是這個(gè)游戲的參與者。

  C.最初同意成為游戲參加者然后在游戲進(jìn)行中退出的人可以避免被罰10分。

  D.一個(gè)拒絕玩這個(gè)游戲的人不能在游戲中被宣布為失敗者。

  解析:由于拒絕玩這個(gè)游戲,會被剪掉20分,說明依舊是游戲的參與者,選B。

  32.受到廣泛贊揚(yáng)的斯坦尼斯拉夫斯基演員訓(xùn)練法來自他做年輕演員時(shí)對戲劇不熟練和易受影響的老套做法。對該方法的理解必須基于斯坦尼斯拉夫斯基個(gè)人的演藝研究,從而從典型化的姿勢、反復(fù)嘗試的嗓音語調(diào)和標(biāo)準(zhǔn)的感情模式的誘惑中解脫出來。盡管他的美國門徒有過要求,但是這位俄國導(dǎo)演從未打算寫一本對表演問題提出嚴(yán)格解決方法的教科書。

  關(guān)于表演這個(gè)問題,上文作者對斯坦尼斯拉夫斯基的演員訓(xùn)練法持有怎樣的觀點(diǎn)?

  A.斯坦尼斯拉夫斯基的美國門徒,在使用他的方法訓(xùn)練時(shí),使得這種方法喪失了原有的靈活性和探索性。

  B.表演本質(zhì)上是自發(fā)的感情表達(dá),系統(tǒng)的訓(xùn)練經(jīng)常干涉它。

  C.實(shí)際上唯一需要給予年輕演員的建議是他們必須系統(tǒng)地抵制他們在戲劇表演中所表現(xiàn)出來的那些老套的模式。

  D.斯坦尼斯拉夫斯基的方法主要是針對那些必須克服有做作和不成熟表演行為的年輕演員。

  解析:斯坦尼斯拉夫斯基之所以未打算對表演問題提出嚴(yán)格解決方法的教科書,本意是不想使該方法失去靈活性和探索性。因此選擇A。

  33.已發(fā)現(xiàn)有國產(chǎn)嬰幼兒奶粉三聚氰胺含量超標(biāo)。

  如果上述斷定是真的,那么在下述三個(gè)斷定中不能確定真假的是:

 、、國產(chǎn)嬰幼兒奶粉沒有三聚氰胺含量不超標(biāo)的。

  Ⅱ、有國產(chǎn)嬰幼兒奶粉三聚氰胺含量沒超標(biāo)。

 、蟆⑺袊a(chǎn)嬰幼兒奶粉三聚氰胺含量都未超標(biāo)。

  A.只有Ⅰ和Ⅱ。    B.Ⅰ、Ⅱ和Ⅲ。    C.只有Ⅰ和Ⅲ。    D.只有Ⅱ。

  解析:選項(xiàng)1不能確定真假,從部分不能推出整體;選項(xiàng)2不能確定真假,從一部分不能退出另一部分具有相反的結(jié)論。條件3明顯可以判定為假,選擇A。

  34.股民購買股票的目的就是盈利。股民購買股票后,股票價(jià)格上漲,就會盈利,否則,就會虧損。一般情況下,盈利或者虧損達(dá)到10%就應(yīng)該拋出,以避免那些不可預(yù)測的突然變化所引起的損失。在股市上,有經(jīng)驗(yàn)的股民往往會選擇跟莊戰(zhàn)術(shù)來買賣股票。即大股東買進(jìn)的時(shí)候,他們也買進(jìn),大股東拋出的時(shí)候,他們就拋出。因?yàn)榇蠊蓶|有可能掌握更多上市公司和市場行情方面的信息。這些天,天虹公司的股票一直在下跌,但是大股東們卻一直在大量購進(jìn)這只股票。

  以上事實(shí)最能支持以下哪種預(yù)測?

  A.天虹公司的股票價(jià)格會繼續(xù)下降,但速度會放慢。

  B.天虹公司的股票價(jià)格會在短期內(nèi)大幅度上漲。

  C.股民盲目跟莊不是一個(gè)好的盈利方法。

  D.最近大股東對天虹公司股價(jià)變動判斷失誤,有可能造成巨大經(jīng)濟(jì)損失。

  解析:題目結(jié)論如下:大股東買進(jìn)的股票會漲,因?yàn)樗麄冋莆樟烁嗌鲜泄竞褪袌鲂星榉矫娴男畔ⅰK赃x擇B。

  35.作為身體的精密平衡系統(tǒng)的一部分,人類心臟分泌某種激素,這些激素能夠控制血液中的含鹽量和體內(nèi)參與循環(huán)的血量。人體僅需要極少的這種激素。它在控制血壓時(shí)極其重要,并且在患心臟病的病人血中經(jīng)常檢出。

  基于上面的研究成果,下面的哪個(gè)論點(diǎn)一定正確?

  A.假如心臟分泌的這種激素缺乏,將導(dǎo)致低血壓。

  B.一個(gè)僅僅由機(jī)械閥制作的設(shè)備被用作人工心臟,它將不能執(zhí)行人類心臟的所有功能。

  C.少量這種激素的分泌對人體的影響是長期性的。

  D.任何控制血壓的藥將通過影響心臟此種激素分泌量達(dá)到效果。

  解析:選擇B,因?yàn)橛蓹C(jī)械閥制作的人工心臟,顯然不具有分泌激素的功能,因此就不能執(zhí)行人類心臟的所有功能,題干反復(fù)說明就是為了得出這樣的結(jié)論。

  36.某出版社近年來出版物的錯(cuò)字率較前幾年有明顯的增加,引起了讀者的不滿和有關(guān)部門的批評,這主要是由于該出版社大量引進(jìn)非專業(yè)編輯所致。當(dāng)然,近年來該社出版物的大量增加也是一個(gè)重要原因。

  上述議論的漏洞,也類似地出現(xiàn)在以下哪項(xiàng)中?

  I 美國航空公司近兩年來的投訴率比前幾年有明顯的下降。這主要是由于該航空公司在裁員整頓的基礎(chǔ)上,有效地提高了服務(wù)質(zhì)量。當(dāng)然,“9.11”事件后航班乘客數(shù)量的銳減也是一個(gè)重要原因。

  II 統(tǒng)計(jì)數(shù)字表明:近年來我國心血管病的死亡率,即由心血管病導(dǎo)致的死亡率在整個(gè)死亡人數(shù)中的比例,較前有明顯增加,這主要是由于隨著經(jīng)濟(jì)的發(fā)展,我國民眾的飲食結(jié)構(gòu)和生活方式發(fā)生了容易誘發(fā)心血管病的不良變化。當(dāng)然,由于心血管病主要是老年病,因此,我國人口的老齡化,即人口中老年人比例的增加也是一個(gè)重要原因。

  III 某市去年的高考錄取率比前年增加15%,這主要是由于各中學(xué)狠抓了教育質(zhì)量。當(dāng)然,另一個(gè)重要原因是,該市今年參加高考的人數(shù)比去年增加了20%。

  A.僅僅I和II。   B.僅僅II和III。   C.僅僅I和III。   D.僅僅I。

  解析:題目中的邏輯錯(cuò)誤是強(qiáng)加因果,出版物數(shù)量的增加與錯(cuò)字率的增加沒有因果關(guān)系。根據(jù)這一思路對選項(xiàng)進(jìn)行判斷,選項(xiàng)I航班數(shù)銳減與投訴率的下降沒有因果關(guān)系,與題目中犯了同樣的錯(cuò)誤;選項(xiàng)II,心血管病是老年病,我國人口老齡化,因此我國心血管死亡率增加,這是一個(gè)有效地推理過程;選項(xiàng)III,參加高考的人數(shù)增加,并不能推出錄取比例增加的這一結(jié)論。也是屬于沒有因果關(guān)系。另外從絕對和相對概念這個(gè)角度分析,同樣可以得到相似的結(jié)論。選C

  37.從前,一個(gè)孤島上有一個(gè)奇怪的風(fēng)俗:凡是漂流到這個(gè)島上的外鄉(xiāng)人都要作為祭品被殺掉,但允許被殺的人在臨死前說一句話,然后由這個(gè)島上的長老判定這句話是真的還是假的。如果說的是真話,則將這個(gè)外鄉(xiāng)人在真理之神面前殺掉;如果說的是假話,則將他在錯(cuò)誤之神面前殺掉。有一天,一位哲學(xué)家漂流到了這個(gè)島上,他說了一句話,使得島上的人沒有辦法殺掉他。

  該哲學(xué)家必定說了下面哪一句話?

  A.你們這樣做不合乎理性。    B.我將死在真理之神面前。

  C.無論如何我都會死。      D.我將死在錯(cuò)誤之神面前。

  解析:A選項(xiàng),可以判斷為真,會在真理之神面前殺掉;B選項(xiàng),可以判定為真,因?yàn)樗麜涝谡胬碇衩媲埃贿x項(xiàng)C,同樣可以判定為真,選項(xiàng)D我將死在錯(cuò)誤之神面前,如果為真,那么說明他說的是假話,表明他將死在真理之神面前為真,這本身與這句話矛盾了,所以這句話自背反命題,選D。

  38.有的地質(zhì)學(xué)家認(rèn)為,如果地球的未勘探地區(qū)中單位面積的平均石油儲藏量能和已勘探地區(qū)一樣的話,那么,目前關(guān)于地下未開采的能源含量的正確估計(jì)因此要乘上一萬倍,由此可得出結(jié)論,全球的石油需求,至少可以在未來五個(gè)世紀(jì)中得到滿足,即便此種需求每年呈加速上升的趨勢。為使上述論證成立,以下哪項(xiàng)是必須假設(shè)的?

  A. 地球上未勘探地區(qū)的總面積是已勘探地區(qū)的一萬倍。

  B. 在未來至少五個(gè)世紀(jì)中,世界人口的增長率不會超過對石油需求的增長率。

  C. 新技術(shù)將使未來對石油的勘探和開采比現(xiàn)在更為可行。

  D. 地球上未勘探地區(qū)中儲藏的石油可以被勘測和開采出來。

  解析:答案為D,題目中通過假設(shè)得出了未來五個(gè)世紀(jì)的石油需求相會得到滿足的結(jié)論,假設(shè)條件為地球上未勘探地區(qū)的單位面積石油儲量與已勘探地區(qū)一樣多。但僅有這個(gè)前提還是不夠的,還需要這些為勘探的時(shí)候都是可以被開采出來的這一假設(shè)。

  39.某礦山發(fā)生了一起嚴(yán)重的安全事故。關(guān)于事故原因,甲乙丙丁四位負(fù)責(zé)人有如下斷定:

  甲:如果造成事故的直接原因是設(shè)備故障,那么肯定有人違反操作規(guī)程。

  乙:確實(shí)有人違反操作規(guī)程,但造成事故的直接原因不是設(shè)備故障

  丙:造成事故的直接原因確實(shí)是設(shè)備故障,但并沒有人違反操作規(guī)程。

  。涸斐墒鹿实闹苯釉蚴窃O(shè)備故障。

  如果上述斷定只有一個(gè)人的斷定為真,那么以下斷定都不可能為真,除了:

  A.甲的斷定為真,有人違反了操作規(guī)程。

  B.甲的斷定為真,但沒有人違反操作規(guī)程。

  C.乙的斷定為真。

  D.丙的斷定為真。

  解析:這道題屬于邏輯推理題,由于只有一個(gè)人判斷為真,因此丙、丁的判斷首先排除,因?yàn)檫@兩個(gè)判定并不沖突。甲、乙的推斷有矛盾,因此兩項(xiàng)必有一個(gè)是正確的。甲的推斷結(jié)構(gòu)是如果B推出A,那么 C必然推出B;乙的推斷結(jié)構(gòu)是C推出B,但B不能退出A,是甲判定的一個(gè)逆向命題。選項(xiàng)B是正確答案,因?yàn)檫x項(xiàng)B肯定了甲的推斷,同時(shí)否定了乙的發(fā)生,因此是真命題。其他選項(xiàng)均不正確。

  40.心臟的搏動引起血液循環(huán)。對同一個(gè)人,心率越快,單位時(shí)間進(jìn)入循環(huán)的血液量越多。血液中的紅血球運(yùn)輸氧氣。一般地說,一個(gè)人單位時(shí)間通過血液循環(huán)獲得的氧氣越多,他的體能及其發(fā)揮就越佳。因此,為了提高運(yùn)動員在體育比賽中的競技水平,應(yīng)該加強(qiáng)他們在高海拔地區(qū)的訓(xùn)練,因?yàn)樵诟吆0蔚貐^(qū),人體內(nèi)每單位體積血液中含有的紅血球數(shù)量,要高于在低海拔地區(qū)。

  以下哪項(xiàng)是題干的論證必須假設(shè)的?

  A.運(yùn)動員在高海拔地區(qū)的心率不低于在低海拔地區(qū)。

  B.不同運(yùn)動員的心率基本相同。

  C.運(yùn)動員的心率比普通人慢。

  D.在高海拔地區(qū)訓(xùn)練能使運(yùn)動員的心率加快。

  解析:此題論證說明運(yùn)動員在高海拔地區(qū)訓(xùn)練體內(nèi)的紅血球數(shù)量高于低海拔地區(qū),從而達(dá)到最佳體能的目的。B和C與題意不符,選A。

 41~45題基于以下題干:

  七個(gè)學(xué)生R、S、T、V、W、X、Y,被分成兩個(gè)學(xué)習(xí)小組。第一組有三名成員,第二組有四名成員。學(xué)生們的分組必須符合以下要求:

  R和T不能在同一個(gè)小組;

  如果S在第一組,那么V必須在第一組;

  如果W在第一組,那么T必須在第二組;

  X必須在第二組。

  41.如果W在第一組,那么以下哪項(xiàng)也一定在第一組?

  A.R    B.S    C.T    D.V

  解析:根據(jù)條件第一組有三名成員。因?yàn)镽和T不能在一組,又因?yàn)闂l件:如果W在第一組,那么T必須在第二組,則R可以在第一組

  42.如果T和Y都在第一組,那么以下哪項(xiàng)一定是真的?

  A.S和V在同一組。   B.S和W在同一組。   C.V和R在同一組。   D.W和T在同一組。

  解析:如果T和Y在第一組,R和T又不能在一組,可知R只能在第二組,而X必須在第二組。如果S在第一組,根據(jù)條件,V也需在第一組,而第一組只有三個(gè)成員,那么S和V就不能在同一組。而如果W在第一組,那么T必須在第二組。但是T已經(jīng)在第一組,則W必須在第二組,加之第二組有四人,那么S也需在第二組,那么S和W為一組。

  43.如果W和T在同一組,那么以下哪項(xiàng)可能在同一組,除了:

  A.R和S    B.S和Y    C.T和Y    D.V和Y

  答案:B

  解析:

  條件分析:

  1、R1且T2;或者 R2且T1

  2、S1→V1;即S2的時(shí)候,V可以自由分配。

  3、W1→T2;即W2的時(shí)候,T可以自由分配。

  4、X2

  43題的臨時(shí)題干要求WT在同一組,根據(jù)條件3,他們不可能在第一組,不然就違背了題設(shè)。所以,這個(gè)時(shí)候,WT都在都在第二組。第二組已經(jīng)有了WTX三個(gè)成員,只有一個(gè)空位了。根據(jù)條件1,可以知道,R在第一組。因此,除了跟WTX搭伴的元素外,任何二人組合都必須在第一組了。

  選項(xiàng)A,第一組RSV;第二組WTXY,有可能。

  選項(xiàng)B,第一組RSY;則第一組還必須有V,顯然超編了。因?yàn)榈谝唤M只能有三個(gè)人。因此,此項(xiàng)為答案。

  選項(xiàng)C,依然是第一組RSV;第二組WTXY這樣的組合。

  選項(xiàng)D,第一組RVY;第二組WTXS,不矛盾。

  44.如果V和Y在同一組,那么以下哪項(xiàng)一定是真的?

  A.R在第一組。    B.S在第一組。    C.W在第二組。    D.Y在第二組。

  解析:如果V和Y在第一組,那么根據(jù)條件如果S在第一組,那么V必須在第一組;說明S可以在第一組,又因?yàn)槿绻鸚在第一組,那么T必須在第二組,但是R和T又不能在一組,則R必須在第一組,W和T可以在第二組。則答案C正確。

  45.如果S在第一組,那么以下哪項(xiàng)一定是真的?

  A.T在第一組。    B.T在第二組。    C.Y在第一組。    D.Y在第二組。

  解析:如果S在第一組,根據(jù)條件V必須在第一組,如果W在第一組,那么T必須在第二組;R和T不能在同一個(gè)小組;那么W或R可以在第一組,X必須在第二組。因此在第一組的可以是S或V或R或W,因此答案D正確。

  46~50題基于以下題干:

  六位教授F、G、H、J、K、L,將評審馬、任、孫、吳博士的論文4篇。評審需遵守以下原則:

 。1)每位教授只評審一篇博士論文;

 。2)每篇博士論文至少有一位教授評審;

 。3)H與F評審?fù)黄┦空撐模?/p>

 。4)L只與其他教授中的一位同評一篇博士論文;

  (5)G評審馬博士的論文;

  (6)J評審馬博士或吳博士的論文;

  (7)H不評審吳博士的論文。

  46.如果K不評審孫博士的論文,那么以下哪項(xiàng)一定是真的?

  A.L評審馬博士的論文           B.L評審孫博士的論文

  C.F和H評審任博士的論文        D.F和H評審孫博士的論文

  解析:H不評審吳博士論文,H與F又評審?fù)黄撐;那么H和F有可能評審馬或任或?qū)O博士的論文,但是G評審馬博士的論文,J評審馬博士或者吳博士的論文,那么H與F就有可能評審任或?qū)O博士的論文,K不評審孫博士的論文,K就有可能評審任博士的論文,那么H和F就會評審孫博士的論文。

  47.以下哪項(xiàng)完整地列出了L可能評審的博士論文?

  A.馬、任      B.馬、孫、吳     C.馬、任、吳       D.馬、任、孫、吳

  解析:根據(jù)條件四,L只與其他教授中的一位同評一篇博士論文,而每篇博士論文至少有一位教授評審,也就是說一篇論文有可能是兩個(gè)教授或者三個(gè)教授一起評論。因此,L可以評論馬、任、孫、吳的博士論文。

  48.以下哪項(xiàng)可能是真的?

  A.F和G評審馬博士的論文。

  B. F和L評審吳博士的論文。

  C. K評審吳博士的論文并且L評審馬博士的論文。

  D. L評審任博士的論文并且F評審孫博士的論文。

  解析:G評審馬博士的論文,J評審馬博士或吳博士的論文,H不評審吳博士的論文,H與F又評審?fù)黄撐;那么H和F有可能評審馬或任或?qū)O博士的論文,那么H與F就有可能評審任或?qū)O博士的論文。加之題一,K不評審孫博士的論文,K就有可能評審任博士的論文,條件三H與F評審?fù)黄┦空撐,那么H和F就會評審孫博士的論文。條件四:L只與其他教授中的一位同評一篇博士論文,L可以與K評審任博士論文,故D正確

  49.以下哪項(xiàng)不可能是真的?

  A. L和G評審馬博士的論文        B. L和K評審馬博士的論文

  C. L和K評審任博士的論文        D. L和K評審孫博士的論文

  解析:根據(jù)條件(4)L只與其他教授中的一位同評一篇博士論文,而條件(5)G評審馬博士的論文,那么可知L和G可評審馬博士論文,故A正確。根據(jù)條件(6)J評審馬博士或吳博士的論文,可以得知L可以和J可以評審吳博士或者馬博士的論文,H不評審吳博士的論文,那么H可以評論任博士和孫博士的論文。K可以評論吳博士、任博士、孫博士的論文。加之條件(4),得知答案C和D正確,故B錯(cuò)誤

  50.如下哪位教授不可能審閱任博士的論文?

  A.F   B.K    C.J   D. L

  解析:條件(6)J評審馬博士或吳博士的論文;而條件(1)每位教授只評審一篇博士論文,根據(jù)兩個(gè)條件可以推斷J不可能審閱任或?qū)O博士的論文,故選C

  模擬題1參考答案:

01. C 02. A 03. C 04. D 05. C
06. C 07. A 08. B 09. A 10. A
11. A 12. B 13. B 14. A 15. C
16. D 17. D 18. C 19. A 20. B
21. C 22. C 23. B 24. B 25. C
26. A 27. B 28. D 29. D 30. C
31. B 32. A 33. A 34. B 35. B
36. C 37. D 38. D 39. B 40. A
41. A 42. B 43. B 44. C 45. D
46. D 47. D 48. D 49. B 50. C
 

 

 

(50題,每題2分,滿分100分)

  1.任何人都沒有吃過雅各島上的任何水果,所以無法知道雅各島上任何水果的口味。

  為了合乎邏輯的推出上述結(jié)論,需要假設(shè)下面哪項(xiàng)為前提?

  A.如果一種水果有人品嘗過,就可以知道其口味。(充分條件,不是前提)

  B.只憑某些人的品嘗無法真正知道某種水果的口味。

  C.要知道某種水果的口味,需要有人去品嘗。

  D.人們是通過嗅覺來確定水果口味的。

  解析:這道題需要補(bǔ)全假設(shè),結(jié)論是:任何人都無法知道任何水果的味道,前提是:任何人都沒有吃過雅各島上的任何水果。補(bǔ)充前提是:要知道水果的口味,需要有人去品嘗,應(yīng)該選擇C。選項(xiàng)A,是充分條件,但不是構(gòu)成結(jié)論的原因,屬于強(qiáng)拉因果;B、D都為不合理的假設(shè)。

  2.最近的一項(xiàng)研究指出:“經(jīng)常吃沙棘果對兒童的智力發(fā)育有益。”研究人員對560名兒童進(jìn)行調(diào)查,發(fā)現(xiàn)那些經(jīng)常吃沙棘果的兒童,其智力水平較很少吃沙棘果的兒童要高。因此,研究人員發(fā)現(xiàn)了沙棘果與兒童智力發(fā)育之間的聯(lián)系。

  以下哪項(xiàng)如果為真,最不可能削弱上述論證?

  A.對成年人的研究發(fā)現(xiàn),每天吃沙棘果的人智力水平并不比很少吃沙棘果人的高。

  B.調(diào)查顯示:沙棘果價(jià)格非常高,只有富裕家庭的兒童才經(jīng)常吃,同時(shí)這些家庭有條件實(shí)現(xiàn)兒童的早期智力開發(fā)。

  C.這項(xiàng)兒童發(fā)育研究的課題負(fù)責(zé)人是沙棘果生產(chǎn)商,其目的就是要擴(kuò)展沙棘果的銷售渠道。

  D.沙棘果是兒童喜歡的食品,家長經(jīng)常把沙棘果作為禮物獎(jiǎng)給智力表現(xiàn)優(yōu)異的孩子。

  解析:吃沙棘果與兒童智力發(fā)展是否存在必然聯(lián)系。通過樣本調(diào)查判斷出吃沙棘果與兒童智力發(fā)展存在正相關(guān)的關(guān)系。B選項(xiàng)說明家庭條件是否富裕與兒童智力發(fā)展存在關(guān)聯(lián)聯(lián)系;C選項(xiàng)說明題目中的結(jié)論是由于生產(chǎn)商的廣告產(chǎn)生的以偏概全的結(jié)論;D選項(xiàng)屬于因果倒置,成績優(yōu)異是因,吃沙棘果是結(jié)果。

  3.凡金屬都是導(dǎo)電的。銅是導(dǎo)電的,所以銅是金屬。

  下面哪項(xiàng)與上述推理結(jié)構(gòu)最相似?

  A.所有的鳥都是卵生動物,蝙蝠不是卵生動物,所以,蝙蝠不是鳥。

  B.所有的鳥都是卵生動物,天鵝是鳥,所以天鵝是卵生動物。

  C.所有從事工商管理的都要學(xué)習(xí)企業(yè)管理,老陳是學(xué)習(xí)企業(yè)管理的,所以,老陳是從事工商管理工作的。

  D.華山險(xiǎn)于黃山,黃山險(xiǎn)于泰山,所以華山險(xiǎn)于泰山。

  解析:題目結(jié)構(gòu)是所有的A具有某中屬性,B屬于A,因此B具有A的屬性,選項(xiàng)C與上述結(jié)論類似。

  4.為了有助于人們選擇最滿意的城市居住,有關(guān)部門實(shí)施了一項(xiàng)評選“最舒適城市”的活動。方法是,選擇十個(gè)方面,包括社會治安、商業(yè)設(shè)施、清潔程度、綠化程度、教育設(shè)施、旅游文化景點(diǎn)等等,每個(gè)方面按實(shí)際質(zhì)量的高低,評以1分至10分之間的某一分值,然后求得十個(gè)分值的平均數(shù)即是這個(gè)城市的舒適性指數(shù)。

  以下哪項(xiàng)是實(shí)施上述活動需要預(yù)設(shè)的前提?

 、瘛⒊鞘械母鞣N舒適性質(zhì)量程度都可以用準(zhǔn)確的數(shù)字表達(dá)。

  Ⅱ、城市的各種舒適性對于居民來說都是同等重要的。

 、、居民有自由選擇居住城市的權(quán)利并且大都樂于這樣做。

  A.僅Ⅰ。      B.僅Ⅲ。      C.僅Ⅰ和Ⅱ。      D.Ⅰ、Ⅱ和Ⅲ。

  解析:這道題是補(bǔ)充假設(shè)的問題,由于每個(gè)人對“舒適程度”的感覺是不一樣的,因此題干結(jié)論若要正確,條件Ⅱ是必須的前提;若題目中結(jié)論正確,需要假設(shè)題目中的十個(gè)指標(biāo)存在量化標(biāo)準(zhǔn),因此條件Ⅰ為必選項(xiàng);條件Ⅲ存在爭議,不過可以從另外一個(gè)角度理解,便于理解題目的結(jié)論,如果居民沒有選擇居住地的權(quán)利,那么題目中的滿意度調(diào)查就沒有意義,所以也得不到上述結(jié)論。

  5.衛(wèi)生部的官員們對牟定縣狂犬病疫情有以下斷定:

  (1)該縣所有的狗都得了狂犬病。

  (2)該縣有些斑點(diǎn)狗得了狂犬病。(真)

  (3)該縣有些狗得了狂犬病。

  (4)該縣有些狗沒得狂犬病。(真)

  其實(shí)上述斷定中只有兩個(gè)與事實(shí)相符。根據(jù)如上的情況,以下哪項(xiàng)結(jié)論是可能成立的?

  I.該縣的狗都是斑點(diǎn)狗。

  II. 該縣沒有斑點(diǎn)狗可能得狂犬病。

  III. 該縣的斑點(diǎn)狗并非可能沒有得狂犬病。

  IV. 該縣的狗不可能都沒得狂犬病。

  A.僅I      B.僅III      C.僅II和IV      D.僅IV。

  解析:這道題屬于邏輯推理中真假話題的問題。解析這種問題按照1、先找矛盾;2、繞開矛盾;3、推出答案的步驟進(jìn)行。首先先找矛盾,首先1和2是一對矛盾,3和4是一對矛盾,只有一個(gè)正確,2、繞開矛盾,如果1正確,那么3和4都假,因此只能2為真,如果2為真,只能判斷4為真,3為假,3、推出答案,條件2有些斑點(diǎn)夠得了狂犬病,可推出該縣沒有斑點(diǎn)狗可能得狂犬病,大家注意必然=沒有……必然;同時(shí)根據(jù)條件4可以得出該縣的狗不可能都沒得狂犬病。

  6.如果一個(gè)兒童的體重與身高的比值超過本地區(qū)80%兒童的水平,就稱其為肥胖兒。根據(jù)歷年的調(diào)查結(jié)果,15年來,臨江市的肥胖兒的數(shù)量一直在穩(wěn)定增長。

  如果以上斷定為真,則以下哪項(xiàng)也必為真?

  A.臨江市每一個(gè)肥胖兒的體重都超過全市兒童的平均體重。

  B.15年來,臨江市的兒童的體育鍛煉越來越不足。

  C.臨江市的非肥胖兒的數(shù)量15年來不斷增長。

  D.15年來,臨江市體重不足標(biāo)準(zhǔn)體重的兒童數(shù)量不斷下降。

  解析:這道題屬于迷惑推理題,大家務(wù)必要小心。首先大家可以看出臨江市兒童數(shù)量是在增長的;兒童由肥胖和非肥胖組成;肥胖兒童的數(shù)量穩(wěn)定增長,說明非肥胖兒童的數(shù)量也在穩(wěn)定增長;沒有證據(jù)表明非肥胖兒童的數(shù)量在不斷下降。

  7.孩子出生后的第一年在托兒所里度過,會引發(fā)孩子的緊張不安。在我們的研究中,有464名12~13歲的兒童接受了特異情景測試法的測驗(yàn),該項(xiàng)測驗(yàn)意在測試兒童1歲時(shí)的狀況與對母親的依附心理之間的關(guān)系。其結(jié)果:有41.5%曾在托兒所看護(hù)的兒童和25.7%曾在家看護(hù)的兒童被認(rèn)為緊張不安,過于依附母親。

  以下哪項(xiàng)如果為真,最沒有可能對上述研究的推斷提出質(zhì)疑?

  A.出生后第一年在家看護(hù)的孩子多數(shù)是由祖父母或外祖父母看護(hù)的,并形成濃厚的親情。

  B.這項(xiàng)研究的主持者被證實(shí)曾經(jīng)在自己的幼兒時(shí)期受到過長時(shí)間來自托兒所阿姨的冷落。

  C.針對孩子母親的另一部分研究發(fā)現(xiàn):由于孩子在家里表現(xiàn)出過度的依附心理,父母因此希望將其送入托兒所予以矯正。

  D.研究中所測試的孩子并不是從托兒所看護(hù)和在家看護(hù)兩種情況下隨機(jī)抽取的。因此,這兩組樣本兒童的家庭很可能有系統(tǒng)性的差異存在。

  解析:這種題最好的解決辦法是否定推斷,把你認(rèn)為可能的答案否定,帶入題目中判斷。如果能夠提出質(zhì)疑,說明你的判斷是正確的。A選項(xiàng)。

  8.在玉米地中套種的小麥有可能得小麥黃葉病,這種病是由于光照不足造成的,一旦光照充足,比如減少玉米植株數(shù)量,或者玉米收割后,癥狀很快就會消失,且不會影響產(chǎn)量。而小麥銹病是由花斑銹菌引起,如果在病癥初起時(shí)不及時(shí)恰當(dāng)應(yīng)對,就會使小麥植株很快枯死。由于小麥銹病和小麥黃葉病初期癥狀很難區(qū)分,所以,在玉米地中套種小麥,必須高度注意小麥銹病的定期篩查和預(yù)防。

  以下哪項(xiàng)最可能是上述論證所假設(shè)的?

  麥黃葉病不會誘發(fā)小麥銹病。

  B.花斑銹菌不會因?yàn)槌渥愎庹斩π←湹那趾αΑ?/p>

  C.小麥銹病如果及時(shí)恰當(dāng)處理,就不會使小麥減產(chǎn)。

  D.科學(xué)家一直沒有找到有效遏制小麥銹病的藥物。

  解析:假設(shè)推斷。陽光充足,小麥黃葉病會消失,產(chǎn)量不影響;花斑銹菌,使小麥植株枯死。因此需要假設(shè)陽光充足不會抑制花斑銹菌的發(fā)病,才能得出小麥黃葉不發(fā)病時(shí),花斑銹菌仍然會發(fā)病,導(dǎo)致兩種病情難以區(qū)分。換一種角度,如果小麥黃葉病不會和花斑銹菌同時(shí)發(fā)病,那么就不存在難以區(qū)分病情的問題了。

  9.公司準(zhǔn)備從五個(gè)業(yè)務(wù)骨干中選幾個(gè)人去中央戲劇學(xué)院進(jìn)修。鑒于業(yè)務(wù)員們的工作關(guān)系,總經(jīng)理建議:

  (1)如果選鞏俐,那么必須選章子怡并且不能選周迅。

  (2)如果選章子怡或者選周迅,則不能選吳孟達(dá)。

  (3)不能既不選周星馳也不選吳孟達(dá)。

  考慮總經(jīng)理的建議,董事會認(rèn)為,鞏俐必須去中央戲劇學(xué)院進(jìn)修,這樣誰將跟她一起去進(jìn)修?

  A.章子怡和周星馳    B.章子怡和吳孟達(dá)   C.周星馳和周迅    D.吳孟達(dá)和周迅

  解析:邏輯推斷。如果鞏麗必須去,根據(jù)條件1章子儀必須去,而周迅一定不去,從而排除C、D選項(xiàng),根據(jù)條件2,如果選章子儀,那么吳孟達(dá)不能去。排除B 。

  繼續(xù)進(jìn)行推理,根據(jù)條件3必須在周星馳和吳孟達(dá)中選一個(gè),因?yàn)楦鶕?jù)條件2已經(jīng)排除吳孟達(dá),所以周星馳必須去。選擇A。

  10.20世紀(jì)60年代早期之前,挪威的斯塔溫格是一個(gè)安靜而和平的小鎮(zhèn)。進(jìn)入60年代以來,這里成為了挪威近海石油勘探中心。從此,暴力犯罪和毀壞公物的現(xiàn)象在斯塔溫格小鎮(zhèn)急劇增加。顯然這些社會問題是是石油繁榮的副產(chǎn)品。

  下列哪一個(gè)選項(xiàng)最強(qiáng)烈的支持上面的論述。

  A.暴力犯罪和毀壞公物在沒有石油繁榮的挪威城鎮(zhèn)保持低水平。

  B.斯塔溫格居民很少對這里是近海石油勘探中心感到遺憾。

  C.挪威社會學(xué)家對斯塔溫格的暴力犯罪和毀壞公物現(xiàn)象持續(xù)增加表示關(guān)切。

  D.非暴力犯罪、毒品、離婚,在這個(gè)小鎮(zhèn)也以同樣的速度增加著。

  解析:題目把社會問題的原因歸結(jié)為石油繁榮,最強(qiáng)烈支持的論據(jù)為在沒有石油繁榮的時(shí)期,社會問題處于低水平。因此選A。

11.認(rèn)為只傷害自己而沒有傷害到別人的行為并沒有什么錯(cuò)誤的想法,通常伴隨著對人與人之間相互依賴的人際關(guān)系的忽視。毀壞一個(gè)人自己的健康或者生命意味著不能你將不能為家庭或者社會提供幫助,相反意味著你將額外享有那些本來就有限的諸如食物、健康服務(wù)和教育等社會資源,而不是相反將它們回報(bào)給社會。

  以下哪個(gè)選項(xiàng)如果正確,最強(qiáng)烈的地支持題干所表達(dá)的觀點(diǎn)。

  A.本來可以避免的疾病和意外事故所造成的花銷增加了每個(gè)人的健康保險(xiǎn)金。

  B.傷害一個(gè)人能夠?qū)е麻g接的利益,諸如可以使別人獲得與健康領(lǐng)域相關(guān)聯(lián)的工作。

  C.一個(gè)人對社會所作的貢獻(xiàn)可以通過他的健康程度來衡量。

  D.由喝酒、吸煙和吸食毒品所導(dǎo)致的主要傷害是由使用那些東西的人來承受的。

  解析:題目中的結(jié)論是傷害自己的行為與傷害別人的行為同樣都是錯(cuò)誤的。原因是傷害自己將不能為家庭和社會提供幫助,同時(shí)會向社會索取的更多。這道題的關(guān)鍵是大家要讀懂題干,這樣從選項(xiàng)中我們不難選出A,本來可以避免的個(gè)人傷害回增加健康保險(xiǎn)金,從而增加社會的負(fù)擔(dān)。

  12.大多數(shù)道路的修理比預(yù)算的要花費(fèi)更多的時(shí)間和金錢,但是去年夜間修理京津塘高速公路和類似的道路并未比預(yù)算花費(fèi)更多的時(shí)間或金錢。因此,在夏季,夜間修理主要道路可能省時(shí)省錢。

  下列哪一個(gè),假如正確,最支持上面得到的結(jié)論?

  A.修理京津塘高速公路的預(yù)算足夠用,所以不可能超過預(yù)算。

  B.夏季,夜間路上通行車輛較少,而且溫度較舒適,允許修路工人工作得更快。

  C.愿意在晚上工作的修路工人較容易找到工作機(jī)會,因?yàn)榇蠖鄶?shù)人寧愿白天工作。

  D.用于道路修理的瀝青在較高溫度下膨脹,在溫度降低的時(shí)候收縮。

  解析:補(bǔ)充論據(jù)。從題干可以看出從特殊樣本推出結(jié)論明顯論據(jù)不足。夏天夜間施工本來和省時(shí)省錢本來沒有關(guān)系,屬于不相關(guān)關(guān)系,因此需要補(bǔ)充適當(dāng)?shù)恼摀?jù),在這兩個(gè)問題中間搭起橋梁,建立關(guān)系,所以選擇B。

  13.偏頭痛一直被認(rèn)為是由食物過敏引起的。但是,如果讓患者停止食用那些已經(jīng)證明會不斷引起過敏性偏頭痛的食物,他們的偏頭痛并沒有停止,因此,顯然存在別的某種原因引起偏頭痛。下列哪項(xiàng)如果為真,最能削弱上面的結(jié)論?

  A.許多患者說誘發(fā)偏頭痛病的那些食物往往是他們最喜歡吃的食物。

  B.許多普通食物只在食用幾天后才誘發(fā)偏頭痛,因此,不容易觀察患者的過敏反應(yīng)和他們食用的食物之間的關(guān)系。

  C.很少有食物過敏會引起像偏頭痛那樣嚴(yán)重的癥狀。

  D.許多不患偏頭痛的人同樣有食物過敏反應(yīng)。

  解析:題目結(jié)論是過敏引發(fā)偏頭痛,但是引發(fā)過敏的問題不僅僅是食物引起的。結(jié)論指明食物過敏與偏頭痛之間不存在這因果關(guān)系,原因是停用了這種食物之后,偏頭痛的病情仍然存在,若要削弱這個(gè)結(jié)論,需要針對上述論據(jù)提出反對。本著這個(gè)思路我們看選項(xiàng),結(jié)論B。表明了停用食物之后仍然存在偏頭痛的原因是因?yàn)樵S多普通食物在食用幾天后才發(fā)作,因此雖然停用了食物,仍會發(fā)作一段時(shí)間。

  14.世界衛(wèi)生組織在全球范圍內(nèi)進(jìn)行了一項(xiàng)有關(guān)獻(xiàn)血對健康的影響的跟蹤調(diào)查。調(diào)查對象分為三組。第一組對象中均有二次以上的獻(xiàn)血記錄,其中最多的達(dá)數(shù)十次;第二組中的對象均僅有一次獻(xiàn)血記錄;第三組對象均從未獻(xiàn)過血。調(diào)查結(jié)果顯示,被調(diào)查對象中癌癥和心臟病的發(fā)病率,第一組分別為0.3%和0.5%,第二組分別為0.7%和0.9%,第三組分別為1.2%和2.7%。一些專家依此得出結(jié)論,獻(xiàn)血有利于減少患癌癥和心臟病的風(fēng)險(xiǎn)。這兩種病已經(jīng)不僅在發(fā)達(dá)國家而且在發(fā)展中國家成為威脅中老年人生命的主要?dú)⑹帧R虼,獻(xiàn)血利己利人,一舉兩得。以下哪項(xiàng)如果為真,將削弱以上結(jié)論?

  I 60歲以上的調(diào)查對象,在第一組中占60%,在第二中占70%,在第三組中占80%。

  II 獻(xiàn)血者在獻(xiàn)血前要經(jīng)過嚴(yán)格的體檢,一般具有較好的體質(zhì)。

  III 調(diào)查對象的人數(shù),第一組為1700人,第二組為3000人,第三組為7000人。

  A.只有I和II。    B.只有II和III。    C.只有I和III    D.只有I

  解析:題目中對獻(xiàn)血與癌癥、心臟病的發(fā)病幾率建立了正相關(guān)關(guān)系。但是樣本仍然存在問題,如果鮮血的人本身身體素質(zhì)就比沒有獻(xiàn)血的身體素質(zhì)好,就不能退出上述結(jié)論。這是最簡單容易想出的悖論。因此淘汰C、D。下面在條件1和3中選一個(gè),條件3說明樣本容量不同,但是題目中使用的是百分?jǐn)?shù)這個(gè)相對指標(biāo),因此不能形成反對結(jié)論的悖論。選A。

  15.許多孕婦都出現(xiàn)了維生素缺乏的癥狀,但這通常不是由于孕婦的飲食中缺乏維生素,而是由于腹內(nèi)嬰兒的生長使她們比其他人對維生素有更高的需求。為了評價(jià)上述結(jié)論的確切程度,以下哪項(xiàng)操作最為重要?

  A.對某個(gè)不缺乏維生素的孕婦的日常飲食進(jìn)行檢測,確定其中維生素的含量。

  B.對孕婦的科學(xué)食譜進(jìn)行研究,以確定有利于孕婦攝入足量維生素的最佳選擇。

  C.對日常飲食中維生素足量的一個(gè)孕婦和一個(gè)非孕婦進(jìn)行檢測,并分別確定她們是否缺乏維生素。

  D.對日常飲食中維生素不足量的一個(gè)孕婦和另一個(gè)非孕婦進(jìn)行檢測,并分別確定她們是否缺乏維生素。

  解析:支持論據(jù)。題目需要根據(jù)結(jié)論找出支持論據(jù),顯然對同量的維生素給孕婦和非孕婦服用,觀察他們是否缺乏維生素,對得出結(jié)論最重要。選C

  16.經(jīng)A省的防疫部門檢測,在該省境內(nèi)接受檢疫的長尾猴中,有1%感染上了狂犬病。但是只有與人及其寵物有接觸的長尾猴才接受檢疫。防疫部門的專家因此推測,該省長尾猴中感染有狂犬病的比例,將大大小于1%。

  以下哪項(xiàng)如果為真,將最有力地支持專家的推測?

  A.在A省境內(nèi),與人及其寵物有接觸的長尾猴,只占長尾猴總數(shù)的不到10%。

  B.在A省,感染有狂犬病的寵物,約占寵物總數(shù)的0.1%。

  C.在與A省毗鄰的B省境內(nèi),至今沒有關(guān)于長尾猴感染狂犬病的疫情報(bào)告。

  D.與健康的長尾猴相比,感染有狂犬病的長尾猴更愿意與人及其寵物接觸。

  解析:選擇D,其他選項(xiàng)均不能支持題干給出的結(jié)論。在與人和動物接觸的長尾猴進(jìn)行檢驗(yàn),發(fā)現(xiàn)1%感染了狂犬病,推斷出該省長尾猴感染狂犬病的比例小于1%,那么只有感染狂犬病的長尾猴更喜歡與人和動物接觸能夠有效地支持這個(gè)結(jié)論。

  17.作為市電視臺的攝像師,最近國內(nèi)電池市場的突然變化讓我非常頭疼。進(jìn)口電池缺貨,我只能用國產(chǎn)電池來代替作為攝像的主要電源。盡管每單位的國產(chǎn)電池要比進(jìn)口電池便宜,但我估計(jì)如果持續(xù)用國產(chǎn)電池替代進(jìn)口電池來提供同樣的電源供應(yīng)的話,我在能源上的支付將會提高。

  說這番話的人在上面這段話中隱含了以下哪項(xiàng)假設(shè)?

  A. 持續(xù)使用國產(chǎn)電池,攝像的質(zhì)量將無法得到保障。

  B. 每單位的進(jìn)口電池要比國產(chǎn)電池價(jià)格貴。

  C. 生產(chǎn)國產(chǎn)電池要比生產(chǎn)進(jìn)口電池成本低。

  D. 以每單位電池提供的電能來計(jì)算,國產(chǎn)電池要比進(jìn)口電池提供得少。

  解析:這是日常生活中經(jīng)常會遇到的問題,答案一目了然選擇D。得出結(jié)論的原因是盡管國產(chǎn)電池比進(jìn)口電池單價(jià)便宜,但是就單位電能來講,國產(chǎn)電池的價(jià)格卻比進(jìn)口電池高。

  18.最近由于氣候的異常變化,使得山東省的大蒜產(chǎn)量受到較大影響而減產(chǎn)。山東大蒜的價(jià)格比平時(shí)同期上漲了兩倍,這就大大提高了大蒜素生產(chǎn)的成本,估計(jì)未來大蒜素的市場價(jià)格將有大幅度的提高。以下哪項(xiàng)如果是真的,最能削弱上述結(jié)論?

  A.去年大蒜的價(jià)格是歷年最低的。

  B.其他替代原料可以用來生產(chǎn)人工大蒜素。

  C.除了山東省外,其他省份也可以提供大蒜。

  D.最近的天氣異常不如專家們估計(jì)的那么嚴(yán)重。

  解析:題目中的推斷是沒有問題的,成本的提高會促使價(jià)格的提高,但是其他省也提供大蒜,而成本并沒有提高,價(jià)格不會上升,在這樣的市場上,山東省的大蒜盡管成本提高了,但是價(jià)格將會保持市場水平,因此C有效地消弱了結(jié)論。

  19.某學(xué)校開設(shè)選修課,規(guī)定:同學(xué)只有選了文化史課的同學(xué)才能選古典音樂課。如果選古典音樂課,那他就不能選文學(xué)欣賞課。如果選文學(xué)欣賞課,就不能選古典音樂課。

  如果以上陳述為真,以下哪項(xiàng)陳述不可能假?

  A.同學(xué)不選文學(xué)欣賞課 或者不選古典音樂課。

  B.同學(xué)選文學(xué)欣賞課 或者不選古典音樂課。

  C.同學(xué)不選古典音樂課,或者不選文化史課。

  D.同學(xué)選古典音樂課,或者不選文學(xué)欣賞課

  解析:邏輯推斷題。不可能假=一定真,因此題目要求在選項(xiàng)中找出一定真的項(xiàng)。選了文化史,才能選古典音樂,這是個(gè)必要條件;但是選了古典音樂就不能選文學(xué)欣賞,二選其一;選項(xiàng)A的論述與題干一樣。

  20.20年前,幾乎所有的公司首腦在選擇重新設(shè)置公司總部的時(shí)候,主要關(guān)心的是土地價(jià)格。今天一個(gè)高級執(zhí)行官計(jì)劃重設(shè)總部時(shí)主要關(guān)心的東西更廣泛了,經(jīng)常包括當(dāng)?shù)氐膶W(xué)校和住房質(zhì)量。

  假如上述信息可靠,下面哪一項(xiàng)最好地解釋了公司首腦們主要關(guān)心問題的變化?

  A.20年前高質(zhì)量的住房和學(xué)校像今天一樣難以發(fā)現(xiàn)。

  B.近年來優(yōu)秀專業(yè)辦公人員缺乏的問題迫使公司找到盡可能多的方法來留住老員工,并且吸引優(yōu)秀新員工的加盟。

  C.公司執(zhí)行官總是考慮自己的決定將怎樣影響公司的利潤。

  D.在過去20年,一些地區(qū)比其他地區(qū)土地價(jià)格變化少。

  解析:選項(xiàng)A屬于強(qiáng)拉因果,不存在相關(guān)關(guān)系;選項(xiàng)B說明了高級執(zhí)行官在公司選址時(shí)關(guān)注學(xué)校和住房質(zhì)量的原因,分別是吸引優(yōu)秀新員工加盟和留住老員工;選項(xiàng)C與題干沒有關(guān)系,并未涉及到利潤這個(gè)因素;選項(xiàng)D,最多可以說明高級執(zhí)行官不在關(guān)心土地價(jià)格的原因,但是并不能說明關(guān)注學(xué)校和住房質(zhì)量的原因。

 

21.用蒸餾麥芽渣提取的酒精作為汽油的替代品進(jìn)入市場,使得糧食市場和能源市場發(fā)生了前所未有的直接聯(lián)系。到1995年,谷物作為酒精的價(jià)值己經(jīng)超過了作為糧食的價(jià)值。西方國家已經(jīng)或正在考慮用從谷物提取的酒精來替代一部分進(jìn)口石油。

  如果上述斷定為真,對于那些己經(jīng)用從谷物提取的酒精來替代一部分進(jìn)口石油的西方國家。以下哪項(xiàng),最可能是1995年后進(jìn)口石油價(jià)格下跌的后果?

  A.一些谷物從能源市場轉(zhuǎn)入糧食市場。

  B.一些谷物從糧食市場轉(zhuǎn)入能源市場。

  C.谷物的價(jià)格面臨下跌的壓力。

  D.谷物的價(jià)格出現(xiàn)上浮。

  解析:谷物可以提取出酒精來代替石油出口,因此可以判斷谷物和石油成為了相互競爭的替代品,因此如果石油價(jià)格下跌,那么谷物的價(jià)格也必然會下調(diào),否則進(jìn)口國將會進(jìn)口價(jià)格低廉的石油取代谷物。

  22.有八個(gè)教授參加學(xué)術(shù)會議后,互留通信地址,以保持聯(lián)絡(luò)。一年后,統(tǒng)計(jì)他們之間的通信情況如下:有一人給其他三個(gè)人寫過信,給其他人寫過兩封信的有三人,給其他人寫過一封信的有四個(gè)人。若以上統(tǒng)計(jì)屬實(shí),則最能得出以下哪項(xiàng)結(jié)論?

  A.通過書信來往,他們遞進(jìn)了友誼,也加強(qiáng)了學(xué)術(shù)聯(lián)系。

  B.八個(gè)教授都收到過其他教授的來信。

  C.至少有一個(gè)人收到信件后沒有都回復(fù)。

  D.雖然參加學(xué)術(shù)會議的時(shí)候大家不熟悉,但現(xiàn)在他們已經(jīng)成為至交。

  解析:選項(xiàng)首先排除A、D,沒有任何證據(jù)表明通過書信加強(qiáng)了學(xué)術(shù)聯(lián)系,也沒有證據(jù)表明他們成為了至交。下面看選項(xiàng)B、C,我們可以通過畫圖的方法來解決這個(gè)問題相對比較簡單,從而得出正確答案為C。

  23.金鳳扒雞是一個(gè)享有盛譽(yù)的百年扒雞品牌,在石家莊銷路很好,深受當(dāng)?shù)匕傩障矏,每天一上市就會被搶購一空。為了打開北京市場,金鳳扒雞改進(jìn)了工藝,用真空包裝,延長了保質(zhì)期,試圖在北京各大超市打開銷路,但是金鳳扒雞在北京卻很難銷售出去。

  下面除了哪一項(xiàng),都有助于解決上述現(xiàn)象?

  A.北京人和石家莊人的飲食習(xí)慣不同。他們更喜歡吃烤鴨而不是扒雞。

  B.恒慧通燒雞也采用真空包裝,保質(zhì)期一樣長,在北京的銷路卻很好。

  C.人們喜歡吃剛出鍋的新鮮扒雞,保質(zhì)期短的反而更受消費(fèi)者青睞。

  D.雖然金鳳扒雞是百年品牌,但是它的銷售地點(diǎn)僅在石家莊市區(qū),北京人對這個(gè)品牌不熟悉。

  解析:這是一道頗具爭議的題目,選項(xiàng)A、B、C、D分別從消費(fèi)者的喜好,競爭者,消費(fèi)偏好和品牌對金鳳扒雞在北京銷路受阻提供了良好的解釋。但是選項(xiàng)B可以從另外一個(gè)角度思考,恒惠通燒雞同樣采用真空包裝,保質(zhì)期一樣長,什么都和金鳳扒雞一樣,為什么銷路很好,而金鳳扒雞為什么銷路不好,這就不能解釋為什么金鳳扒雞在北京銷路不好的原因。

  24.中國自1978年改革開放以來28年間,從人口數(shù)量上說:農(nóng)民的數(shù)量并沒有減少,而是持續(xù)增加的,只不過這種增長速度相對于總?cè)丝诘脑鲩L速度來說,是比較低的。

  下列那一句話直接與上述信息矛盾?

  A.在總?cè)丝谥修r(nóng)民的數(shù)量在1978年到2006年間略微增長了。

  B.農(nóng)民在總?cè)丝谥械谋嚷蕪?978年的81.4%增加到了2006年的85.3%。

  C.中國勞動力的增長率和總?cè)丝诘脑鲩L率在1978年至今的28年里同時(shí)增加了。

  D.中國的勞動力中,農(nóng)民的比率從1978年的78%下降到了75.6%。

  解析:選項(xiàng)B可以看出,農(nóng)民相對總?cè)丝诘脑鲩L速度并不低,增長速度也是增加的。

  25.有三戶人家,每家有一孩子,他們的名字是:小萍(女)、小紅(女)、小虎。孩子的爸爸是老王、老張和老陳;媽媽是劉蓉、李玲和方麗。對于這三家人,已知:

 。1)老王家和李玲家的孩子都參加了少女舞蹈隊(duì)。

 。2)老張的女兒不是小紅。

 。3)老陳和方麗不是一家人。

  根據(jù)以上條件,確定以下哪項(xiàng)是正確的?

  A.老王、劉蓉和小萍是一家。

  B.老張、李玲和小紅是一家。

  C.老王、方麗和小紅是一家。

  D.老陳、方麗和小虎是一家。

  解析:邏輯推斷題。從條件1可以看出小虎不是老王和李玲的孩子,因?yàn)樾』⑹悄泻ⅰD敲纯梢酝瞥鲂∑己托〖t的爸爸可能是老王。根據(jù)條件2,老張的女兒不是小紅,所以可以確定老王的女兒是小萍。鎖定選項(xiàng)A,如果A正確,那么根據(jù)條件3老陳和李玲是一家,根據(jù)條件1,女兒是小紅;得出老陳、方麗和小虎是一家。那么選項(xiàng)D也正確。因此同時(shí)排除A、D。選項(xiàng)B與條件2不符,因此選擇C。

  26.5名犯罪嫌疑人被警察詢問,事后得知其中一名是罪犯。下面是五個(gè)嫌疑人的供述,其中只有3句真話。問,誰是罪犯?

  甲:丁是罪犯。

  乙:我是無辜的。

  丙:戊不是罪犯。

  。杭自谌鲋e。

  戊:乙說的是實(shí)話。

  A.戊    B.甲和丙    C.丁    D. 乙和戊

  解析:從選項(xiàng)入手,選項(xiàng)B,甲和丙是罪犯,可以推出乙、戊、丁、丙都為真,而題目表明只有3項(xiàng)為真,所以放棄B;選項(xiàng)C,丁是罪犯,可以推出甲、乙、丙、戊為真,同樣與題目要求不符;選項(xiàng)D,乙和戊是罪犯,只能推出丁為真。因此這道題選擇A。

  27.學(xué)生上完體育課后回到教室,有15人喝了飲水機(jī)里的純凈水,其中5人很快產(chǎn)生了腹瀉。飲水機(jī)里的純凈水馬上被送去檢驗(yàn),檢驗(yàn)的結(jié)果不能肯定其中有造成腹瀉的有害物質(zhì)。因此,喝了飲水機(jī)里的純凈水不是造成腹瀉的原因。

  如果上述檢驗(yàn)結(jié)果是正確的,則以下哪項(xiàng)對上述論證的評價(jià)最為恰當(dāng)?

  A.題干的論證有漏洞,因?yàn)樗鼪]有考慮到另一個(gè)事實(shí):哪些沒有喝了飲水機(jī)里的純凈水的人沒有造成腹瀉。

  B.題干的論證有漏洞,因?yàn)樗讶鄙僮C據(jù)證明某種情況存在,當(dāng)作有充分證據(jù)證明某種情況不存在。

  C.題干的論證有漏洞,因?yàn)樗鼪]有利用一個(gè)有力的論據(jù):為什么有更多人喝了飲水機(jī)里的純凈水沒有造成腹瀉。

  D.題干的論證有漏洞,因?yàn)樗鼪]有指出造成腹瀉的真正原因。

  解析:理解這道題非常簡單,舉個(gè)例子說明,如果一個(gè)人確實(shí)殺了人,但是法官沒有證據(jù)表明他是兇手,那么可能無法定罪,但終不能得出他不是兇手的結(jié)論,因?yàn)橥瑯記]有證據(jù)表明他不是兇手。題目中給出了同樣的問題,檢驗(yàn)結(jié)果不能肯定有造成腹瀉的有害物質(zhì),但是并不能說明純凈水不是造成腹瀉的原因。

  28.一項(xiàng)全球范圍的調(diào)查顯示,近 10年來:吸煙者的總數(shù)基本保持不變;每年只有10%的吸煙者改變自己的品牌,即放棄原有的品牌而改吸其他品牌;煙草制造商用在廣告上的支出占其毛收入的10%。在Z煙草公司的年終董事會上,懂事A認(rèn)為,上述統(tǒng)計(jì)表明,煙草業(yè)在廣告上的收益正好等于其支出,因此,此類廣告完全可以不做。以下哪項(xiàng),構(gòu)成對懂事A的結(jié)論的最有力質(zhì)疑?

  A.懂事A的結(jié)論忽視了:今年來各種品牌的香煙的價(jià)格都有了很大的變動。

  B.懂事A的結(jié)論基于一個(gè)錯(cuò)誤的假設(shè):每個(gè)吸煙者在某個(gè)時(shí)候只喜歡一種品牌。

  C.懂事A的結(jié)論基于一個(gè)錯(cuò)誤的假設(shè):每個(gè)煙草制造商只生產(chǎn)一種品牌。

  D.懂事A的結(jié)論忽視了:世界煙草業(yè)是一個(gè)由處于競爭狀態(tài)的眾多經(jīng)濟(jì)實(shí)體組成的。

  解析:董事A的判斷為廣告的支出正好與收益相等,因此得出廣告可以不做的結(jié)論。但是忽略了如果不做廣告是否能夠獲得與做廣告時(shí)除去廣告費(fèi)用后相同的收益,因?yàn)檫@部分收入也是由廣告產(chǎn)生的效益。因?yàn)橥袠I(yè)的其他競爭者會因?yàn)閺V告,增加10%的顧客,而不做廣告的煙草公司將有減少10%顧客的風(fēng)險(xiǎn)。因此選擇D

  29.在收款局的5個(gè)賬單收款員中,楊先生收款的不成功率最高。然而楊先生是這個(gè)局的職員中最好的賬單收款員。

  下面哪一項(xiàng),如果正確,最有助于解決上述短文中的明顯分歧?

  A.在加入收款局之前,楊先生是一個(gè)大百貨公司的信貸部的一名職員。

  B.收款局中的其他四個(gè)收款員都認(rèn)為楊先生是一個(gè)非常能干的帳單收款員。

  C.楊先生在過去的幾年內(nèi),每年收款成功的比率都保持相當(dāng)?shù)胤(wěn)定。

  D.這個(gè)收款局的大多數(shù)最困難的事情都是派楊先生去做的。

  解析:從題干入手分析,楊先生是最好的賬單收款員,但是收款不成功率最高。題目要求找到解釋這一矛盾的原因。顯然D最合適。

  30.在一次工程碩士入學(xué)考試中,據(jù)統(tǒng)計(jì),在英語、數(shù)學(xué)、語文和邏輯四個(gè)科目中:70%的考生英語及格了,75%的考生數(shù)學(xué)及格了,80%的考生語文及格了,85%的考生邏輯及格了。如果以上情況屬實(shí),那么,以下哪個(gè)選項(xiàng)一定正確:

  A.在此次考試中,大部分考生四門課都及格了。

  B.在此次考試中,大部分考生四門課沒有都及格。

  C.在此次考試中,有不少于10%的考生,四門課程都及格了。

  D.在此次考試中,至少有10%的考生,四門課程都沒及格。

  解析:不少于=大于等于,至少=大于等于。選線排除A、B,選項(xiàng)過于籠統(tǒng),并且與C、D存在重復(fù)關(guān)系。選項(xiàng)D與題干明顯不符合,選擇C。

 31.某游戲的游戲規(guī)則有這樣一條規(guī)定:任何在游戲中拒絕成為選手的人將在拒絕的時(shí)候被減掉10分。

  下列哪一個(gè)是此條規(guī)則所暗含的?

  A.同意參加游戲的所有人的得分將高于那些因?yàn)榇艘?guī)則而被判罰分的人。

  B.一個(gè)人在拒絕玩這個(gè)游戲的同時(shí),依然是這個(gè)游戲的參與者。

  C.最初同意成為游戲參加者然后在游戲進(jìn)行中退出的人可以避免被罰10分。

  D.一個(gè)拒絕玩這個(gè)游戲的人不能在游戲中被宣布為失敗者。

  解析:由于拒絕玩這個(gè)游戲,會被剪掉20分,說明依舊是游戲的參與者,選B。

  32.受到廣泛贊揚(yáng)的斯坦尼斯拉夫斯基演員訓(xùn)練法來自他做年輕演員時(shí)對戲劇不熟練和易受影響的老套做法。對該方法的理解必須基于斯坦尼斯拉夫斯基個(gè)人的演藝研究,從而從典型化的姿勢、反復(fù)嘗試的嗓音語調(diào)和標(biāo)準(zhǔn)的感情模式的誘惑中解脫出來。盡管他的美國門徒有過要求,但是這位俄國導(dǎo)演從未打算寫一本對表演問題提出嚴(yán)格解決方法的教科書。

  關(guān)于表演這個(gè)問題,上文作者對斯坦尼斯拉夫斯基的演員訓(xùn)練法持有怎樣的觀點(diǎn)?

  A.斯坦尼斯拉夫斯基的美國門徒,在使用他的方法訓(xùn)練時(shí),使得這種方法喪失了原有的靈活性和探索性。

  B.表演本質(zhì)上是自發(fā)的感情表達(dá),系統(tǒng)的訓(xùn)練經(jīng)常干涉它。

  C.實(shí)際上唯一需要給予年輕演員的建議是他們必須系統(tǒng)地抵制他們在戲劇表演中所表現(xiàn)出來的那些老套的模式。

  D.斯坦尼斯拉夫斯基的方法主要是針對那些必須克服有做作和不成熟表演行為的年輕演員。

  解析:斯坦尼斯拉夫斯基之所以未打算對表演問題提出嚴(yán)格解決方法的教科書,本意是不想使該方法失去靈活性和探索性。因此選擇A。

  33.已發(fā)現(xiàn)有國產(chǎn)嬰幼兒奶粉三聚氰胺含量超標(biāo)。

  如果上述斷定是真的,那么在下述三個(gè)斷定中不能確定真假的是:

 、、國產(chǎn)嬰幼兒奶粉沒有三聚氰胺含量不超標(biāo)的。

 、、有國產(chǎn)嬰幼兒奶粉三聚氰胺含量沒超標(biāo)。

 、、所有國產(chǎn)嬰幼兒奶粉三聚氰胺含量都未超標(biāo)。

  A.只有Ⅰ和Ⅱ。    B.Ⅰ、Ⅱ和Ⅲ。    C.只有Ⅰ和Ⅲ。    D.只有Ⅱ。

  解析:選項(xiàng)1不能確定真假,從部分不能推出整體;選項(xiàng)2不能確定真假,從一部分不能退出另一部分具有相反的結(jié)論。條件3明顯可以判定為假,選擇A。

  34.股民購買股票的目的就是盈利。股民購買股票后,股票價(jià)格上漲,就會盈利,否則,就會虧損。一般情況下,盈利或者虧損達(dá)到10%就應(yīng)該拋出,以避免那些不可預(yù)測的突然變化所引起的損失。在股市上,有經(jīng)驗(yàn)的股民往往會選擇跟莊戰(zhàn)術(shù)來買賣股票。即大股東買進(jìn)的時(shí)候,他們也買進(jìn),大股東拋出的時(shí)候,他們就拋出。因?yàn)榇蠊蓶|有可能掌握更多上市公司和市場行情方面的信息。這些天,天虹公司的股票一直在下跌,但是大股東們卻一直在大量購進(jìn)這只股票。

  以上事實(shí)最能支持以下哪種預(yù)測?

  A.天虹公司的股票價(jià)格會繼續(xù)下降,但速度會放慢。

  B.天虹公司的股票價(jià)格會在短期內(nèi)大幅度上漲。

  C.股民盲目跟莊不是一個(gè)好的盈利方法。

  D.最近大股東對天虹公司股價(jià)變動判斷失誤,有可能造成巨大經(jīng)濟(jì)損失。

  解析:題目結(jié)論如下:大股東買進(jìn)的股票會漲,因?yàn)樗麄冋莆樟烁嗌鲜泄竞褪袌鲂星榉矫娴男畔。所以選擇B。

  35.作為身體的精密平衡系統(tǒng)的一部分,人類心臟分泌某種激素,這些激素能夠控制血液中的含鹽量和體內(nèi)參與循環(huán)的血量。人體僅需要極少的這種激素。它在控制血壓時(shí)極其重要,并且在患心臟病的病人血中經(jīng)常檢出。

  基于上面的研究成果,下面的哪個(gè)論點(diǎn)一定正確?

  A.假如心臟分泌的這種激素缺乏,將導(dǎo)致低血壓。

  B.一個(gè)僅僅由機(jī)械閥制作的設(shè)備被用作人工心臟,它將不能執(zhí)行人類心臟的所有功能。

  C.少量這種激素的分泌對人體的影響是長期性的。

  D.任何控制血壓的藥將通過影響心臟此種激素分泌量達(dá)到效果。

  解析:選擇B,因?yàn)橛蓹C(jī)械閥制作的人工心臟,顯然不具有分泌激素的功能,因此就不能執(zhí)行人類心臟的所有功能,題干反復(fù)說明就是為了得出這樣的結(jié)論。

  36.某出版社近年來出版物的錯(cuò)字率較前幾年有明顯的增加,引起了讀者的不滿和有關(guān)部門的批評,這主要是由于該出版社大量引進(jìn)非專業(yè)編輯所致。當(dāng)然,近年來該社出版物的大量增加也是一個(gè)重要原因。

  上述議論的漏洞,也類似地出現(xiàn)在以下哪項(xiàng)中?

  I 美國航空公司近兩年來的投訴率比前幾年有明顯的下降。這主要是由于該航空公司在裁員整頓的基礎(chǔ)上,有效地提高了服務(wù)質(zhì)量。當(dāng)然,“9.11”事件后航班乘客數(shù)量的銳減也是一個(gè)重要原因。

  II 統(tǒng)計(jì)數(shù)字表明:近年來我國心血管病的死亡率,即由心血管病導(dǎo)致的死亡率在整個(gè)死亡人數(shù)中的比例,較前有明顯增加,這主要是由于隨著經(jīng)濟(jì)的發(fā)展,我國民眾的飲食結(jié)構(gòu)和生活方式發(fā)生了容易誘發(fā)心血管病的不良變化。當(dāng)然,由于心血管病主要是老年病,因此,我國人口的老齡化,即人口中老年人比例的增加也是一個(gè)重要原因。

  III 某市去年的高考錄取率比前年增加15%,這主要是由于各中學(xué)狠抓了教育質(zhì)量。當(dāng)然,另一個(gè)重要原因是,該市今年參加高考的人數(shù)比去年增加了20%。

  A.僅僅I和II。   B.僅僅II和III。   C.僅僅I和III。   D.僅僅I。

  解析:題目中的邏輯錯(cuò)誤是強(qiáng)加因果,出版物數(shù)量的增加與錯(cuò)字率的增加沒有因果關(guān)系。根據(jù)這一思路對選項(xiàng)進(jìn)行判斷,選項(xiàng)I航班數(shù)銳減與投訴率的下降沒有因果關(guān)系,與題目中犯了同樣的錯(cuò)誤;選項(xiàng)II,心血管病是老年病,我國人口老齡化,因此我國心血管死亡率增加,這是一個(gè)有效地推理過程;選項(xiàng)III,參加高考的人數(shù)增加,并不能推出錄取比例增加的這一結(jié)論。也是屬于沒有因果關(guān)系。另外從絕對和相對概念這個(gè)角度分析,同樣可以得到相似的結(jié)論。選C

  37.從前,一個(gè)孤島上有一個(gè)奇怪的風(fēng)俗:凡是漂流到這個(gè)島上的外鄉(xiāng)人都要作為祭品被殺掉,但允許被殺的人在臨死前說一句話,然后由這個(gè)島上的長老判定這句話是真的還是假的。如果說的是真話,則將這個(gè)外鄉(xiāng)人在真理之神面前殺掉;如果說的是假話,則將他在錯(cuò)誤之神面前殺掉。有一天,一位哲學(xué)家漂流到了這個(gè)島上,他說了一句話,使得島上的人沒有辦法殺掉他。

  該哲學(xué)家必定說了下面哪一句話?

  A.你們這樣做不合乎理性。    B.我將死在真理之神面前。

  C.無論如何我都會死。      D.我將死在錯(cuò)誤之神面前。

  解析:A選項(xiàng),可以判斷為真,會在真理之神面前殺掉;B選項(xiàng),可以判定為真,因?yàn)樗麜涝谡胬碇衩媲;選項(xiàng)C,同樣可以判定為真,選項(xiàng)D我將死在錯(cuò)誤之神面前,如果為真,那么說明他說的是假話,表明他將死在真理之神面前為真,這本身與這句話矛盾了,所以這句話自背反命題,選D。

  38.有的地質(zhì)學(xué)家認(rèn)為,如果地球的未勘探地區(qū)中單位面積的平均石油儲藏量能和已勘探地區(qū)一樣的話,那么,目前關(guān)于地下未開采的能源含量的正確估計(jì)因此要乘上一萬倍,由此可得出結(jié)論,全球的石油需求,至少可以在未來五個(gè)世紀(jì)中得到滿足,即便此種需求每年呈加速上升的趨勢。為使上述論證成立,以下哪項(xiàng)是必須假設(shè)的?

  A. 地球上未勘探地區(qū)的總面積是已勘探地區(qū)的一萬倍。

  B. 在未來至少五個(gè)世紀(jì)中,世界人口的增長率不會超過對石油需求的增長率。

  C. 新技術(shù)將使未來對石油的勘探和開采比現(xiàn)在更為可行。

  D. 地球上未勘探地區(qū)中儲藏的石油可以被勘測和開采出來。

  解析:答案為D,題目中通過假設(shè)得出了未來五個(gè)世紀(jì)的石油需求相會得到滿足的結(jié)論,假設(shè)條件為地球上未勘探地區(qū)的單位面積石油儲量與已勘探地區(qū)一樣多。但僅有這個(gè)前提還是不夠的,還需要這些為勘探的時(shí)候都是可以被開采出來的這一假設(shè)。

  39.某礦山發(fā)生了一起嚴(yán)重的安全事故。關(guān)于事故原因,甲乙丙丁四位負(fù)責(zé)人有如下斷定:

  甲:如果造成事故的直接原因是設(shè)備故障,那么肯定有人違反操作規(guī)程。

  乙:確實(shí)有人違反操作規(guī)程,但造成事故的直接原因不是設(shè)備故障

  丙:造成事故的直接原因確實(shí)是設(shè)備故障,但并沒有人違反操作規(guī)程。

  。涸斐墒鹿实闹苯釉蚴窃O(shè)備故障。

  如果上述斷定只有一個(gè)人的斷定為真,那么以下斷定都不可能為真,除了:

  A.甲的斷定為真,有人違反了操作規(guī)程。

  B.甲的斷定為真,但沒有人違反操作規(guī)程。

  C.乙的斷定為真。

  D.丙的斷定為真。

  解析:這道題屬于邏輯推理題,由于只有一個(gè)人判斷為真,因此丙、丁的判斷首先排除,因?yàn)檫@兩個(gè)判定并不沖突。甲、乙的推斷有矛盾,因此兩項(xiàng)必有一個(gè)是正確的。甲的推斷結(jié)構(gòu)是如果B推出A,那么 C必然推出B;乙的推斷結(jié)構(gòu)是C推出B,但B不能退出A,是甲判定的一個(gè)逆向命題。選項(xiàng)B是正確答案,因?yàn)檫x項(xiàng)B肯定了甲的推斷,同時(shí)否定了乙的發(fā)生,因此是真命題。其他選項(xiàng)均不正確。

  40.心臟的搏動引起血液循環(huán)。對同一個(gè)人,心率越快,單位時(shí)間進(jìn)入循環(huán)的血液量越多。血液中的紅血球運(yùn)輸氧氣。一般地說,一個(gè)人單位時(shí)間通過血液循環(huán)獲得的氧氣越多,他的體能及其發(fā)揮就越佳。因此,為了提高運(yùn)動員在體育比賽中的競技水平,應(yīng)該加強(qiáng)他們在高海拔地區(qū)的訓(xùn)練,因?yàn)樵诟吆0蔚貐^(qū),人體內(nèi)每單位體積血液中含有的紅血球數(shù)量,要高于在低海拔地區(qū)。

  以下哪項(xiàng)是題干的論證必須假設(shè)的?

  A.運(yùn)動員在高海拔地區(qū)的心率不低于在低海拔地區(qū)。

  B.不同運(yùn)動員的心率基本相同。

  C.運(yùn)動員的心率比普通人慢。

  D.在高海拔地區(qū)訓(xùn)練能使運(yùn)動員的心率加快。

  解析:此題論證說明運(yùn)動員在高海拔地區(qū)訓(xùn)練體內(nèi)的紅血球數(shù)量高于低海拔地區(qū),從而達(dá)到最佳體能的目的。B和C與題意不符,選A。

 41~45題基于以下題干:

  七個(gè)學(xué)生R、S、T、V、W、X、Y,被分成兩個(gè)學(xué)習(xí)小組。第一組有三名成員,第二組有四名成員。學(xué)生們的分組必須符合以下要求:

  R和T不能在同一個(gè)小組;

  如果S在第一組,那么V必須在第一組;

  如果W在第一組,那么T必須在第二組;

  X必須在第二組。

  41.如果W在第一組,那么以下哪項(xiàng)也一定在第一組?

  A.R    B.S    C.T    D.V

  解析:根據(jù)條件第一組有三名成員。因?yàn)镽和T不能在一組,又因?yàn)闂l件:如果W在第一組,那么T必須在第二組,則R可以在第一組

  42.如果T和Y都在第一組,那么以下哪項(xiàng)一定是真的?

  A.S和V在同一組。   B.S和W在同一組。   C.V和R在同一組。   D.W和T在同一組。

  解析:如果T和Y在第一組,R和T又不能在一組,可知R只能在第二組,而X必須在第二組。如果S在第一組,根據(jù)條件,V也需在第一組,而第一組只有三個(gè)成員,那么S和V就不能在同一組。而如果W在第一組,那么T必須在第二組。但是T已經(jīng)在第一組,則W必須在第二組,加之第二組有四人,那么S也需在第二組,那么S和W為一組。

  43.如果W和T在同一組,那么以下哪項(xiàng)可能在同一組,除了:

  A.R和S    B.S和Y    C.T和Y    D.V和Y

  答案:B

  解析:

  條件分析:

  1、R1且T2;或者 R2且T1

  2、S1→V1;即S2的時(shí)候,V可以自由分配。

  3、W1→T2;即W2的時(shí)候,T可以自由分配。

  4、X2

  43題的臨時(shí)題干要求WT在同一組,根據(jù)條件3,他們不可能在第一組,不然就違背了題設(shè)。所以,這個(gè)時(shí)候,WT都在都在第二組。第二組已經(jīng)有了WTX三個(gè)成員,只有一個(gè)空位了。根據(jù)條件1,可以知道,R在第一組。因此,除了跟WTX搭伴的元素外,任何二人組合都必須在第一組了。

  選項(xiàng)A,第一組RSV;第二組WTXY,有可能。

  選項(xiàng)B,第一組RSY;則第一組還必須有V,顯然超編了。因?yàn)榈谝唤M只能有三個(gè)人。因此,此項(xiàng)為答案。

  選項(xiàng)C,依然是第一組RSV;第二組WTXY這樣的組合。

  選項(xiàng)D,第一組RVY;第二組WTXS,不矛盾。

  44.如果V和Y在同一組,那么以下哪項(xiàng)一定是真的?

  A.R在第一組。    B.S在第一組。    C.W在第二組。    D.Y在第二組。

  解析:如果V和Y在第一組,那么根據(jù)條件如果S在第一組,那么V必須在第一組;說明S可以在第一組,又因?yàn)槿绻鸚在第一組,那么T必須在第二組,但是R和T又不能在一組,則R必須在第一組,W和T可以在第二組。則答案C正確。

  45.如果S在第一組,那么以下哪項(xiàng)一定是真的?

  A.T在第一組。    B.T在第二組。    C.Y在第一組。    D.Y在第二組。

  解析:如果S在第一組,根據(jù)條件V必須在第一組,如果W在第一組,那么T必須在第二組;R和T不能在同一個(gè)小組;那么W或R可以在第一組,X必須在第二組。因此在第一組的可以是S或V或R或W,因此答案D正確。

  46~50題基于以下題干:

  六位教授F、G、H、J、K、L,將評審馬、任、孫、吳博士的論文4篇。評審需遵守以下原則:

 。1)每位教授只評審一篇博士論文;

 。2)每篇博士論文至少有一位教授評審;

 。3)H與F評審?fù)黄┦空撐模?/p>

  (4)L只與其他教授中的一位同評一篇博士論文;

  (5)G評審馬博士的論文;

 。6)J評審馬博士或吳博士的論文;

 。7)H不評審吳博士的論文。

  46.如果K不評審孫博士的論文,那么以下哪項(xiàng)一定是真的?

  A.L評審馬博士的論文           B.L評審孫博士的論文

  C.F和H評審任博士的論文        D.F和H評審孫博士的論文

  解析:H不評審吳博士論文,H與F又評審?fù)黄撐模荒敲碒和F有可能評審馬或任或?qū)O博士的論文,但是G評審馬博士的論文,J評審馬博士或者吳博士的論文,那么H與F就有可能評審任或?qū)O博士的論文,K不評審孫博士的論文,K就有可能評審任博士的論文,那么H和F就會評審孫博士的論文。

  47.以下哪項(xiàng)完整地列出了L可能評審的博士論文?

  A.馬、任      B.馬、孫、吳     C.馬、任、吳       D.馬、任、孫、吳

  解析:根據(jù)條件四,L只與其他教授中的一位同評一篇博士論文,而每篇博士論文至少有一位教授評審,也就是說一篇論文有可能是兩個(gè)教授或者三個(gè)教授一起評論。因此,L可以評論馬、任、孫、吳的博士論文。

  48.以下哪項(xiàng)可能是真的?

  A.F和G評審馬博士的論文。

  B. F和L評審吳博士的論文。

  C. K評審吳博士的論文并且L評審馬博士的論文。

  D. L評審任博士的論文并且F評審孫博士的論文。

  解析:G評審馬博士的論文,J評審馬博士或吳博士的論文,H不評審吳博士的論文,H與F又評審?fù)黄撐;那么H和F有可能評審馬或任或?qū)O博士的論文,那么H與F就有可能評審任或?qū)O博士的論文。加之題一,K不評審孫博士的論文,K就有可能評審任博士的論文,條件三H與F評審?fù)黄┦空撐模敲碒和F就會評審孫博士的論文。條件四:L只與其他教授中的一位同評一篇博士論文,L可以與K評審任博士論文,故D正確

  49.以下哪項(xiàng)不可能是真的?

  A. L和G評審馬博士的論文        B. L和K評審馬博士的論文

  C. L和K評審任博士的論文        D. L和K評審孫博士的論文

  解析:根據(jù)條件(4)L只與其他教授中的一位同評一篇博士論文,而條件(5)G評審馬博士的論文,那么可知L和G可評審馬博士論文,故A正確。根據(jù)條件(6)J評審馬博士或吳博士的論文,可以得知L可以和J可以評審吳博士或者馬博士的論文,H不評審吳博士的論文,那么H可以評論任博士和孫博士的論文。K可以評論吳博士、任博士、孫博士的論文。加之條件(4),得知答案C和D正確,故B錯(cuò)誤

  50.如下哪位教授不可能審閱任博士的論文?

  A.F   B.K    C.J   D. L

  解析:條件(6)J評審馬博士或吳博士的論文;而條件(1)每位教授只評審一篇博士論文,根據(jù)兩個(gè)條件可以推斷J不可能審閱任或?qū)O博士的論文,故選C

 

OR 點(diǎn)我咨詢
標(biāo)簽:

上一篇:【在職研究生輔導(dǎo)資料】工程碩士復(fù)習(xí)資料 2
下一篇:【在職研究生輔導(dǎo)資料】工程碩士復(fù)習(xí)資料 4

    相關(guān)閱讀